Величина равная отношению работы электрического тока ко времени его прохождения называется

Работа и мощность электрического тока. Закон Джоуля-Ленца

1. Электрический ток, проходя по цепи, производит разные действия: тепловое, механическое, химическое, магнитное. При этом электрическое поле совершает работу, и электрическая энергия превращается в другие виды энергии: во внутреннюю, механическую, энергию магнитного поля и пр.

Как было показано, напряжение ​( (U) )​ на участке цепи равно отношению работы ​( (F) )​, совершаемой при перемещении электрического заряда ​( (q) )​ на этом участке, к заряду: ​( U=A/q )​. Отсюда ​( A=qU )​. Поскольку заряд равен произведению силы тока ​( (I) )​ и времени ​( (t) )​ ​( q=It )​, то ​( A=IUt )​, т.е. работа электрического тока на участке цепи равна произведению напряжения на этом участке, силы тока и времени, в течение которого совершается работа.

Единицей работы является джоуль (1 Дж). Эту единицу можно выразить через электрические единицы:

( [A] )​= 1 Дж = 1 В · 1 А · 1 с

Для измерения работы используют три измерительных прибора: амперметр, вольтметр и часы, однако, в реальной жизни для измерения работы электрического тока используют счётчики электрической энергии.

Если нужно найти работу тока, но при этом сила тока или напряжение неизвестны, то можно воспользоваться законом Ома, выразить неизвестные величины и рассчитать работу по формулам: ​( A=frac{U^2}{R}t )​ или ​( A=I^2Rt )​.

2. Мощность электрического тока равна отношению работы ко времени, за которое она совершена: ​( P=A/t )​ или ​( P=IUt/t )​; ​( P=IU )​, т.е. мощность электрического тока равна произведению напряжения и силы тока в цепи.

Единицей мощности является ватт (1 Вт): ​( [P]=[I]cdot[U] )​; ​( [P] )​ = 1 А · 1 В = 1 Вт.

Используя закон Ома, можно получить другие формулы для расчета мощности тока: ​( P=frac{U^2}{R};P=I^2R )​.

Значение мощности электрического тока в проводнике можно определить с помощью амперметра и вольтметра, измерив соответственно силу тока и напряжение. Можно для измерения мощности использовать специальный прибор, называемый ваттметром, в котором объединены амперметр и вольтметр.

3. При прохождении электрического тока по проводнику он нагревается. Это происходит потому, что перемещающиеся под действием электрического поля свободные электроны в металлах и ионы в растворах электролитов сталкиваются с молекулами или атомами проводников и передают им свою энергию. Таким образом, при совершении током работы увеличивается внутренняя энергия проводника, в нём выделяется некоторое количество теплоты, равное работе тока, и проводник нагревается: ​( Q=A )​ или ​( Q=IUt )​. Учитывая, что ​( U=IR )​, ​( Q=I^2Rt )​.

Количество теплоты, выделяющееся при прохождении тока но проводнику, равно произведению квадрата силы тока, сопротивления проводника и времени.

Этот закон называют законом Джоуля-Ленца.

Содержание

  • ПРИМЕРЫ ЗАДАНИЙ
    • Часть 1
    • Часть 2
  • Ответы

ПРИМЕРЫ ЗАДАНИЙ

Часть 1

1. Силу тока в проводнике увеличили в 2 раза. Как изменится количество теплоты, выделяющееся в нём за единицу времени, при неизменном сопротивлении проводника?

1) увеличится в 4 раза
2) уменьшится в 2 раза
3) увеличится в 2 раза
4) уменьшится в 4 раза

2. Длину спирали электроплитки уменьшили в 2 раза. Как изменится количество теплоты, выделяющееся в спирали за единицу времени, при неизменном напряжении сети?

1) увеличится в 4 раза
2) уменьшится в 2 раза
3) увеличится в 2 раза
4) уменьшится в 4 раза

3. Сопротивления резистор ​( R_1 )​ в четыре раза меньше сопротивления резистора ​( R_2 )​. Работа тока в резисторе 2

1) в 4 раза больше, чем в резисторе 1
2) в 16 раз больше, чем в резисторе 1
3) в 4 раза меньше, чем в резисторе 1
4) в 16 раз меньше, чем в резисторе 1

4. Сопротивление резистора ​( R_1 )​ в 3 раза больше сопротивления резистора ​( R_2 )​. Количество теплоты, которое выделится в резисторе 1

1) в 3 раза больше, чем в резисторе 2
2) в 9 раз больше, чем в резисторе 2
3) в 3 раза меньше, чем в резисторе 2
4) в 9 раз меньше, чем в резисторе 2

5. Цепь собрана из источника тока, лампочки и тонкой железной проволоки, соединенных последовательно. Лампочка станет гореть ярче, если

1) проволоку заменить на более тонкую железную
2) уменьшить длину проволоки
3) поменять местами проволоку и лампочку
4) железную проволоку заменить на нихромовую

6. На рисунке приведена столбчатая диаграмма. На ней представлены значения напряжения на концах двух проводников (1) и (2) одинакового сопротивления. Сравните значения работы тока ​( A_1 )​ и ​( A_2 )​ в этих проводниках за одно и то же время.

1) ​( A_1=A_2 )
2) ( A_1=3A_2 )
3) ( 9A_1=A_2 )
4) ( 3A_1=A_2 )

7. На рисунке приведена столбчатая диаграмма. На ней представлены значения силы тока в двух проводниках (1) и (2) одинакового сопротивления. Сравните значения работы тока ( A_1 )​ и ​( A_2 ) в этих проводниках за одно и то же время.

1) ​( A_1=A_2 )
2) ( A_1=3A_2 )
3) ( 9A_1=A_2 )
4) ( 3A_1=A_2 )

8. Если в люстре для освещения помещения использовать лампы мощностью 60 и 100 Вт, то

А. Большая сила тока будет в лампе мощностью 100 Вт.
Б. Большее сопротивление имеет лампа мощностью 60 Вт.

Верным(-и) является(-ются) утверждение(-я)

1) только А
2) только Б
3) и А, и Б
4) ни А, ни Б

9. Электрическая плитка, подключённая к источнику постоянного тока, за 120 с потребляет 108 кДж энергии. Чему равна сила тока в спирали плитки, если её сопротивление 25 Ом?

1) 36 А
2) 6 А
3) 2,16 А
4) 1,5 А

10. Электрическая плитка при силе тока 5 А потребляет 1000 кДж энергии. Чему равно время прохождения тока по спирали плитки, если её сопротивление 20 Ом?

1) 10000 с
2) 2000 с
3) 10 с
4) 2 с

11. Никелиновую спираль электроплитки заменили на нихромовую такой же длины и площади поперечного сечения. Установите соответствие между физическими величинами и их возможными изменениями при включении плитки в электрическую сеть. Запишите в таблицу выбранные цифры под соответствующими буквами. Цифры в ответе могут повторяться.

ФИЗИЧЕСКАЯ ВЕЛИЧИНА
A) электрическое сопротивление спирали
Б) сила электрического тока в спирали
B) мощность электрического тока, потребляемая плиткой

ХАРАКТЕР ИЗМЕНЕНИЯ
1) увеличилась
2) уменьшилась
3) не изменилась

12. Установите соответствие между физическими величинами и формулами, по которым эти величины определяются. Запишите в таблицу выбранные цифры под соответствующими буквами.

ФИЗИЧЕСКИЕ ВЕЛИЧИНЫ
A) работа тока
Б) сила тока
B) мощность тока

ФОРМУЛЫ
1) ​( frac{q}{t} )
2) ​( qU )
3) ( frac{RS}{L} )
4) ​( UI )
5) ( frac{U}{I} )

Часть 2

13. Нагреватель включён последовательно с реостатом сопротивлением 7,5 Ом в сеть с напряжением 220 В. Каково сопротивление нагревателя, если мощность электрического тока в реостате составляет 480 Вт?

Ответы

Работа и мощность электрического тока. Закон Джоуля-Ленца

3.1 (62.67%) 150 votes

  • Взрослым: Skillbox, Geekbrains, Хекслет, Eduson, XYZ, Яндекс.
  • 8-11 класс: Умскул, Лектариум, Годограф, Знанио.
  • До 7 класса: Алгоритмика, Кодланд, Реботика.
  • Английский: Инглекс, Puzzle, Novakid.

Работа и мощность постоянного тока

Электрическое поле совершает работу, когда перемещает свободные заряды вдоль проводника. Эта работа называется работой тока.

Возьмем произвольный участок цепи (например, нить лампы накаливания). Пусть за некоторый промежуток времени Δt через поперечное сечение проводника проходит заряд Δq. Электрическое поле при таком передвижении совершает работу А = ΔqU, где U – напряжение между концами рассматриваемого участка проводника.

Сила тока рассчитывается по формуле: I = Δq/Δt. Следовательно, работа тока: А = IUΔt.

Работа тока на участке цепи – есть произведение напряжения, силы тока и времени, в течение которого шел ток.

Закон сохранение энергии говорит, что работа должна быть равна разности конечной и начальной энергии. Тогда получается, что энергия, выделяемая на рассматриваемом участке цепи за время Δt аналогична величине работы тока.

Представим ситуацию, когда на участке цепи не совершается механическая работа, не изменяется химический состав проводника, но происходит нагревание. Вспомним, что нагревание – это увеличение внутренней энергии, которая передается внешним телам в виде энергии.

Рассмотрим более подробно нагревание проводника. Электрическое поле разгоняет электроны в определенном направлении. Они, в свою очередь, сталкиваются с ионами кристаллической решетки, передавая энергию. Энергия около положений равновесия возрастает, и внутренняя энергия увеличивается.

Температура обозначает степень нагретости тел – меру кинетической энергии. Следовательно, когда температура самого проводника увеличивается, то окружающие тела начинают получать от него тепло. Когда цепь замыкается, температура перестает расти и стабилизируется.

Электрическое поле продолжает работать в полости проводника, что провоцирует выделение энергии. Но в этом случае остается постоянной внутренняя энергия, потому что проводник отдает в окружающую среду количество теплоты, равное работе тока.

Соберем все вышесказанное воедино и получим формулу:

A = IUΔt = I2RΔt = U2Δt/R = Q.

При последовательном соединении проводников работа поля рассчитывается как: A = I2RΔt.

При параллельном: A = U2Δt/R (с учетом того, что напряжение на всех проводниках одинаково).

Закон Джоуля-Ленца

Какое количество теплоты выделяет проводник в окружающую среду? На этот вопрос ответили английский ученый Д. Джоуль и русский ученый Э.Х. Ленц.

Закон Джоуля-Ленца звучит так: количество теплоты, выделяемое в проводнике с током, равно произведению квадрата силы тока, сопротивления проводника и времени прохождения тока по проводнику: Q = I2RΔt.

Мощность тока

Каждый электрический прибор потребляет столько энергии, сколько ему нужно.

Мощность тока – есть отношение работы тока ко времени его прохождения:

P = A/Δt или, через закон Ома для участка цепи: P = IU = I2R = U2/R.

Мощность тока выражается в ваттах (Вт).

  • Взрослым: Skillbox, Geekbrains, Хекслет, Eduson, XYZ, Яндекс.
  • 8-11 класс: Умскул, Лектариум, Годограф, Знанио.
  • До 7 класса: Алгоритмика, Кодланд, Реботика.
  • Английский: Инглекс, Puzzle, Novakid.

Работа электрического тока в цепи определяется по формулам: $A = Uq$ и $A = UIt$. Но часто, кроме самой работы, нам важна скорость ее выполнения. В механике у нас была такая величина — мощность. 

Что называют мощностью? Как рассчитать мощность? 

Мощность — это физическая величина, равная отношению работы ко времени, за которое она была совершена. Она определяется по формуле: $N = frac{A}{t}$.

На данном уроке мы рассмотрим мощность как величину, характеризующую работу именно электрического тока.

Мощность тока и ее связь с напряжением и силой тока

В электричестве мощность обозначается буквой $P$, а не $N$. При этом смысл этой величины остается тем же. Эта величина численно равна работе, которая совершается в единицу времени:
$P = frac{A}{t}$, где $P$ — мощность электрического тока.

Как рассчитать мощность электрического тока через напряжение и силу тока?

Вы уже знаете, что работа электрического тока определяется по формуле: $A = UIt$. Подставим это выражение в определение мощности:
$P = frac{A}{t} = frac{UIt}{t} = UI$.

Мощность электрического тока — это величина, численно равная произведению напряжения на силу тока:
$P = UI$.

Единицы измерения мощности тока

Что принимают за единицу мощности?

Единицей мощности является $1 space ватт$ ($Вт$).

Из формулы $P = frac{A}{t}$ мы получим, что $1 space Вт = 1 frac{Дж}{с}$.

Как выражается единица мощности через единицы напряжения и силы тока?

Из формулы $P = UI$ следует:

$1 space ватт = 1 space вольт cdot 1 space ампер$,
$1 space Вт = 1 space В cdot А$.

Кратные единицы мощности

На практике часто используют кратные единицы мощности для удобства. К ним относятся гектоватт ($гВт$), киловатт ($кВт$) и мегаватт ($МВт$).

$1 space гВт = 100 space Вт$,
$1 space кВт = 1000 space Вт$,
$1 space МВт = 1 space 000 space 000 space Вт$.

Измерение мощности электрического тока

Мощность электрического тока напрямую зависит от напряжения и силы тока в цепи. Соответственно, для того, чтобы определить мощность тока, нам понадобится два прибора: амперметр и вольтметр. Умножив показания этих приборов друг на друга, мы получим численное значение мощности.

Также для измерения мощности напрямую существуют специальные приборы — ваттметры (рисунок 1). Они непосредственно измеряют мощность электрического тока в цепи.

Рисунок 1. Лабораторный ваттметр

Мощность, потребляемая некоторыми приборами

В таблице 1 представлены значения мощности для некоторых приборов. Для бытовых приборов она всегда указывается в паспорте каждого устройства.

Устройство Потребляемая мощность $P$, $Вт$
Лампа карманного фонаря 1
Лампа накаливания 40-200
Холодильник 160
Кондиционер 800
Утюг 1200-2200
Стиральная машина 2200
Пылесос 1500-3000
Лампа звезды башни Кремля 5000
Электропоезд 6 500 000
Таблица 1. Значения мощности тока для некоторых приборов и устройств

Упражнения

Упражнение №1

В цепь с напряжением в $127 space В$ включена электрическая лампа, сила тока в которой равна $0.6 space А$. Найдите мощность тока в лампе.

Дано:
$U = 127 space В$
$I = 0.6 space А$

$P — ?$

Посмотреть решение и ответ

Скрыть

Решение:

Мощность электрического тока в лампе рассчитывается по формуле: $P = UI$.

$P = 127 space В cdot 0.6 space А = 76.2 space Вт$.

Ответ: $P = 76.2 space Вт$.

Упражнение №2

Электроплитка рассчитана на напряжение $220 space В$ и силу тока $3 space А$. Определите мощность тока в плитке.

Дано:
$U = 220 space В$
$I = 3 space А$

$P — ?$

Посмотреть решение и ответ

Скрыть

Решение:

Мощность электрического тока в плитке рассчитаем по формуле: $P = UI$.

$P = 220 space В cdot 3 space А = 660 space Вт$. 

Ответ: $P = 660 space Вт$.

Упражнение №3

Пользуясь таблицей 1, вычислите, какую работу совершает за $1 space ч$ электрический ток в лампе карманного фонаря, осветительной лампе мощностью $200 space Вт$, в лампе звезды башни Кремля.

Дано:
$t = 1 space ч$
$P_1 = 1 space Вт$
$P_2 = 200 space Вт$
$P_3 = 5000 space Вт$

СИ:
$t = 3600 space с$

$A_1 — ?$
$A_2 — ?$
$A_3 — ?$

Посмотреть решение и ответ

Скрыть

Решение:

Мощность тока по определению равна работе, которую ток совершает за единицу времени: $P = frac{A}{t}$.

Выразим отсюда работу и рассчитаем ее для каждой лампы:
$A = Pt$.

Работа тока в лампе карманного фонаря:
$A_1 = P_1 t$,
$A_1 = 1 space Вт cdot 3600 space с = 3600 space Дж = 3.6 space кДж$.

Работа тока в осветительной лампе:
$A_2 = P_2 t$,
$A_2 = 200 space Вт cdot 3600 space с = 720 space 000 space Дж = 720 space кДж$.

Работа тока в лампе звезды башни Кремля:
$A_3 = P_3 t$,
$A_3 = 5000 space Вт cdot 3600 space с = 18 space 000 space 000 space Дж = 18 space МДж$.

Ответ: $A_1 = 3.6 space кДж$, $A_2 = 720 space кДж$, $A_3 = 18 space МДж$.

Упражнение №4

Рассмотрите один-два электроприбора, используемые в квартире. Найдите по паспорту приборов их мощность. Определите работу тока в них за $10 space мин$.

Если вы не можете найти паспорт прибора, внимательно рассмотрите его. Часто производители указывают мощность на самом устройстве. Мы возьмем пылесос мощностью $2000 space Вт$ и фен для волос мощностью $2200 space Вт$ (рисунок 2).

Рисунок 2. Бытовые приборы с известной мощностью

Дано:
$t = 10 space мин$
$P_1 = 2000 space Вт$
$P_2 = 2200 space Вт$

СИ:
$t = 600 space с$

$A_1 — ?$
$A_2 — ?$

Посмотреть решение и ответ

Скрыть

Решение:

Мощность тока по определению равна работе, которую ток совершает за единицу времени: $P = frac{A}{t}$.

Выразим отсюда работу и рассчитаем ее для каждого прибора:
$A = Pt$.

Работа тока в пылесосе, совершенная за $10 space мин$ его использования:
$A_1 = P_1t$,
$A_1 = 2000 space Вт cdot 600 space с = 1 space 200 space 000 space Дж = 1.2 space МДж$.

Работа тока в фене для волос, совершенная за $10 space мин$ его использования:
$A_2 = P_2t$,
$A_2 = 2200 space Вт cdot 600 space с = 1 space 320 space 000 space Дж = 1.32 space МДж$.

Ответ: $A_1 = 1.2 space МДж$, $A_2 = 1.32 space МДж$.

Физика, 10 класс

Урок 30. Закон Джоуля — Ленца. ЭДС

Перечень вопросов, рассматриваемых на уроке:

1) Работа электрического тока;

2) Мощность электрического тока;

3) Закон Джоуля — Ленца;

4) Сторонние силы;

5) Электродвижущая сила.

Глоссарий по теме

Работа тока на участке цепи равна произведению силы тока, напряжения на этом участке и времени, в течении которого совершалась работа.

Мощность тока равна отношению работы тока ко времени прохождения тока.

Количество теплоты, выделяемое проводником с током, равно произведению квадрата силы тока, сопротивления проводника и времени прохождения тока.

Любые силы, действующие на электрически заряженные частицы, за исключением электростатических (кулоновских) сил, называются сторонними силами.

Электродвижущая сила (ЭДС) в замкнутом проводящем контуре равна отношению работы сторонних сил по перемещению заряда вдоль контура к этому заряду.

Основная и дополнительная литература по теме урока:

Обязательная литература:

1. Г.Я. Мякишев., Б.Б.Буховцев., Н.Н.Сотский. Физика.10 класс. Учебник для общеобразовательных организаций М.: Просвещение, 2017. – С. 343 – 347.

Рымкевич А.П. Сборник задач по физике. 10-11 класс. — М.: Дрофа,2009.- 68 – 74.

Дополнительная литература.

http://kvant.mccme.ru/1972/10/zakon_dzhoulya-lenca.htm

Основное содержание урока

При упорядоченном движении заряженных частиц в проводнике электрическое поле совершает работу, равную произведению заряда, прошедшего через проводник, и напряжения.

Сила тока равна отношению заряда прошедшего через проводник ко времени прохождения

Выразим заряд из формулы силы тока

через силу тока и время:

после подстановки в формулу (1) получим

Работа тока на участке цепи равна произведению силы тока, напряжения и времени, в течение которого шёл ток.

Из закона Ома для участка цепи выразим напряжение через силу тока и напряжение

и подставив в формулу работы получим:

При последовательном соединении проводников для определения работы тока удобнее пользоваться этой формулой, так как сила тока одинакова во всех проводниках.

При параллельном соединении проводников формулой:

так как напряжение на всех проводниках одинаково.

Работа тока показывает, сколько электроэнергии превратилось в другие виды энергии за конкретный период времени. Для электроэнергии справедлив закон сохранения энергии.

Мощность определяется по формуле:

Мощность тока равна отношению работы тока ко времени прохождения тока.

Так же формулу для мощности можно переписать в нескольких эквивалентных формах:

Если на участке цепи не совершается механическая работа и ток не производит химических действий, то происходит только нагревание проводника.

Электрическое поле действует с силой на свободные электроны, которые начинают упорядоченно двигаться, одновременно участвуя в хаотическом движении, ускоряясь в промежутках между столкновениями с ионами кристаллической решетки. Во время этих столкновений расходуется кинетическая энергия заряженных частиц. Именно эта энергия и становится теплом. Последующие столкновения электронов с другими ионами увеличивают амплитуду их колебаний и соответственно температуру всего проводника.

В неподвижных металлических проводниках вся работа тока идет на увеличение их внутренней энергии:

Количество теплоты, выделяемое проводником, по которому течет ток, равно работе тока.

Количество теплоты, выделяемое проводником с током, равно произведению квадрата силы тока, сопротивления проводника и времени прохождения тока по проводнику:

При последовательном соединении большее количество теплоты выделяется в проводнике с большим сопротивлением, а при параллельном соединении – с меньшим.

Измерения, приводящие к закону Джоуля-Ленца, можно выполнить, поместив в калориметр с водой проводник с известным сопротивлением и пропуская через него ток определенной силы в течение известного времени. Количество выделяющейся при этом теплоты определяют, составив уравнение теплового баланса.

Если соединить проводником два металлических шарика, несущих заряды противоположных знаков, под влиянием электрического поля этих зарядов в проводнике возникает кратковременный электрический ток. Заряды быстро нейтрализуют друг друга, и электрическое поле исчезнет.

Чтобы ток был постоянным, надо поддерживать постоянное напряжение между шариками. Для этого необходимо устройство, которое перемещало бы заряды от одного шарика к другому в направлении, противоположном направлению сил, действующих на эти заряды со стороны электрического поля шариков. В таком устройстве на заряды, должны действовать силы неэлектростатического происхождения. Одно лишь электрическое поле заряженных частиц не способно поддерживать постоянный ток в цепи.

Любые силы, действующие на электрически заряженные частицы, за исключением сил электростатического происхождения (то есть кулоновских), называют сторонними силами. Необходимости сторонних сил для поддержания постоянного тока в цепи объясняет закон сохранения энергии.

Электростатическое поле потенциально. Работа этого поля при перемещении в нем заряженных частиц вдоль замкнутой электрической цепи равна нулю. Прохождение же тока по проводникам сопровождается выделением энергии — проводник нагревается. Следовательно, в цепи должен быть какой-то источник энергии, поставляющий ее в цепь. Работа этих сил вдоль замкнутого контура отлична от нуля. Внутри источника тока заряды движутся под действием сторонних сил против кулоновских сил (электроны от положительно заряженного электрода к отрицательному), а во внешней цепи их приводит в движение электрическое поле.

Действие сторонних сил характеризуется важной физической величиной, называемой электродвижущей силой (сокращенно ЭДС).

Электродвижущая сила источника тока равна отношению работы сторонних сил при перемещении заряда по замкнутому контуру к величине этого заряда:

Электродвижущую силу выражают в вольтах.

Разбор тренировочных заданий

1. Электрочайник со спиралью нагревательного элемента сопротивлением 30 Ом включен в сеть напряжением 220 В. Какое количество теплоты выделится в нагревательном элемента за 5 мин?

1) 7260000 Дж;

2) 2200 Дж;

3) 484000 Дж.

Дано:

R=30Ом

U=220B

t=5мин=300с

Найти Q-?

Решение. Количество теплоты выделяемой нагревательным элементом определяется законом Джоуля – Ленца:

Правильный ответ 3) 484000 Дж.

2. Определите работу сторонних сил при перемещении по проводнику заряда 10 Кл, если ЭДС равно 9 В. Ответ округлите до десятых.

Дано:

q=10Кл

=9В

Найти: Аст

Решение. Из формулы ЭДС выражаем

Правильный ответ: 90 Дж.

Сила тока


Сила тока

4.6

Средняя оценка: 4.6

Всего получено оценок: 90.

Обновлено 19 Июля, 2021

4.6

Средняя оценка: 4.6

Всего получено оценок: 90.

Обновлено 19 Июля, 2021

Электричество играет огромную роль в современном мире. Многие бытовые устройства бесполезны, когда отсутствует электроэнергия. При этом любое из них требует ток определенного напряжения и силы, эти значения практически всегда есть на шильдиках устройств. В данной статье мы поговорим о том, что такое сила электрического тока, дадим её определение, приведём формулу силы тока.

Физический смысл силы электрического тока

Электрический ток — это направленное движение заряженных частиц. При таком движении заряды совершают полезную работу. Очевидно, что чем быстрее заряды проходят по проводнику, тем совершённая работа будет больше.

Движение зарядов по проводнику

Рис. 1. Движение зарядов по проводнику

Однако использование обычной линейной скорости движения в случае электрического тока недостаточно. Дело в том, что ток — это движение большого числа носителей заряда, и их число может быть различно для одной и той же работы. Необходимо учитывать эту разницу наряду с линейной скоростью движения зарядов.

Поэтому гораздо целесообразнее для характеристики движения зарядов использовать не их линейную скорость, а скорость протекания общего заряда через сечение проводника. Чем больший заряд пройдёт через сечение проводника за единицу времени, тем больше «скорость» электрического тока, тем большую работу он сможет совершить.

Величина, равная отношению заряда, прошедшего по проводнику, ко времени прохождения, называется силой тока (для обозначения используется латинская буква $I$):

$$I={Delta q over Delta t}$$

Из данной формулы можно вывести единицу измерения силы тока. Поскольку заряд измеряется в кулонах, а время — в секундах, то единица силы тока равна кулону в секунду, или амперу (в честь физика А. Ампера):

$$[I]={Кл over с}=А$$

Сила тока

Рис. 2. Сила тока.

Постоянный и переменный ток

Поскольку сила тока — это фактически скорость протекания заряда по проводнику, эта величина, как и обычная линейная скорость, может быть мгновенной, изменяющейся или средней. Если величина $Delta t$ при расчете близка к нулю, то сила тока получается мгновенной. Если эта величина значительно больше нуля, то сила тока получается средней за это время.

Как и обычная линейная скорость, сила тока может расти, уменьшаться и даже становиться отрицательной. Отрицательная сила тока означает, что заряды движутся в противоположную сторону относительно направления, принятого в проводнике за положительное.

Если мгновенное значение силы тока не меняет знак, ток называется постоянным. Если изменение знака силы тока происходит, такой ток называется переменным.

Постоянный ток может быть:

  • «строго постоянным» — когда мгновенное значение не меняется;
  • изменяющимся — если мгновенное значение изменяется, но не до нуля;
  • пульсирующим — если мгновенное значение падает до нуля.

Для переменного тока принято считать, что мгновенные положительные и отрицательные значения силы тока равны, и такой переменный ток называется симметричным. Если мгновенные положительные и отрицательные значения силы тока не равны, то такой ток называется «ассиметричным». Он рассматривается как сумма постоянного и симметричного переменного токов.

Отметим, что поскольку работа электрического тока происходит при любом движении зарядов, независимо от направления, приведённая формула силы тока не годится для нахождения средней силы переменного тока. Если ток переменный, то заряды фактически не движутся по проводнику, а только колеблются с некоторой амплитудой. Получается, что в среднем через сечение проводника заряды не идут, а работа тока при этом все равно происходит. Сила переменного тока вычисляется с помощью специальных формул, учитывающих изменение направление движения зарядов по проводнику.

Постоянный и переменный ток

Рис. 3. Постоянный и переменный ток.

Заключение

Что мы узнали?

Сила тока — это физическая величина, характеризующая скорость прохождения заряда через сечение проводника. Она равна отношению заряда, прошедшего через проводник, ко времени прохождения. Если мгновенная сила тока не меняется, такой ток называется постоянным. Ток, в котором сила меняет знак, называется переменным.

Тест по теме

Доска почёта

Доска почёта

Чтобы попасть сюда — пройдите тест.

    Пока никого нет. Будьте первым!

Оценка доклада

4.6

Средняя оценка: 4.6

Всего получено оценок: 90.


А какая ваша оценка?

Содержание:

Электрический ток:

Для работы электроприборов необходим электрический ток. Фонарики, электронные часы, аудиоплейеры, радиоприёмники и телефоны используют электроэнергию батареек или аккумуляторов. Лампы, холодильники, телевизоры, пылесосы работают от электросети, то есть получают электроэнергию по проводам с электростанции.

Что же такое электрический ток и что нужно, чтобы он возник и существовал необходимое время?

Слово «ток» означает движение или течение чего-либо. А что может перемещаться в проводниках, соединяющих батарейку с лампой, холодильник — с электростанцией?

Явление электризации тел обусловлено наличием в них электрически заряженных частиц — электронов, а также положительных и отрицательных ионов, всегда находящихся в состоянии беспорядочного теплового движения (рис. 28). Есть много веществ, в которых при определённых условиях заряженные частицы могут свободно перемещаться на значительные расстояния по всему объёму тела. Например, в технике давно применяют металлические проводники, в которых носителями электричества являются свободные электроны. Если на все свободные заряженные частицы действовать какой-либо силой в одном направлении, то их беспорядочное перемещение дополнится движением в направлении приложенной силы. Говорят, что в теле возникает электрический ток.

Электрический ток — это упорядоченное (направленное) движение заряженных частиц.

Если в объёме проводника создать электрическое поле, то под его действием свободные заряженные частицы будут перемещаться в направлении приложенных к ним электрических сил, то есть в проводнике возникает электрический ток (рис. 29). Например, в проводнике, соединяющем шарик заряженного электрометра с Землёй, возникает электрическое поле, а вместе с ним и электрический ток, который прекращается, как только весь заряд шарика, образующий электрическое поле, переходит в землю.

Чтобы электрический ток в проводнике протекал как можно дольше, в нём необходимо постоянно поддерживать электрическое поле, то есть обеспечивать на одном конце проводника избыток зарядов определённого знака, а на другом — их недостаток (дефицит). Такое постоянное распределение зарядов на концах проводника создаётся и поддерживается источниками электрического тока.

Источниками электрического тока называют устройства, в которых происходит преобразование энергии определённого вида в электрическую энергию.

В каждом источнике тока выполняется работа по разделению положительно и отрицательно заряженных частиц, которые накапливаются на полюсах источника. Соответственно эти полюса условно обозначают знаками «+» и «-».

На практике используют разнообразные источники электрического тока. По видам преобразуемой энергии их разделяют на: химические (гальванические элементы, аккумуляторы), световые (фотоэлементы, солнечные батареи), тепловые (термоэлементы), механические (электрофорная машина, генераторы электрического тока разного рода).

Если к гальваническому элементу с помощью проводников присоединить электрическую лампу (рис. 30), то под действием электрического поля заряженные частицы в проводнике приходят в движение, возникает электрический ток, лампа загорается.

Электрический ток - определение и понятия с примерами
Рис. 30

Первый химический источник тока создал в 1799 г. итальянский физик Алессандро Вольта и назвал его гальваническим элементом в честь основателя учения об электричестве Луиджи Гальвани. Этот элемент давал напряжение около 1 вольта (1 В). Для получения более высокого напряжения А. Вольта собрал батарею (так называемый вольтов столб) из 20 цинковых, 20 медных и 20 суконных кружков, сложенных один на другой (рис. 31).

Электрический ток - определение и понятия с примерами
Рис. 31

В гальванических элементах происходят химические реакции, благодаря которым выполняется работа по разделению разных зарядов, то есть химическая энергия преобразуется в электрическую.

Опыт. Опустим в раствор серной кислоты две пластины — цинковую и медную. Получим простейший гальванический элемент (рис. 32). В нём происходит перераспределение положительно и отрицательно заряженных частиц вещества. В результате обе пластины электризуются, и между ними возникает электрическое поле. Эти пластины называются электродами (полюсами) источника тока.

Электрический ток - определение и понятия с примерами
Рис. 32

Гальванический элемент состоит из цинкового стакана 1, заполненного желеобразной смесью (рис. 33). В смесь вставлен угольный стержень 2. Сверху стакан залит слоем смолы 3.

Электрический ток - определение и понятия с примерами
Рис. 33

В результате химических реакций цинковый стакан становится отрицательно заряженным (отрицательный электрод), а угольный стержень — положительно заряженным (положительный электрод). Между электродами возникает электрическое поле. Если положительный и отрицательный электроды соединить проводником, то в нём возникнет электрический ток.

Несколько гальванических элементов можно соединить в батарею. Если необходимо получить большее напряжение, то используют последовательное соединение элементов: отдельные элементы соединяют между собой разноимёнными полюсами. На рисунке 34 изображена батарея из трёх элементов, в которой стержень первого элемента соединён с цинковым стаканом второго, а угольный стержень второго — со стаканом третьего элемента. Цинковые стаканы изолированы один от другого. От цинкового стакана первого элемента и угольного стержня третьего отведены две жестяные полоски: первая — отрицательный полюс батареи, вторая — положительный.

Электрический ток - определение и понятия с примерами
Рис. 34

Для получения большего тока используют параллельное соединение элементов: отдельные элементы соединяют в батарею одноимёнными полюсами, то есть корпус — с корпусом, а стержень — со стержнем. Аккумулятор (от латинского аккумуле — накапливаю) — это химический источник, в котором электрическая энергия накапливается при пропускании электрического тока в кислотном или щелочном растворе.

Аккумуляторы бывают кислотные и щелочные. Кислотный аккумулятор состоит из однородных электродов (рис. 35), например свинцовых пластин, помещённых в раствор серной кислоты. В щелочных аккумуляторах электроды изготовлены из разных металлов, например железа и никеля, и опущены в раствор едкой щёлочи. Для того чтобы аккумулятор стал источником тока, его необходимо «зарядить». Для этого через него пропускают ток от какого-либо другого источника. В процессе зарядки и в результате химических реакций один электрод заряжается положительно, а другой — отрицательно. После зарядки аккумулятора его можно использовать как самостоятельный источник тока. Полюса аккумуляторов обозначены знаками «+» и «-». При зарядке аккумулятора его положительный полюс соединяют с положительным полюсом источника тока, а отрицательный — с отрицательным.
Аккумуляторы используют для запуска автомобильных двигателей (рис. 36), освещения автомобилей и железнодорожных вагонов. Они питают электроэнергией подводные лодки. Аппаратура на искусственных спутниках, космических кораблях и станциях также питается от установленных на них аккумуляторов, которые заряжаются от солнечных батарей.

Под действием света, падающего на поверхность пластин из селена или кремния, в них происходит перераспределение положительных и отрицательных электрических зарядов (рис. 37). На этом основаны конструкция и действие солнечных батарей (рис. 38), в которых происходит прямое преобразование энергии солнечного излучения в электрическую энергию.

Электрический ток - определение и понятия с примерами
Рис. 38

В Институте полупроводников HAH Украины разработаны солнечные батареи с КПД 18 %, то есть близкие к максимально возможному. А учёные Национального технического университета «КПИ» использовали солнечные батареи для создания фотоэлектрической станции мощностью 5 кВт.

Если спаять две проволоки, изготовленные из разных металлов, а место спая нагреть, то в проводах возникнет электрический ток (рис. 39). Такой источник тока называют термоэлементом, или термопарой. В нём внутренняя энергия нагревателя преобразуется в электрическую энергию.
Электрический ток - определение и понятия с примерами
Рис. 39

В элекгрофорной машине два диска из органического стекла (с размещёнными по кругу металлическими полосками) вращаются в противоположных направлениях. В результате трения проволочных щёток о полоски на кондукторах (полюсах) машины накапливаются заряды противоположных знаков (рис. 7 на с. 10). Механическая энергия вращения дисков преобразуется в электрическую энергию.
На тепловых, атомных, ветровых и гидроэлектростанциях электрический ток вырабатывают с помощью генераторов электрического тока (от латинского слова генератор — производитель, создатель). Об устройстве и действии генераторов вы узнаете позднее. 

Электрическая цепь и её составляющие

Электрическая энергия, сосредоточенная в источниках электрического тока, — очень выгодный и удобный вид энергии, поэтому широко применяется в промышленности, технике, быту. Электродвигатели, электрические лампы, нагреватели, плиты, телевизоры и компьютеры называют приёмниками, или потребителями электрической энергии.

Чтобы доставить электрическую энергию от источника к потребителю, их соединяют между собой проводниками электрического тока. Для этого используют преимущественно медные или алюминиевые провода.

Простейшая электрическая цепь состоит из источника тока (рис. 40), потребителя электроэнергии (лампа, электродвигатель), соединительных проводов и устройства для замыкания и размыкания цепи — выключателя (ключа).

Электрический ток - определение и понятия с примерами
Рис. 40

Для того чтобы в цепи проходил ток, она должна быть замкнутой, то есть состоять только из проводников электрического тока. Если в каком-либо месте отсоединить провод или случится его обрыв, то в цепи тока не будет. На этом основано действие выключателей.

Чертёж, на котором изображают различные способы соединения элементов электрической цепи, называют схемой электрической цепи.

Приборы и соединения на схемах изображают с помощью условных обозначений. Некоторые из них приведены на странице I форзаца.

На рисунке 41 изображена схема простейшей электрической цепи.

Электрический ток - определение и понятия с примерами
Рис. 41

Электрический ток в металлах

Электрический ток — это упорядоченное движение свободных электрических зарядов под действием электрического поля источника тока. Какие это заряды? Как они перемещаются ?

Рассмотрим внутреннее строение металлических проводников. В каждом металле часть электронов покидает своё место в атоме, при этом атом превращается в положительный ион. Положительные ионы в металлах размещаются в строгом порядке, образуя кристаллические решётки (рис. 42). Между ионами хаотически движутся свободные электроны в виде электронного газа.

Электрический ток - определение и понятия с примерами
Рис. 42

Отрицательный заряд всех свободных электронов по абсолютному значению равен положительному заряду всех ионов кристаллических решёток. Поэтому в обычных условиях металлический проводник электронейтрален.

Итак, какие электрические заряды перемещаются под действием электрического поля в металлических проводниках?

В 1899 г. К. Рикке включал в основной провод питания трамвайных линий в Штутгарте последовательно три металлических цилиндра, тесно прижатых один к другому торцами: два крайних — медные, а средний — алюминиевый. Через эти цилиндры более года проходил электрический ток. В результате точного взвешивания оказалась, что диффузии в металлах не происходило: в медных цилиндрах не было атомов алюминия и наоборот.

Таким образом, К. Рикке доказал, что при прохождении по проводнику электрического тока в металлах ионы не перемещаются. Следовательно, электрический ток в металлических проводниках образуется благодаря упорядоченному движению электронов.

Теперь остаётся выяснить: как движутся свободные электроны?

При отсутствии в проводнике электрического поля движение электронов хаотическое, как движение молекул газов или жидкостей. В любой момент скорости движения разных электронов отличаются значением и направлением.

При наличии в проводнике электрического поля электроны, сохраняя своё хаотическое движение, начинают смещаться к положительному полюсу источника. Одновременно с беспорядочным движением электронов возникает их упорядоченное движение. На рисунке 43 схематично показана траектория движения одного электрона из точки 1 в точку 2 под действием электрического поля.

Электрический ток - определение и понятия с примерами
Рис. 43

Отсюда следует: электрический ток в металлах (металлических проводниках) — это упорядоченное движение электронов под действием электрического поля, которое создаётся источником электрического тока.

Исследованием электронной проводимости металлов занимался украинский учёный А. Э. Малиновский (1884-1937). Он представил свою интерпретацию взаимодействия свободных электронов и положительных ионов в металлах. Сделал уточнения к теории экспериментов, которые в 1916 г. проводили американский физик Р. Толмен и шотландский физик Б. Стюарт. Они раскручивали до большой скорости катушку из тонкого медного провода вокруг её оси. Потом катушку резко тормозили и регистрировали в цепи кратковременный электрический ток, обусловленный инерцией носителей заряда, которыми оказались именно электроны.

Действия и направление электрического тока

Движение электрически заряженных частиц в веществе проводников глаз человека не воспринимает. При этом направленное движение заряженных частиц связано с целым рядом явлений, по которым можно определить наличие электрического тока в цепи.

Электрический ток - определение и понятия с примерами
Рис. 44

Опыт 1. Присоединим к полюсам источника тока никелиновый или нихромовый провод (рис. 44). Видим, что он нагревается, накаляется до красного свечения и провисает. Наблюдаем тепловое действие тока. Под действием электрического тока в электрических лампах вольфрамовая нить накаляется до яркого свечения, нагреваются спирали электроутюгов и электроплит.

Тепловое действие тока широко используется при контактной сварке металлов (рис. 45). Через детали в процессе сварки пропускают сильный ток. В результате в местах контактов детали сильно нагреваются и свариваются.

Электрический ток - определение и понятия с примерами
Рис. 45

Опыт 2. На железный гвоздь или стержень намотаем несколько десятков витков изолированного медного провода. Освободив его концы от изоляции, присоединим их к источнику тока. Видим, что гвоздь приобретает свойство притягивать к себе мелкие железные предметы: опилки, гвозди, скрепки и т. п. (рис. 46), то есть он стал магнитом. В этом опыте проявляется магнитное действие электрического тока.

Электрический ток - определение и понятия с примерами
Рис. 46

Электрический ток - определение и понятия с примерами
Рис. 47

Опыт 3. По рисунку 47 собираем электрическую цепь. Если в банке чистая (дистиллированная) вода, то электрическая лампа не горит. Если в воду добавить кристаллы медного купороса, то лампа засветится.

Следовательно, в растворе медного купороса проходит электрический ток. Если через определённое время вынуть из банки отрицательный электрод, то увидим, что на нём выделилась чистая медь, то есть прохождение электрического тока сопровождается химическим превращением веществ.
Химическое действие тока используют для получения чистых металлов.

Световое действие тока вы можете наблюдать при свечении ламп дневного света (рис. 48). Под действием электрического поля газы, находящиеся в лампе, начинают светиться. В природе световое действие электрического тока наблюдается во время электрического разряда — молнии (рис. 49).
А какое направление имеет электрический ток?

Электрический ток - определение и понятия с примерами
Рис. 46

Электрический ток - определение и понятия с примерами
Рис. 47

Ещё раз вспомним, что электрический ток — это упорядоченное движение заряженных частиц. Движение каких именно заряженных частиц в электрическом поле принимают за направление тока?

На практике чаще всего имеют дело с электрическим током в металлических проводниках, поэтому за направление тока в цепи целесообразно принять направление движения электронов в электрическом поле, то есть от отрицательного полюса источника к положительному.

Но вопрос о направлении тока возник в науке, когда об электронах и ионах ничего не было известно. Полагали, что во всех проводниках могут перемещаться как положительные, так и отрицательные заряды.

За направление электрического тока условно принимают то направление, в котором перемещаются (или могли бы перемещаться) в проводнике положительные заряды, то есть направление от положительного полюса источника тока к отрицательному.

Убедимся, что от направления тока зависит его механическое действие.

Опыт 3. Присоединим к батарее гальванических элементов электродвигатель со стрелкой на его шкиве. Вал двигателя будет вращаться в определённом направлении (рис. 50, а). А если поменять полюса батареи, то вал двигателя вращается в противоположном направлении (рис. 50, б). Работа электродвигателя — это пример механического действия электрического тока, которое состоит в том, что рамка из провода, помещённая в магнитное папе, вращается в определённом направлении, если через неё проходит ток. Направление поворота при этом зависит от направления тока, что мы и наблюдаем в ходе опыта. Действие магнитного поля на проводник с током будем изучать позднее.
Направление тока учитывают во всех правилах и законах электрического тока.

Электрический ток - определение и понятия с примерами
Рис. 50

Пример №1

Если между параллельными металлическими пластинами, присоединенными к кондукторам работающей электрофорной машины, разместить лёгкие пёрышки, то они будут интенсивно перемещаться от одной пластины к другой. Какое физическое явление моделирует данный опыт?
Ответ: электрический ток. Пёрышки совершают упорядоченное движение частиц в электрическом поле.

Пример №2

Будет ли действовать элемент Вольта, если оба его электрода — цинковые или медные?
Ответ: нет, поскольку не будет различия в зарядах электродов.

Пример №3

Начертите схему электрической цепи, состоящей из батареи аккумуляторов, двух параллельно соединённых ламп амперметра, выключателя, соединительных проводников.
Ответ: схема электрической цепи показана на рисунке 51.
Электрический ток - определение и понятия с примерами
Рис. 51

Сила тока

Опыты показывают, чем больше электрических за{)ядов проходит через проводник за определённое время, тем больше проявляется действие электрического тока — тепловое, химическое, магнитное, механическое и световое. Чем больше заряженных частиц перемещается от одного полюса источника тока к другому, тем больший общий заряд переносится частицами.

Электрический заряд, который проходит через поперечное сечение проводника за единицу времени, определяет силу тока в цепи.

Сила тока — это физическая величина, которая характеризует электрический ток и определяется отношением электрического заряда, прошедшего через поперечное сечение проводника, ко времени его прохождения.

Силу тока обозначают латинской буквой I. Формула для определения силы тока имеет вид:
Электрический ток - определение и понятия с примерами
где q — электрический заряд, проходящий через проводник за время t.

За единицу силы тока принят один ампер (1 А). Эта единица названа в честь французского физика Андре-Мари Ампера. На Международной конференции мер и весов в 1948 г. принято решение в основу определения единицы силы тока положить явление взаимодействия двух проводников с током.
Возьмём два гибких прямых проводника, разместим их параллельно и присоединим к источнику тока. При замыкании цепи в проводниках проходит электрический ток, в результате они взаимодействуют между собой: притягиваются, если направление тока в них одинаковое (рис. 56, а), либо отталкиваются, если направление противоположное (рис. 56, б).

Электрический ток - определение и понятия с примерами
Рис. 56

Этот опыт впервые выполнил А.-М. Ампер. Он измерял силу взаимодействия проводников с током. Оказалось, что эта сила зависит от длины проводников, расстояния между ними, среды, в которой они размещены, и от силы тока в проводниках. Учёный определил, что два очень тонких и длинных параллельных проводника длиной Imb безвоздушном пространстве (вакууме), расстояние между которыми равно 1 м, а сила тока в каждом из них одинакова, взаимодействуют с силой 0,0000002 Н.

Один ампер (1 А) — это сила тока, который, протекая в двух параллельных прямолинейных бесконечной длины тонких проводниках, размещённых в вакууме на расстоянии 1 м, вызывает на каждом участке проводника длиной 1 м силу взаимодействия 2 ∙ 10-7 Н.

Применяют также дольные и кратные единицы силы тока: миллиампер (мА), микроампер (мкА), килоампер (кА):
1 мА = 1 ∙ 10-3 A; 1 мкА = 1 ∙ 10-6A; 1 кА = 1 ∙ 1O-3А.

Через единицу силы тока 1А определяют единицу электрического заряда.

ПосколькуЭлектрический ток - определение и понятия с примерами, то q = It. Поскольку I = 1 A, t = 1 с, то единица электрического заряда — один кулон (1 Кл).
1 кулон = 1 ампер ∙ 1 секунда, или 1 Кл = 1A∙1c = 1A∙с.

Из определения силы тока следует, что при силе тока IA через поперечное сечение проводника каждую секунду проходит электрический заряд 1 Кл, то есть 1Электрический ток - определение и понятия с примерами. Зная заряд электрона, можно определить, что при силе тока 1 А через поперечное сечение проводника проходят 6,25 ∙ 1018 электронов в секунду.

Диапазон (границы) значений силы тока, с которыми приходится иметь дело в физике, небольшой по сравнению с другими величинами и составляет от 10-6(0,000001) до 106 (100 000) А.

В электрических лампах, нагревательных приборах сила тока достигает нескольких ампер. При электросварке, когда нужно разогреть металлы до высокой температуры, сила тока достигает значений в несколько тысяч ампер. Ещё большая сила тока (в сотни тысяч ампер) возникает во время молнии. При этом воздух нагревается до температуры 20 000 0С; за очень короткое время (до 10-4 с) его давление повышается до 300 атм (3,03 ∙ 107 Па), что соответствует интенсивному акустическому удару, — мы слышим гром.

Химическое действие тока используется при зарядке аккумуляторов, хромировании и никелировании деталей и изделий, при электрохимическом получении металлов. Сила тока во время этих процессов составляет от нескольких ампер (зарядка аккумуляторов) до сотен и даже тысяч ампер (получение чистых металлов).

Магнитное действие тока используют в электромагнитах, двигателях и др. При работе мощных двигателей сила тока может достигать сотен ампер.

В таблице 1 приведены значения силы тока в некоторых технических устройствах и приборах.

Таблица 1 Сила тока в различных технических устройствах и приборах

Устройство, прибор Сила тока, А
Электронный микроскоп 0,00001
Кинескоп телевизора 0,00012
Рентгеновская установка 0,02-0,10
Электробритва 0,08
Электрический фонарик 0,3
Велосипедный генератор (при Электрический ток - определение и понятия с примерами) 0,3
Электрическая лампа 0,3-0,4
Пылесос 1,9-4,2
Электроплитка 3-4
Генератор автомобиля 17
Двигатель троллейбуса 160-220
Двигатель электровоза 350
Аппарат для контактной сварки 10 000

Для измерения силы тока в цепи используют прибор — амперметр (рис. 57). Шкала амперметра на рисунке 57, а проградуирована в амперах (А), а на рисунке 57, б — в микроамперах (сокращённое обозначение — мкА, международное — μA). IIa шкалах соответственно написаны буквы А и μA. На схемах амперметр изображают буквой А в кружке (рис. 58).

Любой измерительный прибор при включении в цепь не должен влиять на значение измеряемой величины. Поэтому конструкция амперметра такова, чтобы при включении в цепь сила тока в ней почти не изменялась. При этом электрическое сопротивление должно быть как можно меньшим (об электрическом сопротивлении узнаете позднее).
 

Электрический ток - определение и понятия с примерами
Рис. 57

Электрический ток - определение и понятия с примерами
Рис. 58

Для измерения силы тока в цепи амперметр включают в цепь последовательно с тем прибором, в котором измеряют силу тока. Для этого надо «разорвать» цепь, то есть отсоединить от прибора один из проводников, и в образовавшийся промежуток включить амперметр с помощью двух клемм или зажимов на его корпусе. Возле одной из клемм амперметра стоит знак «плюс» («+»), возле другой — «минус» (иногда «-» не указывают). Клемму со знаком «+» надо обязательно соединить с проводом, который отходит от положительного полюса источника тока.

Поскольку по закону сохранения электрического заряда количество зарядов, поступающих в цепь от одного из полюсов источника тока, равно количеству зарядов, которые возвращаются на второй полюс источника, то сила тока одинакова на разных участках цепи с последовательно соединёнными приборами. Поэтому для измерения силы тока в такой цепи амперметр можно включать в любом месте, его показания всегда будут одинаковы.

Опыт 2. Собираем электрическую цепь (рис. 59) и измеряем силу тока в спирали лампочки карманного фонарика. Сила тока равна 1 А.

В технике применяют разнообразные амперметры. По их шкалам либо другим обозначениям видно, на какую наибольшую силу тока они рассчитаны. Превышать эту силу тока нельзя, так как прибор может выйти из строя. На практике также используют амперметры с цифровыми индикаторами. Например, на рисунке 60 показано, что измеренная таким амперметром сила тока в цепи равна 0,0625 А.
Современными амперметрами можно измерять силу тока до 105 А.

Электрическое напряжение

Под действием электрического поля, которое создаёт источник тока, заряженные частицы перемещаются по проводнику. При этом выполняется работа: накаляется спираль электролампы, приходит в движение электрический двигатель и т. д. Это говорит о том, что главную роль в прохождении электрического тока в проводниках играет электрическое поле. Для характеристики электрического поля вводят физическую величину, которая называется электрическим напряжением, или напряжением.

Напряжение — это физическая величина, которая определяется отношением работы электрического поля на данном участке цепи к электрическому заряду, прошедшему по этому участку. Оно характеризует электрическое поле, которое образует ток.

Напряжение обозначается большой латинской буквой U. Формула для определения напряжения:
Электрический ток - определение и понятия с примерами,

где А — работа, выполненная электрическим полем при прохождении тока;
q — значение электрического заряда, перенесённого током.

Единица напряжения в СИ — один вольт (1 В). Названа так в честь Алессандро Вольта, создавшего первый гальванический элемент.

Один вольт (1 В) — это напряжение на концах проводника, при котором работа по перемещению электрического заряда в один кулон (1 Кл) поэтому проводнику равна одному джоулю (1 Дж).

Отсюда Электрический ток - определение и понятия с примерами.

Кроме вольта применяют дольные и кратные ему единицы: милливольт (мВ) и киловольт (кВ).
1 мВ = 0,001 В; 1 кВ = 1000 В.

Электрические приборы работают при разном напряжении. Например, при контактной сварке напряжение составляет 0,1 В, бытовые приборы работают при напряжении 220 В, мощные двигатели — 380 В, а двигатели электровоза — 1500 В.

Разные источники тока характеризуются рабочим напряжением. В гальваническом элементе и аккумуляторе (химических источниках тока) значение напряжения небольшое. Если в гальваническом элементе медный и железный электроды, то напряжение 0,78 В, медный и цинковый — 1,1 В, серебряный и цинковый — 1,56 В. Среднее напряжение свинцового кислотного аккумулятора составляет 2 В, а железоникелевого щелочного — 1,25 В.

Термоэлементы (термопары) и фотоэлементы (солнечная батарея) создают ещё меньшие напряжения. Например, термоэлемент из графита и карбида титана при нагревании спая до 1 000°C создаёт напряжение 52 мВ (0,052 В).

Солнечная кремниевая батарея площадью 160 см2 при освещении солнечными лучами даёт 2 В. Чтобы получить большие напряжения, гальванические элементы, аккумуляторы, термо- и фотоэлементы соединяют в батареи.

В таблице 2 приведены значения напряжения в некоторых технических устройствах и приборах.

Таблица 2 Напряжение в различных технических устройствах и приборах

Устройство, прибор Напряжение, В
Электронный микроскоп 130 000
Кинескоп телевизора 16 000
Рентгеновская установка 70 000-200 000
Электробритва 220
Электрический фонарик 4,5
Велосипедный генератор (при Электрический ток - определение и понятия с примерами) 7,2
Электрическая лампа 220
Электроплитка 220
Генератор автомобиля 12
Двигатель троллейбуса 550
Двигатель электровоза 1 500
Аппарат для контактной сварки 0,1

Для измерения напряжения в электрических цепях используют вольтметр (рис. 62, а) — для школьных опытов, б — для лабораторных работ).

Чтобы отличить вольтметры от амперметров или других электрических измерительных приборов, на их шкалах ставят букву V. На схемах вольтметр изображают так, как это показано на рисунке 62, в. Как и у амперметра, возле одного зажима вольтметра ставят знак «+». Этот зажим обязательно надо соединять с проводом, идущим от положительного полюса источника тока. Иначе стрелка прибора будет отклоняться в обратную сторону, и он может выйти из строя, то есть важно учитывать направление электрического тока.
Вольтметр включают иначе, чем амперметр.

Опыт. 2 Измеряем напряжение, которое даёт батарея гальванических элементов (рис. 63). Напряжение на полюсах батареи составляет 4,6 В. (Примечание: нельзя таким образом включать амперметр, так как он выйдет из строя!) Подсоединим теперь вольтметр к одному из зажимов выключателя и лампы. Вольтметр ничего не показывает (рис. 64, а). А если подсоединить вольтметр к обоим зажимам лампы, то он покажет, какое напряжение поступает на лампу (рис. 64, б). Это напряжение равно 4 В.

Вольтметр подсоединяем параллельно к участку цепи, на котором нужно измерить напряжение, то есть зажимы вольтметра надо подсоединить к тем точкам цепи, между которыми необходимо измерить напряжение. При этом через вольтметр проходит определённый ток из цепи, что приводит к изменению значения напряжения в точках соединения. Чтобы это изменение было как можно меньшим, электрическое сопротивление вольтметра должно быть большим (об электрическом сопротивлении узнаете позднее).

На практике также используют вольтметры с цифровыми индикаторами. Измеряем напряжение в электросети, оно равно 217 В (рис. 65).

Современными вольтметрами можно измерять напряжение до 106 В.
Электрический ток - определение и понятия с примерами
Рис. 65

Кстати:
У некоторых рыб есть органы, вырабатывающие электрический ток. Например, электрический сом даёт разряды напряжением до 360 В, электрический скат — до 220 В, электрический угорь — до 650 В и силой тока 2 А.

Электрическое сопротивление проводников

Вы уже знаете, что электрический ток в цепи — это упорядоченное движение заряженных частиц в электрическом поле. Чем сильнее действие электрического поля на заряженные частицы, которые в нём перемещаются, тем больше сила тока в цепи.

При этом действие электрического поля характеризуется напряжением. Итак, возникает вопрос: зависит ли сила тока в проводнике от напряжения на его концах?

Опыт 1. Собираем электрическую цепь, используя гальванический элемент, ключ, амперметр, никелиновую спираль от магазина сопротивлений, к которой параллельно присоединяем вольтметр (рис. 67).

Замкнём цепь и зафиксируем показания приборов (рис. 67, а). К гальваническому элементу последовательно присоединим такой же и снова замкнём цепь. Мы видим, что напряжение на спирали при этом увеличилось в два раза, а амперметр показывает вдвое большую силу тока (рис. 67, б).

Если соединить последовательно три элемента, то напряжение на спирали увеличится в три раза, во столько же раз увеличится и сила тока в электрической цепи.

Электрический ток - определение и понятия с примерами
Рис. 67

Графически это можно изобразить следующим образом (рис. 68). На горизонтальной оси в выбранном масштабе откладываем значения напряжения, а на вертикальной — соответствующие им значения силы тока. Наносим точки на плоскость и получаем график линейной зависимости: чем большее напряжение приложено к участку цепи, тем больший ток в цепи.

Следовательно, сила тока в проводнике прямо пропорциональна напряжению на концах проводника.

Электрический ток - определение и понятия с примерами
Рис. 68

Опыт 2. Собираем электрическую цепь (рис. 69). При её замыкании амперметр показывает определённое значение силы тока. Разомкнув цепь, присоединим к лампе никелиновый провод длиной 1— 2 м. Снова замкнём цепь и видим, что сила тока в цепи уменьшилась. Если вместо никелинового провода включить в цепь такого же размера нихромовый провод, то амперметр покажет ещё меньшую силу тока. При включении медного провода такого же размера сила тока в цепи возрастает.

Электрический ток - определение и понятия с примерами
Рис. 69
Если при замыкании цепи каждый раз присоединять к концам этих проводников вольтметр, то он будет показывать одинаковое напряжение. Следовательно, сила тока в цепи зависит не только от напряжения, но и от свойств проводников, включённых в цепь.

Зависимость силы тока от свойств проводника объясняется тем, что направленному движению свободных электронов в металлическом проводнике противодействуют их хаотические столкновения с ионами кристаллической решётки, находящимися в состоянии теплового движения (колебаний). Это противодействие приводит к уменьшению скорости направленного движения заряженных частиц, то есть к уменьшению силы тока в цепи.

Величина, характеризующая свойство проводника противодействовать направленному перемещению свободных зарядов внутри него, называется электрическим сопротивлением проводника.

Проводники, имеющие одинаковые геометрические размеры (длину и площадь поперечного сечения), но изготовленные из разных металлов, имеют разные значения электрического сопротивления, что объясняется различиями в строении их кристаллических решёток.

Электрическое сопротивление обозначают большой латинской буквой R. Единица электрического сопротивления в СИ — один ом (1 Ом); названа в честь немецкого физика Георга Ома.

Значение электрического сопротивления один ом (1 Ом) имеет такой проводник, в котором проходит ток силой один ампер (1 А) при напряжении один вольт (1 В) на его концах.

На практике используют следующие единицы сопротивления: миллиом (мОм), килоом (кОм), мегаом (МОм).
1 мОм =1∙10-3Ом; 1 кОм = 1 ∙ 1O3 Ом; 1 МОм = 1 • 106 Ом.

Схемы электрических и электронных приборов состоят из совокупности электрических цепей, сила тока и напряжение которых зависят от значений электрического сопротивления специальных деталей — резисторов разных конструкций. Значение сопротивления резисторов составляет от десятых долей ома до десяти тысяч мегаомов.
 

Закон Омa для однородного участка электрической цепи

Явления, происходящие в электрических цепях, характеризуются взаимосвязанными физическими величинами: силой тока, напряжением и сопротивлением. Вы, например, знаете, что сила тока в проводнике прямо пропорциональна напряжению на его концах.

Впервые явления в электрических цепях подробно изучил Георг Ом. В 1826 г. ему удалось экспериментально установить зависимость между силой тока, напряжением и сопротивлением в электрических цепях. Эта зависимость оказалась очень важной и получила название закона Ома. Чтобы понять его физический смысл, выполним опыты.

Опыт 1. Используя источник тока, амперметр, спираль из никелинового провода (резистор), вольтметр, ключ и соединительные проводники, собираем электрическую цепь (рис. 70, а). На рисунке 70, б приведена схема этой цепи. Амперметр, включённый в цепь последовательно, будет показывать силу тока, протекающего в спирали. Вольтметр, присоединённый к спирали параллельно, покажет напряжение на её концах. Сопротивление спирали не изменяется.

Электрический ток - определение и понятия с примерами
Рис. 70

Замкнём ключ и снимем показания вольтметра и амперметра: U = 4 В; J = 1A. Если увеличить напряжение в два раза, то есть U = 8 В, то амперметр покажет силу тока в два раза большую, то есть I = 2 А. Отсюда следует: при постоянном сопротивлении проводника сила тока, протекающего в нём, прямо пропорциональна напряжению на его концах (рис. 68 на с. 56).

ОПЫТ 2. Собираем такую же электрическую цепь, как в предыдущем опыте, но вместо одного нихромового включаем поочерёдно провода, сопротивление которых равно соответственно 1 Ом, 2 Ом, 4 Ом (рис. 71, а).

Напряжение на концах каждого проводника в ходе опыта было постоянным, контролируем его значение по показаниям вольтметра. Силу тока в цепи измеряем амперметром.

Результаты опытов следующие: напряжение на концах проводников составляет 2 В; при включении проводника с сопротивлением 1 Ом сила тока в цепи равна 2 A, 2 Ом — 1 А, 4 Ом — 0,5 А. Отложив эти значения на осях координат, построим график (рис. 71, б). Видим, что это гипербола, то есть чем больше сопротивление проводника, тем меньший ток в нём протекает.

Электрический ток - определение и понятия с примерами
Рис. 71

Следовательно, при постоянном напряжении сила тока в проводнике обратно пропорциональна его сопротивлению.

По результатам обоих опытов получаем формулу зависимости силы тока в проводнике от напряжения на концах проводника U и его сопротивления R:
Электрический ток - определение и понятия с примерами

Эта формула выражает закон Ома.

Сила тока на участке электрической цепи прямо пропорциональна напряжению на данном участке и обратно пропорциональна сопротивлению этого участка.

На законе Ома основан экспериментальный способ определения сопротивления проводника. Из закона Ома следует, что

Электрический ток - определение и понятия с примерами

Итак, для определения сопротивления проводника необходимо измерить напряжение на нём и силу тока, затем значение напряжения разделить на значение силы тока. Из формулы также следует, что единица электрического сопротивления равна отношению единицы напряжения к единице силы тока, то есть  Электрический ток - определение и понятия с примерами .

Если известны сопротивление и сила тока на участке цепи, то по закону Ома можно вычислить напряжение на его концах:
Электрический ток - определение и понятия с примерами

Чтобы определить напряжение на концах участка цепи, силу тока на этом участке необходимо умножить на его сопротивление.

Кстати:

Г. Ом после выхода книги «Теоретические исследования электрических цепей», в которой он изложил открытый им закон, писал, что «рекомендует её добрым людям с тёплым чувством отца, не ослеплённого обезьяньей любовью к детям, но удовлетворённого указанием на открытый взгляд, с которым его ребёнок смотрит на недобрый мир». Мир действительно оказался для него недобрым. Спустя год после выхода его работы в одном из журналов была опубликована статья, в которой исследования Г. Ома подвергались сокрушительной критике. «Тот, кто набожными глазами смотрит на Вселенную, — говорилось в статье, — должен отвернуться от этой книги, поскольку она является плодом безусловных заблуждений, преследующих единственную цель — преуменьшить величие природы».

Злобные и необоснованные нападки на Г. Ома не прошли бесследно. Теорию Ома не приняли. И вместо научных исследований он был вынужден время и энергию тратить на полемику с оппонентами. В одном из своих писем учёный писал: «Появление на свет “Электрических цепей” принесло мне огромные страдания, и я готов проклинать время их зарождения».

Но это были временные трудности. Постепенно теория Ома получила повсеместное признание. Закон Ома внёс такую ясность в правила расчёта токов и напряжений в электрических цепях, что американский учёный Дж. Генри, узнав об открытии Ома, не удержался от восклицания: «Когда я впервые прочитал теорию Ома, то она мне показалась молнией, внезапно озарившей комнату, погружённую во тьму».

Пример №4

Сила тока в цепи электрической лампы равна 0,3 А. Сколько электронов проходит через поперечное сечение спирали за 5 мин?

Дано:
I = 0,3 А
t = 5 мин = 300 с
e = -l,6∙ 10-19Кл π = 3,14

Решение
Заряд, прошедший через поперечное сечение проводника за время протекания тока, вычисляем по формуле:
q = It.
Разделив это значение на элементарный заряд, определим количество электронов, прошедших через поперечное сечение проводника:
Электрический ток - определение и понятия с примерами
Подставив значения известных величин, получим:

Электрический ток - определение и понятия с примерамиэлектронов.

N — ?

Ответ: через поперечное сечение спирали проходит 5,6 ∙ 1020 электронов. 

Пример №5

Какое сопротивление имеет вольтметр, рассчитанный на 150 В, если сила тока в нём не должна превышать 0,01 А?

Дано:
U = 150 В
I = 0,01 А

Решение

Сопротивление вольтметра определим по закону Ома: Электрический ток - определение и понятия с примерами
Подставив значения, получим: R = 150 В: 0,01 A = 15 000 Ом = 15 кОм. 

R — ?

Ответ: сопротивление вольтметра равно R = 15 кОм.

Пример №6

Что изменилось на участке цепи, если включённый с ней последовательно амперметр показывает увеличение силы тока?
Ответ: увеличилось напряжение или уменьшилось сопротивление.

Расчет сопротивления проводника

Причиной электрического сопротивления проводника является взаимодействие свободных электронов с ионами кристаллической решётки металла. Рассмотрим, от чего зависит сопротивление проводника.

Опыт 1. В электрическую цепь (рис. 78) поочерёдно включаем проводники из одного материала, одинакового диаметра, но разной длины. Силу тока измеряем амперметром, а напряжение — вольтметром.

По результатам опытов делаем вывод: чем длиннее проводник, тем больше его электрическое сопротивление. Поскольку чем длиннее проводник, тем большее противодействие испытают на своём пути частицы, перемещающиеся направленно.

Опыт 2. Включаем в электрическую цепь поочередно проводники, изготовленные из одного материала и одинаковой длины, но с разным поперечным сечением. Измерив силу тока в проводниках и напряжение на их концах, убеждаемся: чем толще проводник, тем меньше его электрическое сопротивление. Увеличение толщины проводника равнозначно «расширению русла», по которому перемещаются заряды, поэтому сопротивление проводника уменьшается.

Опыт 3. Включаем в электрическую цепь поочерёдно проводники, изготовленные из разных веществ, но одинаковой длины и поперечного сечения. Оказывается, что электрическое сопротивление проводника зависит от того, из какого вещества он изготовлен. Это объясняется тем, что разные металлы имеют разные кристаллические структуры, следовательно, тормозящее действие столкновений ионов и свободных электронов оказывается разным.

Зависимость сопротивления проводника от его размеров и вещества, из которого он изготовлен, впервые установил опытным путём Г. Ом: сопротивление проводника прямо пропорционально его длине, обратно пропорционально площади поперечного сечения и зависит от вещества, из которого он изготовлен.
Электрический ток - определение и понятия с примерами

Рис. 78

Зависимость сопротивления проводника от вещества, из которого он изготовлен, характеризуют удельным сопротивлением вещества.

Удельное сопротивление вещества — это физическая величина, численно равная сопротивлению изготовленного изданного вещества проводника длиной 1 м и площадью поперечного сечения 1 м2.

Если длину проводника обозначить буквой I, площадь его поперечного сечения — S, удельное сопротивление — р, то сопротивление проводника определим по формуле:
Электрический ток - определение и понятия с примерами

Из этой формулы можно определить удельное сопротивление вещества:

Электрический ток - определение и понятия с примерами

Поскольку единица сопротивления — 1 Ом, единица площади — 1 м2, единица длины — 1 м, то единица удельного сопротивления равна:

Электрический ток - определение и понятия с примерами.

На практике площадь поперечного сечения проводников обычно выражают в квадратных миллиметрах, поэтому единицей удельного сопротивления вещества в этом случае является Электрический ток - определение и понятия с примерами

В таблице 3 приведены экспериментально полученные значения удельного сопротивления веществ, широко применяемых на практике.

Таблица 3 Удельные сопротивления некоторых веществ (при t = 20 °C)

Вещество Электрический ток - определение и понятия с примерами Вещество Электрический ток - определение и понятия с примерами Вещество Электрический ток - определение и понятия с примерами
Серебро 0,016 Свинец 0,21 Нихром (сплав) 1,1
Медь 0,017 Никелин(сплав) 0,40 Фехраль(сплав) 1.3
Золото 0,024 Манганин(сплав) 0,43 Графит 13
Алюминий 0,028 Константан (сплав) 0,50 Фарфор 1019
Вольфрам 0,050 Ртуть 0,96 Эбонит

1020

Железо 0,100

Лучшими проводниками электричества являются серебро, медь, золото. Но для практических целей (например, создание электросетей) проводники изготовляют из алюминия, меди и железа. В нагревательных элементах используют нихромовые и фехралевые проводники. Фарфор и эбонит являются очень хорошими изоляторами.

Итак, для разных веществ значения удельного сопротивления изменяются в очень широких пределах, что объясняется их разным внутренним строением.

Зависимость сопротивления проводника от температуры

На практике часто приходится изменять силу тока в цепи, то увеличивая её, то уменьшая. Например, изменяя силу тока в электроплитке, мы регулируем температуру её нагревания.

Для регулирования силы тока в электрической цепи применяют специальные приборы — реостаты. На рисунке 79 показан внешний вид реостатов (их условное обозначение см. в таблице на с. I форзаца). Такие реостаты называют ползунковыми. В них на керамический цилиндр намотан провод, покрытый тонким слоем окалины, поэтому витки провода изолированы один от другого. Над обмоткой размещён металлический стержень, вдоль которого перемещается ползунок. От трения ползунка об витки слой окалины под контактами ползунка стирается, и электрический ток в цепи проходит от витков провода к ползунку, а через него — в стержень с зажимом на конце. Реостат включают в цепь с помощью этого зажима и зажима, соединённого с одним из концов обмотки и размещённого на корпусе реостата.

Электрический ток - определение и понятия с примерами
Рис. 79
Перемещая ползунок, можно увеличивать либо уменьшать сопротивление включённого в цепь реостата.
На рисунке 80 изображён реостат (а) и магазины сопротивлений (б), позволяющие изменять сопротивление в цепи не плавно, а скачкообразно.

Электрический ток - определение и понятия с примерами
Рис. 80 

Каждый реостат рассчитан на определённое сопротивление и допустимую силу тока, превышать которую не следует, поскольку обмотка реостата может накалиться и перегореть. Сопротивление реостата и наибольшее допустимое значение силы тока указаны на корпусе реостата. А зависит ли сопротивление проводника от его состояния, в частности температуры?

ОПЫТ 1. Собираем электрическую цепь из источника тока, стальной спирали, амперметра и ключа (рис. 81). Нагреваем спираль в пламени горелки. Амперметр, включённый в цепь, покажет уменьшение силы тока с повышением температуры.

Электрический ток - определение и понятия с примерами
Рис. 81

Следовательно, с изменением температуры сопротивление металлических проводников изменяется: при повышении температуры — увеличивается, при её понижении — уменьшается.
Согласно научным исследованиям в ограниченном диапазоне температур сопротивление металлических проводников возрастает прямо пропорционально температуре (рис. 82), что определяют по формуле:
Электрический ток - определение и понятия с примерами

Где R — сопротивление проводника при определённой температуре; R0 — сопротивление проводника при 0 oC; t — температура проводника по шкале Цельсия; а — температурный коэффициент сопротивления.

Электрический ток - определение и понятия с примерами
Рис. 82

Температурный коэффициент сопротивления , характеризует зависимость сопротивления вещества от температуры и определяется относительным изменением сопротивления проводника при нагревании на 1°C:
Электрический ток - определение и понятия с примерами

У чистых металлов (с минимальными примесями):

Электрический ток - определение и понятия с примерами

Например, сопротивление вольфрамовой нити лампы накаливания возрастает при прохождении по ней тока в 10 раз и более.

Сплав константан (медь с никелем) имеет очень малый температурный коэффициент сопротивления, приблизительно 10 5 oC. Удельное сопротивление константана большое — р = 10-6 Ом • м. Такие сплавы используют для изготовления эталонных сопротивлений и дополнительных сопротивлений в измерительных приборах, если необходимо, чтобы сопротивление значительно не изменялось при колебаниях температуры.

Зависимость сопротивления металлических проводников от температуры используют в термометрах сопротивления, рабочим элементом которых является платиновый провод. Изменения температуры окружающей среды определяют по изменению сопротивления провода. Такими термометрами можно измерять очень низкие и очень высокие температуры среды, когда жидкостные термометры непригодны.

Для измерения сопротивления проводников используют приборы омметры разных конструкций. На рисунке 83 для измерения сопротивления резистора к нему присоединили омметр. Цифровой индикатор омметра показывает, что сопротивление проводника равно 39,1 Ом.

Пример №7

Сопротивление одной катушки медного провода равно 1,5 Ом, а другой такой же катушки — 6 Ом. Во сколько раз длина провода в первой катушке меньше, чем во второй?
Ответ: Сопротивление проводника прямо пропорционально его длине. Чем короче медный провод, тем меньше его сопротивление. Отсюда, длина медного провода в первой катушке в 4 раза меньше, чем во второй.

Пример №8

Реостат изготовлен из никелинового провода длиной 40 м и площадью поперечного сечения 0,5 мм2. Напряжение на зажимах реостата 80 В. Чему равна сила тока, который проходит в реостате?

Дано:
I = 40 м
S = 0,05 мм2
U = 80 В
р = 0,40 Электрический ток - определение и понятия с примерами

Решение
Силу тока определим по закону Ома:

Электрический ток - определение и понятия с примерами.
Неизвестное сопротивление никелинового
провода определим по формуле Электрический ток - определение и понятия с примерами.

I — ?

Удельное сопротивление р находим по таблице 3 (с. 66). Тогда силу тока можно вычислить по формуле:Электрический ток - определение и понятия с примерами. Подставив значения, получим:

Электрический ток - определение и понятия с примерами

Ответ: сила тока в реостате равна 2,5 А.

Последовательное соединение проводников

Электрические цепи на практике состоят из нескольких потребителей, которые могут быть соединены последовательно, параллельно или последовательно и параллельно (смешанное соединение).
При последовательном соединении потребителей * (проводников) их соединяют поочерёдно один за другим без разветвлений проводов между ними.

Oпыт  К источнику тока присоединим последовательно две лампы (рис. 84, а), начертим схему этой электрической цепи (рис. 84, б). Если выключить одну лампу, то цепь разомкнётся, и вторая лампа погаснет (рис. 84, в). После выполнения лабораторных работ вам уже известно следующее.

1.    При последовательном соединении проводников сила тока в любой части электрической цепи одинакова, то есть

I=I1=I2

Электрический ток - определение и понятия с примерами
Рис. 84

2.    Полное напряжение U в цепи при последовательном соединении, или напряжение на полюсах источника тока, равно сумме напряжений на всех отдельных участках цепи, то есть (в случае двух участков)
U = U1 + U2.

Тогда, по закону Ома, общее сопротивление цепи R при последовательном соединении равно сумме всех сопротивлений отдельных проводников либо отдельных участков цепи, то есть (в случае двух проводников)
R = R1 + R2.

Параллельное соединение проводников

Последовательно соединённые приборы отключаются все одновременно при размыкании цепи, что не всегда удобно. Например, при освещения дома или комнаты нет необходимости одновременно включать все лампы. При последовательном соединении, выключая одну из ламп, мы выключаем и остальные. Если же необходимо, чтобы приборы работали в цепи независимо, используют параллельное соединение.

При параллельном соединении потребителей (проводников) выводы каждого из них присоединяют к общей для всех паре зажимов (точек или узлов цепи).

На рисунке 85, а показано параллельное соединение двух электрических ламп, а на рисунке 85, б — схему этого соединения (в точках А и В — узлы цепи). Если одну лампу выключить, то другая продолжает светить.

Выполнив опыты, убеждаемся, что напряжение на участке цепи AB и на концах всех параллельно соединённых проводников одинаково, то есть

U = U1 = U2.

В быту и технике удобно применять параллельное соединение потребителей, поскольку они рассчитаны на одинаковое напряжение.

Электрический ток - определение и понятия с примерами
Рис. 85

При параллельном соединении ток I в точке А (рис. 85, б) разветвляется на два тока — I1 и I2 которые сходятся снова в точке В. Так поток воды в реке разделяется на два рукава, потом снова соединяясь (рис. 85, в). Отсюда очевидна связь между значениями силы тока в ветвях параллельной цепи: сила тока в неразветвлённом участке цепи равна сумме токов в отдельных параллельно соединённых проводниках, то есть

Z = Z1+Z2.

При параллельном соединении как бы увеличивается толщина проводника, поэтому общее сопротивление цепи R становится меньше наименьшего из сопротивлений проводников, включённых в цепь. Из закона Ома можно вывести соотношение для определения общего сопротивления цепи при параллельном соединении:
Электрический ток - определение и понятия с примерами

Если цепь состоит из двух параллельно соединённых одинаковых ламп с сопротивлением Электрический ток - определение и понятия с примерами каждая, то общее сопротивление цепи будет в два раза меньше сопротивления одной лампы: Электрический ток - определение и понятия с примерами

В электрических цепях часто встречается смешанное (или сложное) соединение. Это комбинация последовательного и параллельного соединений. В случае трёх резисторов возможны два варианта смешанного соединения. В первом случае (рис. 86, а) есть два последовательно соединённых участка, один из них является параллельным соединением. Общее сопротивление цепи в данном случае равно:
Электрический ток - определение и понятия с примерами
Во втором случае (рис. 86, б) вся цепь рассматривается как параллельное соединение, в котором одна ветка сама является последовательным соединением. Общее сопротивление цепи в данном случае равно:
Электрический ток - определение и понятия с примерами
При большем количестве резисторов собирают разные, более сложные схемы смешанного соединения.

Электрический ток - определение и понятия с примерами
Рис. 86

Пример №9

Два проводника с сопротивлениями R1 = 2 Ом и R2 = 3 Ом соединены последовательно. Сила тока в цепи равна 1 А. Определите сопротивление цепи и общее напряжение на проводниках.

Дано:
R1 = 2 Ом
R2 = 3 Ом
I = 1 А
 

Решение
Сила тока во всех последовательно
соединённых проводниках одинакова;
I1 = I2 = I = 1 А.

Общее сопротивление цепи составляет:
R = R1 + R2; R = 2 Ом + 3 Ом = 5 Ом.

R-? U-?

По закону Ома: U = IR, U = 1 А ∙ 5 Ом = 5 В.
Ответ: общее сопротивление цепи R = 5 Ом, напряжение U = 5 В.

Пример №10

В осветительную сеть комнаты включены две одинаковые электрические лампы. Сопротивление каждой лампы равно 440 Ом, напряжение в сети — 220 В. Определите общее сопротивление цепи и силу тока в проводах подводки.

Дано:
R1= Электрический ток - определение и понятия с примерами= 440 Ом
R2 = Электрический ток - определение и понятия с примерами= 440 Ом
U= 220 В
 

Решение
Если сопротивления ламп одинаковы,
то при параллельном соединении
проводников оно равно Электрический ток - определение и понятия с примерами

R = 440 Ом : 2 = 220 Ом.
По закону Ома: Электрический ток - определение и понятия с примерами
I = 220 В : 220 Ом = 1 А.

R-? I -?

Ответ: общее сопротивление цепи R = 220 Ом, сила тока I — 1 А.

Работа электрического тока

Все известные вам электрические приборы действуют за счёт электрической энергии, которую поставляет источник электрического тока. В результате получаем свет, тепло, звук, механическое движение и т. д., то есть разнообразные виды энергии.

Работа электрического тока —это физическая величина, характеризующая преобразование электрической энергии в другие виды энергии.

Вы уже знаете, что напряжение — это физическая величина, характеризующая электрическое поле, которое перемещает свободные заряды, создавая ток. Напряжение на концах определённого участка цепи определяется отношением работы А электрического тока на этом участке к электрическому заряду д, прошедшему по ней, то есть:

Электрический ток - определение и понятия с примерами

Из приведённого соотношения получаем формулу для определения работы электрического тока на участке цепи:

Электрический ток - определение и понятия с примерами

Чтобы определить работу электрического тока на участке цепи, необходимо напряжение на концах этого участка умножить на электрический заряд, прошедший по ней.

За время t ток силой I переносит в цепи электрический заряд q = It. Тогда формула для работы А электрического тока имеет такой вид:

Электрический ток - определение и понятия с примерами

где U — напряжение на концах участка; I — сила тока в цепи; t — время выполнения работы.

Чтобы определить работу электрического тока на участке цепи, напряжение на концах этого участка умножаем на силу тока в ней и время, в течение которого выполнялась работа.

Единицей работы электрического тока, как и механической работы, является 1 Дж. Из формулы для работы электрического тока следует: 1 Дж = 1B-1A-1c = 1B-A-с.

Чтобы измерить работу электрического тока в цепи, необходимо иметь вольтметр, амперметр и часы.

Опыт. Собираем электрическую цепь (рис. 87). C помощью вольтметра определяем напряжение, прилагаемое к электрической лампе, а с помощью амперметра — силу тока в спирали лампы. Итак, вольтметр показывает напряжение 2,3 В, амперметр — силу тока 1,2 A.

Для определения работы тока в течение 10 мин, или 600 с, используем формулу A = UIt. Подставим значения:

А = 2,3 В . 1,2 А.600 с =1656 B.A.c= 1656 Дж = 1,656 кДж.
Электрический ток - определение и понятия с примерами

Рис. 87

Следовательно, работа силы тока равна 132 кДж. На практике работу электрического тока также измеряют специальным прибором — электрическим счётчиком, внешний вид которого вы видите на рисунке 88. Счётчик электрической энергии есть в каждом доме или квартире. Его конструкция объединяет свойства всех ранее перечисленных приборов.

Мощность электрического тока

На баллоне или цоколе электрической лампы, корпусе многих бытовых электроприборов, в инструкциях к ним обязательно имеются надписи: «220 В; 60 Вт», «мощность электрического утюга 2,2 кВт» и т. д. Вы уже знаете из механики, что в ваттах измеряют мощность, следовательно, здесь речь идёт о мощности электрического тока.

Мощность электрического ток а — физическая величина, , характеризующая способность электрического тока выполнять определённую работу за единицу времени.

Мощность электрического тока обозначают большой латинской буквой Р. Если работа электрического тока А выполнена за время t, то мощность электрического тока P определяется по формуле:
Электрический ток - определение и понятия с примерами

Используя известное вам соотношение А = UIt, формулу мощности электрического тока представим в следующем виде:

Электрический ток - определение и понятия с примерами

Мощность электрического тока определяется произведением напряжения на концах участка цепи и силы тока на этом участке.

Единицей мощности электрического тока является один ватт (1 Вт). Из формулы мощности следует, что 1 Вт =1B-1A=1B -А.

Используют также кратные единицы мощности: гектоватт (гВт), киловатт (кВт), мегаватт (МВт), гигаватт (ГВт).
1 гВт = 100 Вт:
1 кВт = 1 000 Вт;
1 МВт = 1 000 000 Вт;
1 ГВт = 1 000 000 000 Вт.

Для измерения мощности электрического тока в цепи применяют вольтметр и амперметр. Есть специальный прибор — ваттметр, которым мощность электрического тока можно измерять непосредственно в цепи.

На рисунке 89 вы видите шкалу такого прибора.

Электрический ток - определение и понятия с примерами
Рис. 89

В таблице 4 приведены значения мощности некоторых потребителей электрического тока.

Таблица 4 Мощность некоторых электрических приборов

Устройство, прибор Мощность, Вт
Лампочка карманного фонарика   1,0
Лампы осветительные (бытовые) (1,1 — 1,6).102
Холодильник бытовой (0,15-2,0).102
Электрический утюг   (0,3-1,0).103
Стиральная машина (0,35 — 2,0).103
Электрическая плитка (6-8).102
Электропылесос (1.0 — 1,2).103
Двигатель трамвая (45-50).103
Двигатель электровоза 650.103
Электродвигатель прокатного стана (6-9).106

Большинство бытовых приборов рассчитано на напряжение 220 В, но разную силу тока. Следовательно, мощность потребителей электроэнергии разная, поэтому одну и ту же работу они выполняют за разное время.

Из определения мощности электрического тока получаем формулу для расчёта электрической энергии, или работы А:
 Электрический ток - определение и понятия с примерами

где P — мощность электрического тока; t — время прохождения тока.

Работа электрического тока определяется произведением мощности электрического тока и времени его потребления.

Из этой формулы получаем ещё одно выражение для единицы работы электрического тока: 1 Дж = 1 Вт ∙ 1 с = 1 Вт • с.

Если электрическая лампа мощностью 100 Вт будет светить в течение 10 ч, то работа электрического тока будет равна:
P = 100 Вт ∙ 36 000 с — 3 600 000 Дж = 3 600 кДж = 3,6 МДж.

Такое значение работы электрического тока называют киловатт-часом; обозначают 1 кВт. ч.
1 кВт -ч = 3 600 000 Дж = 3 600 кДж = 3,6 МДж.

Показания электрического счётчика, которым измеряет количество электрической энергии (работу электрического тока), потребляемой приборами в квартире, выражаются именно в киловатт-часах. Механическую работу 3 600 кДж человек может выполнить, если, например, мешок массой 50 кг поднимет по ступенькам на высоту более 7 км. На тепловой электростанции, чтобы выработать 1 кВт • ч, нужно сжечь всего 330 г угля.

В таблице на с. III форзаца приведены виды работ, на выполнение каждой из них затрачивается 1 кВт • ч энергии.

Закон джоуля-ленца

Электрический ток нагревает проводник. Это явление вам хорошо известно. Объясняется оно тем, что заряженные частицы, перемещаясь под влиянием электрического поля, взаимодействуют с атомами вещества проводника и передают им свою энергию. В результате работы электрического тока внутренняя энергия проводника увеличивается.

Английский физик Д. Джоуль и российский физик 3. Ленц показали на опыте, что в неподвижных металлических проводниках вся работа электрического тока затрачивается на увеличение их внутренней энергии. Нагретый проводник отдаёт полученную энергию окружающим его телам вследствие теплообмена. Учёные установили, что количество теплоты, которое выделяется проводником с током, определяется произведением квадрата силы тока, сопротивления проводника и времени прохождения тока.
Этот закон получил название закона Джоуля—Ленца:

Электрический ток - определение и понятия с примерами

где Q — количество теплоты, выделяемой проводником с током; I — сила тока в проводнике; R — сопротивление проводника; t — время прохождения тока.

При параллельном соединении проводников напряжения на них однаковы и количество теплоты удобно определять по формуле:

Электрический ток - определение и понятия с примерами
Согласно закону сохранения энергии количество теплоты, которая передаётся окружающей среде, равно работе электрического тока:

Электрический ток - определение и понятия с примерами

Потребители электрического тока

До середины XIX в. для освещения использовались факелы, свечи, керосиновые лампы и газовые горелки. И только в 1878 г. некоторые улицы и площади Парижа начали освещать электрическими свечами — лампами с электрической дугой (рис. 90, а). Электрическую свечу создал российский изобретатель Павел Яблочков, поэтому её называют «свечой Яблочкова».

В 1870 г. другой российский электротехник Александр Лодыгин сконструировал электрическую лампу накаливания. Лампа Лодыгина состояла из стеклянного баллона, в котором размещался тонкий угольный стержень, закреплённый между двумя медными проводниками (рис. 90, б). Угольный стержень при работе лампы раскалялся и становился источником света, но быстро перегорал (за 30—40 мин). Когда А. Лодыгин откачал из баллона воздух, то время работы пампы увеличилось.

Электрический ток - определение и понятия с примерами
Рис. 90

В 1879 г. американский изобретатель Томас Эдисон изобрёл способ получения тонких угольных нитей, используя их в конструкции электрической лампы (рис. 91). Он также предложил удобный способ включения лампы в электросеть с помощью винтового цоколя и патрона. Тем самым Т. Эдисон ускорил распространение электрического освещения.

В начале XX в. создают более экономичные лампы с металлической зигзагообразной нитью (рис. 92).
Один из недостатков этих электроламп — испарение материала нити при её накаливании, поэтому время работы ламп сокращалось. Кроме того, материал, испаряясь, осаждался на стенках стеклянного баллона и затемнял его.

В 1906 г. А. Лодыгин конструирует лампу с нитью из вольфрама. Вольфрам — тугоплавкий металл, он плавится при температуре 3 3800C. Чтобы уменьшить испарение вольфрама, баллон лампы начали наполнять инертными газами — аргоном (с примесью азота), криптоном.

Для уменьшения тепловых потерь вольфрамовую нить в лампе делают в виде спирали (рис. 93). На рисунке 94 показаны современные лампы накаливания.

Электрический ток - определение и понятия с примерами
Рис. 93

Электрический ток - определение и понятия с примерами
Рис. 94

Для освещения в быту и часто на производстве применяют электрические лампы накаливания, рассчитанные на напряжение 220 В, мощностью от 15 до 150—200 Вт; для освещения железнодорожных вагонов используют лампы, рассчитанные на напряжение 50 В, автомобилей — 12 и 6 В, в карманных фонариках — 6,3; 3,5; 2,5 и 1 В. Для специальных потребностей изготовляют лампы накаливания большой мощности. На рисунке 95 вы видите лампу, мощность которой 500 Вт. Лампы такой большой мощности охлаждают специальными вентиляторами.

Время работы электрической лампы накаливания составляет 1 000 ч. В значительной степени это зависит от напряжения, которое подаётся на лампу. Например, если на лампу, рассчитанную на 220 В, подавать напряжение 222 В, то время её работы сократится на 130 ч.

Кроме ламп накаливания человек использует газоразрядные лампы дневного света (их также называют люминесцентными). Эти лампы представляют собой длинную (от 10 до 120 см) стеклянную запаянную трубку (рис. 96). Воздух из трубки выкачивают, вводят каплю ртути и немного газа — аргона, криптона, неона и т. д. Внутри поверхность прозрачного стекла покрывают веществом, светящимся под действием ультрафиолетового излучения, которое сопровождает электрический разряд в газовой смеси. Подбирая состав этого вещества, можно получить свет любого цвета. При свечении ламп дневного света температура в них не превышает 50 °C.

Лампы дневного света экономичнее ламп накаливания в 5—7 раз, а продолжительность их работы в 2—3 раза больше. Тепловое действие тока используют в различных электрона гревательных приборах и установках. В быту широко применяют электрические плиты, утюги, чайники, кипятильники, водонагреватели и электрорадиаторы (рис. 97), в промышленности выплавляют специальные сорта стали и другие металлы, сваривают их (рис. 98), а в сельском хозяйстве обогревают теплицы, инкубаторы, сушат зерно и т. д.

Электрический ток - определение и понятия с примерами
Рис. 97

Электрический ток - определение и понятия с примерами
Рис. 98

Основной частью нагревательного электрического прибора является нагревательный элемент. Это проводник с большим сопротивлением, способный выдерживать, не разрушаясь, нагревание до высоких температур (1000— 1200 0C).

Для изготовления нагревательных элементов применяют преимущественно сплав никеля, железа, хрома и марганца — нихром. Благодаря большому сопротивлению нихрома из него делают очень удобные и компактные нагревательные элементы. В нагревательном элементе проводник в виде провода, ленты или спирали наматывают на каркас либо прикрепляют к арматуре из жаропрочного материала: слюды, керамики. В электрическом утюге (рис. 99) нагревательный элемент (нихромовая лента или спираль) нагревает нижнюю часть (подошву) утюга.

Электрический ток - определение и понятия с примерами
Рис. 99

Пример №11

Амперметр показывает силу тока в цепи 15 А, вольтметр — напряжение на участке 24 В. Какую работу выполняет электрический ток за 20 мин?

Дано:
I = 15 А
U = 24 В
t = 20 мин = 1200 с

Решение
Чтобы вычислить работу электрического
тока, используем формулу:
A = UIt. Подставив значения, получим:
A = 24 В ∙ 15 А • 1200 с = 432 000 Дж = 432 кДж.

A-? 

Ответ: электрический ток выполнил работу А = 432 кДж.

Пример №12

Какую мощность должен иметь электрический двигатель, чтобы за 20 мин выполнить работу 100 кДж?

Дано:
А = 120 кДж = 120 000 Дж
t — 20 мин = 1200 с

Решение
Для определения мощности электродвигателя,
работу электрического тока разделим на время его работы:
Электрический ток - определение и понятия с примерами
Подставив значения, получим: P = 120 000 Дж : 1200 с = 100 Вт.

P-? 

Ответ: мощность электрического двигателя P = 100 Вт.

Пример №13

Две электрические лампы мощностью 60 Вт и 100 Вт включены в сеть 220 В параллельно. Какая из них светит ярче?
Ответ: лампа мощностью 100 Вт.

Электрический ток в растворах и расплавах электролитов

Мы уже знаем закономерности прохождения электрического тока в металлах и как велико практическое значение этого явления. А существуют ли проводники электричества из неметаллов?

Далее изучая явления электрического тока в разных средах, убедимся, что кроме металлов он может существовать в жидкостях, газах и даже вакууме. В этих случаях будем рассматривать замкнутую цепь, в которой определённый участок проводника состоит из вещества в жидком или газообразном состоянии, либо является вакуумным промежутком. Проводники, подводящие напряжение (ток) к этому участку, называют электродами. Электрод, соединённый с положительным полюсом источника тока, называют анодом, а с отрицательным — катодом. При прохождении тока к аноду притягиваются свободные электроны и отрицательные ноны (анионы), а к катоду — положительные ионы (катионы).

Для существования электрического тока в веществе, помещённом в электрическое поле, необходимым является наличие свободных электрических зарядов, которые в веществе могут перемещаться под действием электрического поля на расстояния, ограниченные только размерами образца. В металлических проводниках носителями тока являются свободные электроны, а ионы металла жёстко связаны в узлах кристаллической решётки и могут осуществлять лишь колебательные движения.

В подобном связанном состоянии находятся ионы в ионных кристаллах, например в поваренной соли (NaCl). Электроны, которые отдают атомы металла (Na+), образуют отрицательные ионы галогена (Cl ), в результате возникает химическая связь. Поскольку свободных носителей электричества в ионных кристаллах нет, то при невысоких температурах они являются хорошими изоляторами. Если попытаться образовать электрическую цепь, погрузив в колбу с кристаллами сухой поваренной соли два проводника, присоединённых последовательно с микроамперметром к источнику тока, то никакого тока не зарегистрируем. При расплавливании кристаллов ионы приобретают подвижность, и поваренная соль становится проводником тока. Расплавы солей и других соединений проводят ток. Проводниками являются также водные и другие растворы солей кислот и щелочей.

Дистиллированная вода — прекрасный изолятор, поскольку не содержит свободных электрических зарядов.

Вещества, водные растворы или расплавы которых проводят ‘ электрический ток, называют электролитами.

Опыт. Соберём электрическую цепь (рис. 47) нальём в банку дистиллированную воду. Лампа не светится; амперметр показывает отсутствие электрического тока в цепи.
 

Но если растворить в воде какую-либо соль, кристаллы которой имеют ионную структуру, например поваренную соль (NaCl) или медный купорос (CuSO4), то в цепи возникнет ток, и лампа будет светится.
Молекула воды полярна, её можно представить как объект удлинённой формы, на концах которого сосредоточены электрические заряды противоположных знаков. Поэтому электрическое поле молекул воды способствует распаду ионной кристаллической решётки на свободные ионы (рис. 100, а). Кристаллическая решётка разрушается также при плавлении солей, в результате образуется жидкость, состоящая из свободных ионов.

Расщепление электролита на ионы в водном растворе или расплаве называют электролитической диссоциацией.

Типичные электролиты — это соли, кислоты и щёлочи, многие органические соединения.

Что произойдёт, если в растворе электролита создать электрическое поле (рис. 100, б)? Положительные ионы (катионы) начнут перемещаться к отрицательному электроду — катоду, а отрицательные ионы (анионы) — к положительному электроду (аноду). В цепи возникает электрический ток, обусловленный направленным движением ионов обоих знаков.

Электрический ток - определение и понятия с примерами
Рис. 100 

Таким образом, электрический ток в растворах электролитовэто упорядоченное движение ионов.
Если ток протекает в растворе медного купороса, то вскоре на катоде образуется тонкий слой меди. Следовательно, в растворе под действием электрического поля к катоду перемещаются положительные ионы Cu2+, при контакте с катодом они присоединяют недостающие электроны и в виде нейтральных атомов осаждаются на электроде. В отличие от металлов ток в электролите сопровождается переносом вещества.

Процесс выделения вещества на электродах при протекании электрического тока в растворах или расплавах электролитов называют электролизом.

В 1833—1834 гг. Майкл Фарадей опытным путём установил количественные соотношения для описания явления электролиза. Первый закон Фарадея для электролиза позволяет рассчитать массу вещества, выделившегося на электроде.

Масса вещества, выделившегося на электроде при электролизе, прямо пропорциональна силе тока и времени его прохождения.

Электрический ток - определение и понятия с примерами

где m — масса выделившегося на электроде вещества; k — коэффициент пропорциональности — электрохимический эквивалент данного вещества; I — сила тока в цепи; t — время прохождения тока.

Из первого закона Фарадея можно экспериментально определить значение электрохимического эквивалента данного вещества.

Электрохимический эквивалент определяется отношением массы вещества, выделившегося на электроде при электролизе, к электрическому заряду, прошедшему через электролит.

Единица электрохимического эквивалента в СИ — один килограмм на кулон (Электрический ток - определение и понятия с примерами).

Электролиз широко применяется в промышленности. Используют электролиз (гальваностегию) для покрытия металлических изделий тонким слоем другого металла (никелирование, хромирование).

Пропуская электрический ток через расплавы некоторых солей, можно выделять металлы в чистом виде. Так получают алюминий, рафинированную (сверхчистую) медь и другие металлы.

Посредством электролиза очищают металлы от примесей, например неочищенную медь, добытую из руды. Её отливают в форме толстых листов, которые потом помещают в ванну как аноды. При этом медь анода растворяется, примеси, содержащие ценные и редкие металлы, осаждаются, а на катоде остаётся чистая медь.

В 1836 г. Б. Якоби предложил процесс получения отслаиваемых покрытий (гальванопластику) и применил его для изготовления полых фигур, украсивших Исаакиевский собор в Санкт-Петербурге.

Электрический ток в полупроводниках

Изучая электрические явления, мы в основном пользовались материалами и приборами, изготовленными из веществ, которые были либо изоляторами (стекло, бумага, воздух, пластмасса и др.), либо проводниками (медь, алюминий, сталь, уголь, электролиты и др.). Однако большинство из известных веществ по их электрическим свойствам нельзя отнести ни к проводникам, ни к диэлектрикам. Этот обширный класс веществ, которые по своему удельному сопротивлению занимают промежуточное место между проводниками и диэлектриками, называют полупроводниками.

К полупроводникам относятся 12 химических элементов (В, С, Si, Ge, Sn, P, As, Sb, S, Se, Те, I), соединения элементов III и V групп (InSb, GaAs и др.), соединения элементов II и VI групп (CdS, ZnO и др.), ряд других соединений, некоторые органические вещества. Наибольшее применение в науке и технике имеют полупроводники германий (Ge) и кремний (Si).
Чем полупроводники отличаются от проводников?

Способность любого вещества проводить электрический ток под действием электрического поля называют электрической проводимостью, электропроводностью, или просто проводимостью. Тип проводимости обусловлен видом носителей тока. Металлы имеют электронную проводимость. В электролитах носители тока — свободные положительные и отрицательные ионы, поэтому они имеют ионную проводимость. Чем меньше электрическое сопротивление проводника, тем больше его проводимость, и наоборот. Электрическое сопротивление металлов с повышением температуры возрастает прямо пропорционально (см. рис. 82 на с. 68).

По результатам научных исследований, полупроводники обладают такими основными свойствами.

  1. Электропроводность полупроводников в значительной степени зависит от состояния вещества (температура, освещение, наличие примесей и т. д.).
  2. C повышением температуры электрическое сопротивление полупроводников в отличие от металлов резко падает.
  3. Прохождение тока в полупроводнике не связано с переносом вещества, то есть ток в полупроводниках обусловлен направленным движением электронов, а не ионов.

Рассмотрим подробнее свойства полупроводников.

Опыт. Включаем в электрическую цепь полупроводниковый элемент (рис. 101). При его нагревании стрелка гальванометра показывает возрастание силы тока в цепи. Следовательно, сопротивление полупроводника, а значит, и удельное сопротивление, уменьшаются с повышением его температуры.

Электрический ток - определение и понятия с примерами
Рис. 101

На рисунке 102 изображён график зависимости сопротивления полупроводникового элемента от его температуры. Видим, что сопротивление полупроводника резко уменьшается с повышением температуры.

Электрический ток - определение и понятия с примерами
Рис. 102

Если нагревание полупроводника прекратить, то стрелка гальванометра вернётся в прежнее положение. Полупроводник охлаждается, его сопротивление возрастает.

Электрическое сопротивление полупроводников зависит также от степени их освещения. На рис. 103, а полупроводниковый фотоприёмник закрыт заслонкой, ток в цепи очень мал. При освещении полупроводника (рис. 103, б) сила тока в цепи заметно возрастает. Это говорит об уменьшении сопротивления полупроводника под действием света.

Электрический ток - определение и понятия с примерами
Рис. 103

Зависимость сопротивления полупроводников от освещения и нагревания связана с внутренним строением этих материалов.

Один из типичных полупроводников — германий с порядковым номером 32. Его четыре электронные оболочки имеют 32 электрона. На первой оболочке находятся 2, на второй — 8, на третьей — 18, на четвёртой — 4 электрона (рис. 104, а). Электроны трёх внутренних оболочек не участвуют в химических реакциях. Электроны внешней оболочки слабо связаны с ядром атома. Их называют внешними, или валентными электронами, поскольку они определяют валентность данного элемента — способность его атомов вступать в химическую связь с определённым количеством других атомов. Валентность германия, атом которого имеет на внешней оболочке 4 электрона, равна четырём.

Электрический ток - определение и понятия с примерами
Рис. 104

При сближении двух атомов их валентные электроны вследствие слабой связи со своими ядрами легко взаимодействуют, образуя устойчивую химическую связь, которую называют ковалентной.

В твёрдом состоянии атомы германия расположены в узлах кристаллической решётки. У каждого атома есть четыре равноудалённых соседа, с которыми его объединяют парноэлектронные (ковалентные) связи. На рис. 104, б изображена условная плоская схема структуры связей в кристалле германия (аналогичная схема у кремния, который также находится в IV группе и обладает подобными химическими и физическими свойствами). При низких температурах все валентные электроны атомов заняты в этих связях и не являются свободными. За неимением свободных электронов полупроводники при низких температурах ведут себя как диэлектрики. Для того чтобы полупроводник проводил ток, надо разорвать парноэлектронные связи, то есть освободить электроны.

При нагревании либо освещении кристалла некоторые электроны получают избыточную энергию и становятся свободными. Чем больше нагревается или освещается полупроводник, тем больше в нём появляется свободных электронов и тем меньше его электрическое сопротивление. Освободившийся электрон покидает своё место в системе связей между атомами — появляется вакансия — незаполненная электронная связь, которую называют дыркой. Количество таких дырок тем больше, чем больше электронов освобождается в кристалле при нагревании или освещения.

Дырка ведёт себя как положительный заряд. Дело в том, что недостаток отрицательного заряда в системе электронной связи равносилен наличию в этом месте положительного заряда. Заряд дырки равен по значению заряду электрона. Дырка может захватывать электрон от соседней связи, в результате там появляется новая дырка, а первичная — исчезает. Это эквивалентно перемещению дырки в пространстве, поэтому она может перемещаться в кристалле как свободный электрон.

Свободные электроны и дырки в кристалле полупроводника находятся в состоянии хаотического теплового движения, но характер их движения существенно изменится, если к кристаллу приложить напряжение. Электрическое поле упорядочит движение как свободных электронов, так и дырок. Положительные дырки под действием электрического поля перемещаются к отрицательному полюсу источника — катоду, а свободные электроны к положительному полюсу — аноду. В кристалле полупроводника возникает электрический ток, обусловленный движением носителей двух типов. Говорят, что в полупроводнике имеются дырочная и электронная проводимости.

Проводимость химически чистых полупроводников, возникающая при их нагревании или освещении, называется собственной проводимостью.

Собственная проводимость полупроводников при комнатной температуре небольшая. Оказывается, чтобы значительно увеличить электрическую проводимость полупроводника (то есть уменьшить его электрическое сопротивление), в чистый полупроводник необходимо внести специальные примеси. Проводимость полупроводников, обусловленную наличием примесных атомов, называют примесной проводимостью. Различают электронную примесную проводимость и дырочную примесную проводимость.

Электронная примесная проводимость возникает при замещении некоторых атомов германия или кремния атомами другого вещества с пятью валентными электронами, например мышьяка (As) или сурьмы (Sb). Тогда четыре электрона сурьмы обеспечивают ковалентные связи с соседними атомами, а пятый электрон, слабо связанный с атомом примеси, становится свободным. В веществе объёмом 1 см3 содержится до 1022 атомов. Замещение одного атома германия из миллиона на атом сурьмы или мышьяка приводит к тому, что в каждом кубическом сантиметре полупроводника появляются почти 1016 свободных электронов. Именно эта способность даёт возможность полупроводнику проводить электрический ток. Примеси, которые легко отдают электроны и, следовательно, увеличивают количество свободных носителей, называют донорными примесями, пли донорами.

Итак, за счёт вкрапления в германий или кремний атомов пятивалентного элемента получаем полупроводники с примесной электронной проводимостью (рис. 105, а). Их называют полупроводниками n-типа (от латинского слова negatiυus — отрицательный).

Электрический ток - определение и понятия с примерами
Рис. 105

Дырочная примесная проводимость возникает, если в полупроводниковом кристалле некоторые атомы замещаются другими атомами, имеющими три валентных электрона, например атомами индия (In). В этом случае для образования ковалентной связи с соседними атомами примесному атому необходим ещё один электрон (рис. 105, б). Поэтому в том месте, куда попадает атом примеси, образуется дырка (дефицит электрона). Если заменить один из миллиона атомов основного вещества атомом трёхвалентной примеси, то в каждом кубическом сантиметре полупроводника образуются до 1016 дырок.

Примеси этого типа называют акцепторными (приёмными), или акцепторами. Такой полупроводниковый кристалл обеспечивает электрический ток благодаря дрейфу дырок, которые переносят положительный электрический заряд.

Итак, при вкраплении в германий или кремний атомов трёхвалентного вещества получаем полупроводники с примесной дырочной проводимостью. Их называют полупроводниками р-типа (от латинского positivus — положительный).

Как уже отмечалось, электрическое сопротивление полупроводников зависит от его температуры. Это свойство полупроводников используют для измерения температуры окружающей среды (в диапазонах от -269 до -193 0С; от -103 до + 297 0С; свыше +1000 0С) по силе тока в цепи с полупроводником. Такие приборы называют термисторами, или терморезисторами. Их изготовляют в виде стержней, трубок, дисков, шайб, бусин размером от нескольких микрометров до нескольких сантиметров и применяют для дистанционного измерения температуры, противопожарной сигнализации и пр.

Электрическая проводимость полупроводника возрастает при разрушении связей и образовании свободных электронов и дырок за счёт энергии света, падающего на полупроводник. Это явление называют внутренним фотоэлектрическим эффектом; используют в полупроводниковых приборах — фоторезисторах. C помощью фоторезисторов определяют качество поверхностей, контролируют размеры изделий и т. д.

Позднее вы будете изучать устройство и работу различных полупроводниковых приборов: диодов, транзисторов, электронных интегральных схем. Именно благодаря им электронная аппаратура стала малогабаритной, экономичной и надёжной: первые компьютеры размещались в огромных залах, а современные — помещаются на ладони, музыкальный плейер не стоит на столе, а висит на шнурке. Полупроводниковый лазер даёт возможность записать данные на CD- или DVD-дисках, а флеш-накопитель — переносить и хранить гигантские объёмы информации.

Большой вклад в развитие физики полупроводников, внесли украинские учёные К. Д. Товстюк (1922-2004) и В. Е. Лошкарёв (1903-1974), создавший научную школу специалистов по физике полупроводников.

Электрический ток в газах

Газы являются хорошими изоляторами, поскольку в обычных условиях — при низких температурах и отсутствии внешнего облучения (ультрафиолетового, рентгеновского, радиоактивного) — они состоят из нейтральных атомов или молекул. У них нет свободных электрических зарядов, упорядоченное перемещение которых создаёт электрический ток. Но при определённых условиях можно получить электрический ток и в газах.

Опыт 1 Зарядим (например, от электрофорной машины) алюминиевые диски, соединённые проводниками с выводами электрометра (рис. 106, а). Видим, что отклонение стрелки электрометра остаётся постоянным, поскольку электрическая проводимость воздуха в условиях комнатной температуры и сухого воздуха очень мала, и пластины практически не разряжаются.

Внесём в пространство между дисками пламя спички или свечи (рис. 106, б). Наблюдаем быстрый разряд электрометра. Следовательно, воздух вследствие значительного повышения температуры уже обладает проводимостью и замыкает цепь, то есть в нагретом газе проходит электрический ток.

Электрический ток - определение и понятия с примерами
Рис. 106

Процесс прохождения электрического тока в газах называют газовым разрядом.

При нагревании или излучении часть атомов газа ионизируется — распадается на положительно заряженные ионы и электроны. В газе образуются также отрицательно заряженные ионы.

Ионизация газов при нагревании объясняется тем, что некоторые молекулы перемещаются так быстро, что часть из них при столкновении распадается, превращаясь в ионы. Чем выше температура газа, тем больше образуется ионов. В нашем опыте пламя свечи исполняет роль ионизатора, то есть источника ионов.

Явление, вызывающее ионизацию газа и дальнейшее развитие газового разряда, называют ионизатором.

Как ионизатор действуют рентгеновские лучи, а также радиоактивное излучение (его вы будете изучать позднее). При нормальных условиях окружающий воздух всегда в определённой степени ионизирован под воздействием солнечных лучей и космического излучения (поток быстрых заряженных частиц, поступающих на Землю из глубин Космоса).

Механизмы проводимости газов, растворов и расплавов электролитов аналогичны. Различие в том, что в газах отрицательный заряд переносится преимущественно не отрицательными ионами, а свободными электронами.

Таким образом, в газах сочетаются электронная проводимость, подобная проводимости металлов, с ионной проводимостью, которая аналогична проводимости водных растворов и расплавов электролитов.

Если мы прекращаем нагревать либо облучать газ, то он снова становится диэлектриком. Ток прекращается тогда, когда все ионы и электроны достигают электродов. Кроме того, при сближении электрона и положительно заряженного иона снова образуется нейтральный атом. Этот процесс называют рекомбинацией заряженных частиц.

Если же внешнего электрического поля нет, то заряженные частицы исчезают только при рекомбинации, и газ становится диэлектриком.

В газах разряд можно наблюдать без нагревания и облучения. Иногда он может поддерживать себя самостоятельно. При каких условиях это возможно?

Опыт 2 Возьмём запаянную и наполненную воздухом стеклянную трубку с двумя металлическими электродами, к которым прикладываем напряжение, собрав цепь, изображённую на рисунке 107. Предположим, что на газ в трубке действует какой-либо ионизатор. Если напряжение между электродами трубки малое, то положительно заряженные ионы перемещаются к отрицательному электроду, а электроны и отрицательно заряженные ионы — к положительному. В результате в трубке возникает электрический ток, то есть происходит газовый разряд.

Поскольку в пространстве между электродами одновременно происходит процесс рекомбинации, то не все образованные электроны и ионы достигают электродов трубки.

C увеличением напряжения между электродами сила тока в цепи возрастает. Потом наступает момент, когда сила тока не изменяется. Ток достигает насыщения (рис. 108). Если действие ионизатора (нагревание, облучение) прекращается, то прекращается и разряд, поскольку другие источники ионов отсутствуют. Такой разряд называют несамостоятельным разрядом.

А что же происходит с разрядом в газе, если увеличивать напряжение на электродах трубки?

Опыт показывает, что в газе с увеличением напряжения на электродах трубки сила тока с определённого значения снова увеличивается (рис. 109). Причина в том, что в газе дополнительно происходит ионизация электронным ударом при столкновении электронов, ускоренных электрическим полем, с атомами газа. При этом образуется значительно больше ионов, чем в результате действия ионизатора. Сила тока возрастает в тысячи раз, а количество ионов может стать таким, что отпадает потребность во внешнем ионизаторе.

Поскольку такой разряд не требует для своего поддержания внешнего ионизатора, его называют самостоятельным разрядом.

В зависимости от свойств и состояния газа, характера и размещения электродов, приложенного к электродам напряжения возникают различные виды самостоятельного разряда.

Тлеющий разряд наблюдается при низких давлениях (десятые и сотые доли миллиметра ртутного столба) и напряжении между электродами в несколько сотен вольт.

Тлеющий разряд используют в рекламных трубках (рис. 110). Если трубка наполнена неоном, то возникает красное свечение, если аргоном — синевато-зелёное. В лампах дневного света используют разряд в парах ртути.

Электрический ток - определение и понятия с примерами
Рис. 110

Электрическая дуга — это явление возникновения яркого . светящегося столба газа между двумя угольными электродами при низком напряжении.

Электрическую дугу «зажигают» так. Сначала сближают и вводят в контакт угольные электроды, замыкая цепь. В результате тепловою действия тока, протекающего в точечных контактах с большим электрическим сопротивлением, концы электродов накаляются до свечения. C поверхности электродов при этом с огромной скоростью вылетают электроны и при столкновении ионизируют газ в прилегающем пространстве (о явлении термоэлектронной эмиссии читайте на с. 103). Если теперь развести электроды в разные стороны, то электрический ток в цепи не прекращается, он проходит через ионизированный газ, что сопровождается его накаливанием и ярким свечением.

Сила тока в малой дуге достигает нескольких ампер, а в мощных дугах — нескольких сотен ампер при напряжении около 50 В.

Во время горения дуги воздух в промежутке между угольными электродами нагревается до нескольких тысяч градусов и, поднимаясь вверх в результате конвекции, изгибает светящийся столб в виде дуги, чем и объясняется название этого вида газового разряда (рис. 111).

Электрический ток - определение и понятия с примерами
Рис. 111

Дуговой разряд — мощный источник света. Его используют в прожекторах, проекционных аппаратах и киноаппаратах. В металлургии применяют электропечи, в которых источником тепла является дуговой разряд. Используют его также для сварки металлов.

При коронном разряде светящаяся область напоминает корону, он образуется при атмосферном давлении вблизи острых частей проводника с большим электрическим зарядом.

Газ в этом случае ионизируют ударом электроны, ускоренные сильным электрическим полем, возникающим возле острых заряженных проводников.

Перед грозой или во время грозы часто на вершинах и острых углах высоких предметов возникает кистеобразное свечение, например на корабельных мачтах (рис. 112). Издавна это свечение называют огнями святого Эльма, иногда оно наблюдается даже на кончиках ружейных штыков.
Коронный разряд может возникнуть на проводах высоковольтных линий, что приводит к потерям электроэнергии. Чем выше напряжение в линии, тем толще провод следует использовать для предотвращения коронного разряда.

Искровой разряд возникает при высоком напряжении между  электродами в воздухе (рис. 113, а) и имеет вид пучка ярких зигзагообразных полос, ответвляющихся от тонкого канала.

При искровом разряде газ ионизируется ударом ускоренных сильным электрическим полем электронов, возникающих в некоторых местах между электродами вследствие природной ионизации воздуха.
C помощью искрового разряда можно обрабатывать детали из тугоплавких металлов, поскольку большая энергия этого разряда выделяется в небольшом объёме за очень малый промежуток времени. При отсутстивии теплообмена с окружающей средой в месте разряда температура металла резко повышается, происходит его испарение.

Примером мощного искрового разряда является молния (рис. 113, б). Изучением этого явления природы занимались многие учёные, в частности Б. Франклин, М. В. Ломоносов, Г. В. Рихман. В 1753 г., исследуя атмосферное электричество, Г. В. Рихман погиб от удара молнии.

Многолетними исследованиями установлено, что при движения воздуха за счёт конвекции воздушные потоки и облака в результате столкновений электризуются. При этом часть облака (например, верхняя) электризуется положительно, а часть — отрицательно.

Напряжение между двумя облаками или между облаком и Землёй достигает десятков миллионов вольт. В результате между облаками или между облаком и Землёй возникает гигантская искра — молния (рис. 114). Длина молнии достигает нескольких километров, а диаметр её канала иногда превышает 6 м. Сила тока в канале молнии огромная: от 1—2 до 200 кА. Продолжительность разряда небольшая — тысячные доли секунды. Поэтому общий заряд, протекающий при одной вспышке молнии, не превышает десятков или сотен кулонов.

Электрический ток - определение и понятия с примерами
Рис. 114

Для защиты сооружений (зданий, опор линий электропередачи и т. д.) поблизости устанавливают мачту с заострённым металлическим стержнем, который надёжно соединяют толстым проводом с закопанным глубоко в землю металлическим предметом, то есть заземляют (рис. 115). Это устройство называют молниеотводом (часто — громоотводом).

Электрический ток - определение и понятия с примерами
Рис. 115

Поясним упрощённо принцип действия молниеотвода. Грозовое облако своим электрическим полем наводит в молниеотводе электрический заряд, противоположный по знаку заряду облака. Этот заряд создаёт около острия молниеотвода сильное электрическое поле, в котором начинается «тихий» коронный разряд, забирающий на себя энергию взаимодействия наведённого облаком электрического заряда, чем уменьшает вероятность развития молнии. Размеры территории, защищённой молниеотводом на поверхности Земли, определяются высотой молниеотвода.

Электрический ток в вакууме. Если из стеклянной трубки в опыте 2 на с. 99 (см. рис. 107) с помощью специального насоса откачать воздух, то в ней образуется вакуум, то есть безвоздушное пространство. При этом измерительный прибор показывает, что в цепи трубки тока нет. Этот факт можно объяснить: хотя в промежутке между электродами существует электрическое поле, но свободные носители электрического заряда отсутствуют, поэтому вакуумный промежуток является хорошим изолятором.

И все же в такой вакуумной трубке возможен электрический ток. Если рядом с катодом трубки поместить миниатюрный электронагреватель (нить накала), который нагреет катод до температуры видимого свечения (900—1500 °C), то с его поверхности начнут «испаряться» электроны, образующие электронное «облако», то есть появляются свободные носители электричества. Под действием электрического поля электроны будут перемещаться к аноду и замкнут цепь — измерительный прибор покажет наличие тока. Явление «испарения электронов», названное термоэлектронной эмиссией, в 1883 г. впервые наблюдал Т. Эдисон (эффект Эдисона).

Описанный вакуумный прибор имеет важную особенность — одностороннюю проводимость. Если полярность источника тока, соединённого с трубкой, поменять на противоположную (катод соединить с положительным полюсом источника, а анод — с отрицательным), то ток в цепи исчезнет. В этом случае положительный катод «не отпускает» отрицательные электроны, а отрицательный анод, в свою очередь, отталкивает их назад, то есть цепь не замыкается. Позднее на основе этого свойства были созданы выпрямители переменных (периодически изменяющих направление) токов, а также электронно-лучевые трубки, которые широко применяли в телевизорах и мониторах компьютеров.

Кстати:

  • Молния — достаточно частое явление на Земле. Учёные подсчитали, что ежедневно на земном шаре бывает почти 44 000 гроз (приблизительно одна гроза каждые 2 с). Грозы преобладают во второй половине дня. Их продолжительность — приблизительно 1 ч. Но в тропиках и горах они иногда продолжаются по 12—13 ч.
  • Вольтов столб и батарея открыли возможности для широких экспериментов с сильным электрическим током. В 1802 г. профессор Петербургской медико-хирургической академии В. В. Петров (1761-1834) сконструировал самую мощную для того времени батарею. Она состояла из 4 200 медных и цинковых кружков, помещённых в четыре деревянных ящика. Присоединив медным проводом к полюсам батареи два угольных стержня (электроды), учёный приблизил их один к другому и увидел, что между ними вдруг вспыхнула яркая дуга. Она осветила лабораторию, куски металла в ней очень быстро плавились. Так была открыта электрическая дуга.
  • Способ сварки металлов с помощью электрической дуги в 1881 г. предложил уроженец с. Мостовое Николаевской области, изобретатель Николай Николаевич Бенардос (1842-1905). Он запатентовал почти 100 изобретений в сфере транспорта и энергетики. Н. Бенардос сконструировал угольные электроды разных форм и комбинированные электроды: один из электродов — угольный, а другим служит свариваемый металлический предмет; предложил специальное приспособление для сварки в вертикальном положении; впервые использовал электромагнит для закрепления свариваемых деталей в заданном положении; создал несколько конструкций сварочных полуавтоматов и автоматов; разработал способы подводного сваривания и резания металлов, сварки в газовой струе, точечной и шовной контактной сварки.
    Электрический ток - определение и понятия с примерами
    Николай Бенардос
  • Славные традиции достижений украинских учёных в исследовании проблем сварки металлов продолжил Евгений Оскарович Патон (1870-1953) — основа-Николай Бенардос тель отечественной школы сварки. По его инициативе при прежней АН УССР оганизовали лабораторию (1929 г.), которая в 1934 г. была преобразована в Институт электросварки, директором и научным руководителем которого был Е. О. Патон. Основные труды учёного посвящены научным и технологическим основам дугового сварочного процесса и проблемам его автоматизации; созданию электросварочной аппаратуры.
    Электрический ток - определение и понятия с примерами
    Евгений Патон​
  • В Институте электросварки АН УССР непосредственно под руководством Е. О. Патона в 1939-1940 гг. была завершена разработка метода высокопродуктивной автоматической сварки под флюсом. Институт разработал технологию сварки бронированной
  • стали, что позволило создать поточное механизированное производство бронетанковых корпусов. C помощью автоматической сварки под флюсом было налажено производство авиабомб, артиллерийских снарядов. Довоенный и военный периоды деятельности Института, которому в 1945 г. было присвоено имя Е. О. Патона, можно рассматривать как становление новой научной школы (В. В. Шеверницкий, А. А. Казимиров, Г. В. Раевский, А. М. Макара, Б. Е. Патон, В. И. Медовар, Д. М. Раб-кин, И. И. Фрумин). Именем Е. О. Патона назван сконструированный им мост через Днепр в Киеве (1953 г). Позднее дело отца продолжил сын — Б. Е. Патон, возглавивший Институт электросварки имени Е. О. Патона в 1953 г.
  • Борис Евгеньевич Патон — выдающийся учёный в области сварки, металлургии и технологии металлов, организатор науки, государственный и общественный деятель. Президент Национальной академии наук Украины с 1962 г. Возглавляемый Б. Е. Патоном Институт электросварки вырос в мощный научно-технический комплекс, в структуре которого научно-исследовательский институт, конструкторско-технологические и экспериментальные подразделения, три исследовательских завода, а также инновационные организации, научно-инженерные и аттестационный центры. Под его руководством и при непосредственном участии в Институте проведены глубокие исследования и получены важные результаты по разработке прогрессивных технологий неразъёмного соединения, обработки металлов и неметаллов в разных условиях и средах. К ним относятся сварка и наплавка под флюсом, сварка в защитных газах сплошным и порошковым проводом, электрошлаковая, стыковая сварка плавлением, газотермическое напыление, лучевые технологии и другие процессы.
    Электрический ток - определение и понятия с примерами
    Борис Патон
    Фундаментальные исследования Б. Е. Патона и его учеников взаимодействия сварочных источников нагревания с расплавленным металлом заложили основу для создания новой отрасли металлургии — специальной электрометаллургии. Благодаря этому стало возможным получение сверхчистых специальных сталей и сплавов, цветных металлов, а также уникальных композиционных материалов. Открылись перспективы для создания новейших конструкционных и функциональных материалов XXI века.
    Борис Евгеньевич внёс большой вклад в создание новых типов сварных, конструкций, индустриальных способов сварки магистральных трубопроводов, крупногабаритных резервуаров для хранения нефти, кожухов доменных печей, высотных башенных конструкций.
    Впервые Б. Е. Патон предложил исследования в открытом космосе в сфере технологии металлов, в создании уникальных конструкций. Под его непосредственным руководством в 1984 г. космонавты С. Е. Савицкая и В. А. Джанибеков впервые в мире провели в открытом космосе исключительно важные исследования и впервые осуществили на практике сваривание, резание и напыление металлов.
    Под его руководством сотрудники Института и учёные-медики создали новый способ соединения (сваривания) мягких тканей человека и животных, который широко используется в хирургической практике.

Безопасность человека во время работы c электрическими приборами и устройствами

Электрические цепи рассчитывают на определенную силу тока. Если сила тока в цепи превышает допустимую, то провода могут перегреться, а их изоляция — загореться.

Причины значительного увеличения силы тока в цепи — одновременное включение нескольких мощных потребителей тока (например, электроплитки, электроутюга, стиральной машины, водонагревателя) либо короткое замыкание.

Опыт. Собираем цепь из источника тока напряжением 36 В, электрической лампы, ключа и отрезка тонкого провода. Замыкаем ключ — лампа светится. Теперь к зажимам лампы присоединим перемычку и снова замыкаем цепь. Лампа гаснет, а тонкий проводник раскаляется до свечения (рис. 116) и может перегореть со вспышкой.

Электрический ток - определение и понятия с примерами
Рис. 116

Коротким замыканием называют соединение концов участка цепи проводником, сопротивление которого очень мало по сравнению с сопротивлением участка цепи.

Короткое замыкание может возникнуть при ремонте проводки под током (рис. 117) или при случайном касании оголённых проводов. Сопротивление цепи при коротком замыкании незначительно, поэтому в цепи резко возрастает сила тока, при этом провода могут загораться. Чтобы предупредить это, в электросеть обязательно включают предохранители.
Каково назначение и устройство предохранителей?

Электрический ток - определение и понятия с примерами
Рис. 117

Назначение предохранителей — сразу же отключить линию, если сила тока вдруг превысит допустимую норму.

Основная часть предохранителя (рис. 118) — провод C из легкоплавкого металла (например, свинца), размещённый внутри фарфоровой пробки П. Пробка имеет винтовую нарезку H и центральный контакт К. Нарезка соединена с центральным контактом свинцовым проводом. Пробку вкручивают в патрон, который находится внутри фарфоровой коробки.

Электрический ток - определение и понятия с примерами
Рис. 118

Следовательно, свинцовый провод — это часть общей цепи. Толщина свинцового провода должна выдерживать определённую силу тока, например<, 6,10,16, 25, 32 А (рис. 119). Если сила тока в цепи превысит допустимое значение, то свинцовый провод расплавится, и цепь разомкнётся.
Предохранители с проводником, который плавится, называются плавкими предохранителями.

Электрический ток - определение и понятия с примерами
Рис. 119

На рисунке 120 изображён такой предохранитель со вставкой, которую в случае перегорания можно заменить.

Электрический ток - определение и понятия с примерами
Рис. 120

В различных приборах применяют разные предохранители (рис. 121). На рисунке 122 показан предохранитель-автомат, действующий на основе теплового расширения тел.

При замене предохранителя обязательно соблюдайте следующие правила.
Прежде чем заменить перегоревшие при коротком замыкании предохранители, нужно его обнаружить и устранить. Для этого необходимо:

  1. выключить в квартире или доме все потребители тока;
  2. вставить в гнездо одного предохранителя лампу накаливания, которая соответствует напряжению в сети, а в гнездо другого предохранителя — пробку;
  3. если замыкание связано не с потребителями, а с подводными проводами квартиры, то лампочка ярко будет светить;
  4. если замыкания в подводных проводах нет (спираль лампы накаляется слабо), то необходимо поочерёдно включать все потребители тока;
  5. если замыкания нет ни в одном из потребителей, значит, в вашей квартире сеть перегружена. Выключите лишние потребители тока

При работе с электроприборами необходимо строго и неукоснительно выполнять меры предосторожности. Если этим пренебрегать, то вашей жизни угрожает смертельная опасность в случае электротравмы. Электротравма — это поражение человека электрическим током. Она возникает при контакте тела человека с электрическим кабелем, прибором или оборудованием, которые находятся под напряжением при условии их неисправности либо нарушении правил безопасности во время ремонта и эксплуатации.
При поражении электрическим током имеют значение его сила, напряжение, частота, а также другие факторы: место поражения, время влияния тока на организм, состояние окружающей среды, особенности организма, кожи и одежды пострадавшего.

Человеческое тело — проводник. При контакте частей тела с оголёнными проводами или оборудованием, которые находятся под напряжением, через него проходит ток, действие которого может быть более или менее опасным в зависимости от силы тока. Ток силой 0,6—1,5 мА человек уже ощущает как пощипывание кожи и болевые ощущения.

Ток силой 5—25 мА может провоцировать непроизвольные и неконтролируемые судороги (сокращение мышц), при этом поражённый не может самостоятельно отключиться от источника опасности. Поскольку мышцы-сгибатели сильнее, чем мышцы-разгибатели, то, касаясь неисправного электроприбора или кабеля, мы рискуем ухватиться за опасную поверхность. Понятно, что лучше, если вы прикоснулись к включённому электроприбору тыльной стороной ладони. Поражение током при этом возможно, но если мышцы-сгибатели сократятся, то рука оттолкнётся от источника опасности.
Ток в пределах 50—350 мА вызывает фибрилляцию (беспорядочные сокращения) сердца, то есть нарушение сердечной деятельности, иногда с остановкой сердца.

Степень травматизма зависит от места поражения, какие именно органы поражены током. Есть несколько вариантов прохождения электрического тока через тело человека по линии: нога — нога; рука — рука; рука — нога. Наиболее опасно, если ток поражает сердце.

Степень поражения зависит и от длительности действия тока на организм.
Существенное значение для электротравматизма имеет состояние окружающей среды. Влажность, токопроводящий пол — всё это увеличивает действие тока на организм человека. В особенности опасно пользоваться электроприборами в подсобных помещениях и на открытом воздухе.

Последствия электротравм зависят и от состояния организма пострадавшего. Опьянение, болезненное состояние, юный возраст, психологический стресс уменьшают сопротивление организма и увеличивают степень поражения. Специальная одежда, например резиновые перчатки и сапоги, может защитить от действия тока на организм человека или существенно ослабить его.

Нормативно установлены значения наибольшего допустимого напряжения касания при достаточно продолжительном действии: для постоянного тока — 8 В (ток протекания 1 мА); для переменного тока — 2 В (ток протекания 0,3 мА).

Поскольку значение электрического сопротивления человеческого тела может колебаться от 100 Ом до нескольких десятков килоом, то безопасным считают постоянное напряжение 25 В, а в помещениях с повышенной влажностью — напряжение 15 В.

Учебный физический кабинет в школе относится к помещениям с повышенной опасностью. Во время выполнения лабораторных и других работ учащиеся должны строго соблюдать правила электробезопасности, которые в каждом кабинете физики изложены на специальном плакате.
Все, кто пользуется электрическим оборудованием, обслуживает или ремонтирует его, всегда должны помнить следующее.

  1. Очень опасно одновременно прикасаться обеими руками к двум оголённым проводам.
  2. Очень опасно прикасаться к оголённому проводу, стоя на земле, на влажном или бетонном полу.
  3. Опасно пользоваться неисправными электрическими приборами. Их должны периодически проверять специалисты.
  4. Нельзя собирать, разбирать или исправлять что-либо в электроприборе, не отключив его от источника тока.
  5. Нельзя разбирать выключатели, розетки и другую арматуру электросети, не выкрутив предохранители.

Если вы увидите предупредительные знаки (рис. на с. ГУ форзаца), то это означает: подобное делать категорически запрещается!

А как оказать первую помощь человеку, поражённому электрическим током?
Запомните: при поражении электричеством дорога каждая минута, нужно немедленно принимать меры по оказанию поражённому первой помощи.

Прежде всего необходимо освободить человека от контакта с проводом под напряжением. Если это случилось в помещении, где имеются выключатель или штепсель, то нужно выключить ток выключателем либо вытянуть штепсельную вилку. Если это произошло в цепи, где нет выключателя, надо выкрутить предохранители или выключить главный выключатель — он находится возле счётчика.
Если выключатели размещены очень далеко, а человек находится под током, то необходимо надеть резиновые перчатки или обувь, снять с поражённого сухой деревянной (или из другого изоляционного материала) палкой провод либо перерезать его ножом, перерубить топором, «перекусить» кусачками. Но следует помнить, что во всех перечисленных инструментах ручки должны быть изготовлены из изоляционного материала.

Освободив потерпевшего от тока, надо положить его на спину, освободить грудь от одежды, вызвать врача или «скорую помощь», а если это невозможно, то сделать искусственное дыхание.

  • Заказать решение задач по физике

Пример №14

Почему электролиты имеют ионную проводимость?
Ответ: носителями тока в электролитах являются ионы обоих знаков. Свободных электронов электролиты не имеют.

Пример №15

При прохождении электрического тока в растворе медного купороса на катоде выделилось 52,8 г меди. Какой заряд прошёл через электролит, если электрохимический эквивалент меди равен Электрический ток - определение и понятия с примерами?

Дано:
m = 52,8 г = 0,0528 кг
k = 0,33 ∙ Электрический ток - определение и понятия с примерами
Решение
Чтобы определить заряд, прошедший
в электролите, используем формулу
q = It.
 
q — ?

В формуле, выражающей закон Фарадея m = kit, заменим произведение It на q и получим выражение для заряда : Электрический ток - определение и понятия с примерами.
Подставив значения физических величин, получим:

Электрический ток - определение и понятия с примерами
Ответ: через электролит прошёл заряд 160 кКл.

Пример №16

Почему в технике преимущественно используют не чистые полупроводники, ас донорными или акцепторными примесями?
Ответ: при введении в полупроводниковый кристалл примеси можно увеличить электропроводность кристалла кремния в миллиард раз.

Пример №17

Почему электрический ток в газах называют «электрическим разрядом»?
Ответ: при прохождении тока в газе между двумя разноимённо заряженными телами эти тела разряжаются.

Историческая справка:

Гальвани Луиджи (09.09.1737-04.12.1798) — итальянский физик и физиолог. Родился в Болонье. Окончил Болонский университет, с 1775 г. — профессор этого университета.

Электрический ток - определение и понятия с примерами
Луиджи Гальвани

C 1773 г занимается анатомическим исследованием движений мышц лягушек, а в 1780 г. провёл на них первые электрофизиологические опыты. После 11 лет исследований, что привело учёного к открытию (1786) в ткани лягушки кратковременных импульсов электрического тока («животного» электричества), он опубликовал их результаты в «Трактате о силах электричества при мышечном движении» (1791). Л. Гальвани отмечал: если соединить металлическим проводником мышцы и нервы только что препарированной лягушки, то сразу же происходит сокращение её мышц. Сокращения становятся более сильными и продолжительными, если проводник состоит из двух разнородных металлов, например железа и меди или серебра. Л. Гальвани сделал вывод, что сокращения мышц лягушки обусловлены возникновением в них электрического тока. Однако причину этого учёный ошибочно усматривал в наличии в каждом животном собственного «животного» электричества.

Л. Гальвани — один из основателей учения об электричестве, его исследования с электрофизиологик
«животным» электричеством положили начало новому научному направлению — электрофизиологик.

Вольта Алессандро (18.02.1745-05 03.1827) — итальянский физик, изобретатель гальванического элемента. Родился в Комо, близ Милана. Учился в школе ордена иезуитов. C раннего детства интересовался естественными науками, в частности изучением электрических явлений. В 1774-1779 гг преподавал физику в родном городе, с 1779 г. работал профессором Павийского университета, а в 1815-1819 гг. был деканом философского факультета Падуйского университета.

Электрический ток - определение и понятия с примерами
Алессандро Вольта

В 1769 г. А. Вольта опубликовал работу о лейденской банке, а в 1775 г. — об изобретении им смоляного электрофора — прибора, ставшего прообразом электрофорной машины. В 1781 г. сконструировал чувствительный электроскоп с соло минками и ввёл его в практику измерений. А. Вольта повторил и развил опыты Л. Гальвани. В результате многочисленных экспериментов, проведённых в 1792-1794 гг., пришёл к выводу, что явления, которые наблюдал Л. Гальвани, связаны с наличием цепи из двух разнородных металлов и жидкости. Чтобы доказать свою правоту, А. Вольта полностью исключил физиологические объекты, заменив лапку лягушки своим электрометром. Описывая эти опыты, Вольта впервые ввёл понятие замкнутой цепи электрического тока.

А. Вольта были проведены значительные исследования в области химии. В области физиологии он впервые показал, что нервы животных и человека обладают большой электрической возбудимостью.

Условия существования электрического тока

Электрическим током называется направленное движение заряженных частиц, которые являются носителями тока. Вещества, проводящие электрический ток, называют проводниками.

Проводниками являются металлы в твердом и жидком состояниях, жидкие растворы и расплавы электролитов, а также ионизованные газы.

Носителями тока в металлах являются свободные электроны, в электролитах — положительные и отрицательные ионы, в ионизированных газах — ионы и электроны.

Устройство, поддерживающее постоянную разность потенциалов, называют источником тока.

Сочетание источника тока, нагрузки и соединительных проводов называют электрической цепью.

1 А — сила такого неизменяющегося тока, который при прохождении по двум прямым параллельным проводникам бесконечной длины и пренебрежимо малого радиуса, расположенным на расстоянии 1 м друг от друга в вакууме, вызывает между этими проводниками силу взаимодействия, равную Электрический ток - определение и понятия с примерамиН на каждый метр длины проводников.

До 1800 г. развитие электротехники основывалось на получении статического заряда с помощью трения, что позволяло достигать довольно высоких значений потенциалов на непродолжительное время. Однако после создания в начале XIX в. первых источников тока изучение электрических явлений вступило в новую фазу — начались интенсивные исследования законов постоянного тока.

Начало новой эпохи в изучении электричества связано с именем Алессандро Вольта, который изобрел первую электрическую батарею — вольтов столб (рис. 103).

Электрический ток - определение и понятия с примерами

Вольта назвал свой аппарат «двигателем электрической жидкости» или «электродвижущим аппаратом» и указал на возможность увеличения электрических эффектов при увеличении числа элементов батареи.
20 марта 1800 г. Вольта написал знаменитое письмо с описанием своего столба, состоящего из 20 цинковых, 20 медных и 20 суконных кружочков, смоченных серной кислотой, положенных друг на друга (см. рис. 103).

Жизнь современного общества невозможно представить без электричества, поскольку оно практически повсеместно используется в быту и народном хозяйстве. Сегодня электроэнергия необходима для работы станков и машин, освещения, приготовления пищи, а также для работы многочисленных бытовых приборов и устройств: от холодильника до мобильного телефона.

При работе каждого из этих устройств используются различные свойства электрического тока, проходящего в замкнутой цепи.

В металле при отсутствии электрического поля свободные электроны участвуют в хаотическом тепловом движении, подобно тому как движутся молекулы газов и жидкостей. Скорости электронов при хаотическом движении отличаются как по модулям, так и по направлениям. При тепловом движении заряженных частиц не наблюдается направленного переноса заряда через выделенное поперечное сечение проводника, поскольку при хаотическом движении все направления равновероятны.

Однако если приложить к металлическому проводнику напряжение, то внутри него возникнет электрическое поле, которое будет перемещать свободные электроны вдоль проводника в направлении положительного полюса источника тока. Вследствие этого возникнет направленное (упорядоченное) движение заряженных частиц от одного полюса источника тока к другому. Заметим, что именно направленное движение заряженных частиц, происходящее на фоне их теплового (хаотического) движения, приводит к переносу электрического заряда через произвольное поперечное сечение проводника.

Направление упорядоченного движения положительно заряженных частиц в замкнутой цепи условились считать направлением электрического тока. Поскольку свободные положительно заряженные частицы под действием поля всегда движутся в сторону убывания потенциала, то электрический ток в замкнутой цепи направлен от положительного полюса ( + ) источника к отрицательному ( —).

Движение заряженных частиц внутри проводника нельзя наблюдать непосредственно, но судить о наличии электрического тока можно по его действиям:

  1. тепловому (проводник, по которому проходит ток, нагревается);
  2. химическому (химический состав проводника, по которому проходит ток, изменяется);
  3. магнитному (электрический ток создает в окружающем пространстве магнитное поле);
  4. световому (прохождение тока в газах вызывает их свечение).

Подчеркнем, что магнитное действие тока проявляется у всех проводников без исключения, а тепловое у сверхпроводников отсутствует.

По химическому действию электрического тока на проводники они делятся на два класса. Проводники первого класса — те, проходя через которые ток не вызывает химических действий (металлы, уголь). Ко второму классу относятся проводники, при прохождении тока через которые их химический состав изменяется (электролиты).

По типу носителей тока проводники разделяются на три рода. В проводниках первого рода (металлы и полупроводники) носителями тока являются электроны, а в проводниках второго (электролиты) — положительные и отрицательные ионы. В проводниках третьего рода (ионизованные газы) носителями тока являются электроны и ионы.

Заметим, что наличие носителей тока (свободных электрических зарядов) в проводнике не является достаточным условием для возникновения и существования электрического тока. Все рассмотренные действия электрического тока в замкнутой цепи возможны благодаря работе источника тока, осуществляющего непрерывное разделение зарядов при помощи сторонних сил, имеющих неэлектростатическую (некулоновскую) природу.

Таким образом, для продолжительного существования электрического тока в замкнутой цепи необходимо выполнение следующих условий:

  1. наличие свободных заряженных частиц (носителей тока);
  2. наличие электрического поля, силы которого, действуя на заряженные частицы, заставляют их двигаться упорядоченно;
  3. наличие источника тока, внутри которого сторонние силы перемещают свободные заряды против направления электростатических (кулоновских) сил.

Количественной характеристикой электрического тока является сила тока.
 

Сила тока I — физическая скалярная величина, равная отношению заряда Электрический ток - определение и понятия с примерами прошедшего за промежуток времени Электрический ток - определение и понятия с примерами через поперечное сечение проводника, к этому промежутку:
Электрический ток - определение и понятия с примерами

Как следует из определения, сила тока численно равна заряду, проходящему через выделенное поперечное сечение проводника в единицу времени.
Электрический ток называют постоянным, если его сила и направление не изменяются с течением времени.

При постоянном электрическом токе за любые равные промежутки времени через произвольное поперечное сечение проводника проходят одинаковые заряды. При этом направление упорядоченного движения заряженных частиц остается неизменным.

Единицей силы тока в СИ является ампер (I А). Это основная единица в СИ, определяемая экспериментально посредством измерения силы взаимодействия параллельных проводников с током в вакууме.

При силе тока один ампер через любое поперечное сечение проводника за одну секунду проходит заряд один кулон Электрический ток - определение и понятия с примерами

Для характеристики распределения электрического тока по поперечному
сечению проводника, расположенному перпендикулярно направлению движения зарядов, вводится понятие плотности тока.

Плотностью тока Электрический ток - определение и понятия с примерами называется векторная физическая величина, модуль которой равен отношению силы тока I к площади S поперечного сечения проводника:
Электрический ток - определение и понятия с примерами

Направление вектора плотности токаЭлектрический ток - определение и понятия с примерами совпадает с направлением вектора скорости Электрический ток - определение и понятия с примерами упорядоченного движения положительных носителей тока.
Как следует из определения плотности тока, ее модуль численно равен силе тока, проходящего через перпендикулярное поперечное сечение единичной площади.

Выразим силу тока I в проводнике поперечным сечением S через среднюю скорость дрейфа (направленного движения) Электрический ток - определение и понятия с примерами свободных зарядов под действием внешнего электрического поля напряженностью Электрический ток - определение и понятия с примерами (рис. 104).

Электрический ток - определение и понятия с примерами

За промежуток времени Электрический ток - определение и понятия с примерами заряженные частицы пройдут вдоль проводника расстояние Следовательно, через поперечное сечение S проводника за рассматриваемый промежуток времени Электрический ток - определение и понятия с примерами пройдут заряженные частицы, содержащиеся в цилиндре объемом V = Sl.

Если концентрация (число частиц в единице объема Электрический ток - определение и понятия с примерами) свободных заряженных частиц в проводнике n, то заряд Электрический ток - определение и понятия с примерами прошедший через поперечное сечение проводника за время Электрический ток - определение и понятия с примерами найдем как
Электрический ток - определение и понятия с примерами

где q — заряд одной частицы.

Подставив найденный заряд Электрический ток - определение и понятия с примерами в формулу для силы тока, получим
Электрический ток - определение и понятия с примерами
Соответственно, модуль плотности тока определится соотношением

Электрический ток - определение и понятия с примерами

Для вектора плотности тока Электрический ток - определение и понятия с примерами получим выражение Электрический ток - определение и понятия с примерами

Отметим, что с помощью полученной формулы можно оценить скорость дрейфа свободных носителей тока при остальных известных параметрах:
Электрический ток - определение и понятия с примерами

Единицей плотности тока в СИ является ампер на метр квадратный Электрический ток - определение и понятия с примерами При плотности тока Электрический ток - определение и понятия с примерами через поперечное сечение проводника площадью Электрический ток - определение и понятия с примерамипроходит ток силой 1 А.

Для тока силой I = 1 А в проводнике сечением S= 1 Электрический ток - определение и понятия с примерами средняя скорость направленного движения электронов Электрический ток - определение и понятия с примерами а средняя скорость теплового (хаотического) движения электронов при комнатной температуре Электрический ток - определение и понятия с примерами

Силу тока измеряют амперметром. Он включается в цепь последовательно с тем элементом цепи, силу тока в котором нужно измерить. Условное обозначение амперметра (А) на электрических схемах показано на рисунке 105.

Электрический ток - определение и понятия с примерами

На практике используются амперметры (А), миллиамперметры (mA), микроамперметры Электрический ток - определение и понятия с примерами имеющие различную чувствительность и разные диапазоны.

Поскольку включение амперметра в электрическую цепь не должно сильно изменять силу тока в ней, то сопротивление амперметра должно быть как можно меньше. Обычно сопротивление амперметра составляет величину порядка 0,1 Ом, что, как правило, мало по сравнению с суммарным сопротивлением элементов, включенных в цепь.
 

Электрический ток опасен для жизни! Ток силой около 1 мА, проходящий через тело человека, вызывает неприятные ощущения, а при силе тока выше 5 мА возникают болевые ощущения. Ток силой свыше 10 мА вызывает резкое сокращение мышц, при этом может произойти остановка дыхания. При прохождении тока силой свыше 70 мА начинается беспорядочное сокращение мышц сердца (фибрилляция), которая может привести к смерти.

Как следует из закона Ома, при фиксированном напряжении сила тока зависит от электрического сопротивления тела человека. Если кожа человека сухая, то это сопротивление находится в пределах от Электрический ток - определение и понятия с примерамиОм до Электрический ток - определение и понятия с примерамиОм. При влажной коже сопротивление тела человека может значительно уменьшиться (до значений порядка Электрический ток - определение и понятия с примерамиОм).

Следовательно, при контакте с проводом, находящимся под напряжением 120 В, через тело человека пройдет ток силой 120 мА, что чрезвычайно опасно для жизни.

В то же время токи небольшой силы широко применяются при медицинских процедурах, таких, как высокочастотное прогревание, электростатический душ, электроанестезия, электросон, помогая человеку в борьбе с болезнью.

Условия возникновения электрического тока

Изучая физику в предыдущих классах, вы узнали, что такое электрический ток, каковы его основные свойства и закономерности, где и как он применяется. Напомним, что под электрическим током понимают направленное движение заряженных частиц или тел. Ток сопровождается определенными физическими действиями: магнитными, тепловыми, химическими и т. д. По этим действиям можно легко определить наличие электрического тока в той или иной среде.

Для создания электрического тока необходимы определенные условия:

  1. наличие свободных носителей электрических зарядов;
  2. аличие в среде причин, которые могут двигать заряженные частицы в определенном направлении, например -электрическое или магнитное поле, ускоренное движение проводника.

Условия возникновения электрического тока:

  • —    наличие свободных носителей электрических зарядов;
  • —    существование в среде причин направленного движения частиц.

Рассмотрим закономерности прохождения тока в проводнике, который является частью электрической цепи.

Для перемещения частиц должна быть определенная разность потенциалов, т. е. на концах проводника должны быть различные потенциалы. Тогда носители заряда будут двигаться от конца с большим потенциалом к концу с меньшим потенциалом.

Чтобы это происходило постоянно, между концами проводника должна существовать постоянная разность потенциалов. Выполнение этого условия обеспечивают источники тока, работающие за счет преобразования различных видов энергии в электрическую. Так, в гальванических элементах и аккумуляторах происходит преобразование энергии химических взаимодействий, в термогенераторах — тепловой энергии, в электромеханических генераторах — механической. Кроме источника тока, электрические цепи содержат и другие элементы, соединенные проводниками: различные потребители электрической энергии, измерительные и регулирующие приборы, выключатели.

Электрические цепи содержат источники тока, потребители измерительные и регулирующие приборы, выключатели.

Электрический ток характеризуют физической величиной, называемой силой тока. Она характеризует скорость движения заряженных частиц в проводнике и численно равна отношению перенесенного частицами заряда ΔQ к интервалу времени ∆t, на протяжении которого произошел этот перенос:

Электрический ток - определение и понятия с примерами

Ток называется постоянным, если за любые равные интервалы времени через поперечное сечение проводника произошло перенесение одинаковых зарядов.

Международной системе единиц (СИ) единицей силы тока является ампер: Электрический ток - определение и понятия с примерами

Для измерения силы тока применяются также кратные и долевые единицы:
1 микроампер = 1 мкА = 10-6, А;
1 миллиампер — 1 мА = 10-3 А;
1 килоампер = 1 кА = 103 А.

Электрический ток - определение и понятия с примерами Ом Георг Симон (1787-1854) — немецкий физик, учитель математики и физики, член-корреспондент Берлинской АН. Исследовал электрический ток и явления, ему сопутствующие. Установил зависимость между силой тока и напряжением на участке цепи, названную в его честь законом Ома.

В 1826-1827 гг. немецкий физик Георг Ом открыл закон связи между силой тока, напряжением и сопротивлением участка цепи:
сила тока а однородном участке цепи прямо пропорциональна напряжению и обратно пропорциональна сопротивлению проводника
Электрический ток - определение и понятия с примерами
где I — сила тока; U — напряжение на участке цепи; R — сопротивление проводника.

Сила тока в однородном участке цепи прямо пропорциональна напряжению и обратно пропорциональна сопротивлению проводника: Электрический ток - определение и понятия с примерами

Вспомним, что напряжение U — это энергетическая характеристика электрического поля, которая равна работе А поля по перемещению единичного электрического заряда Q по данному участку цепи:
Электрический ток - определение и понятия с примерами

В СИ она измеряется в вольтах (В).
На практике используются кратные и долевые единицы напряжения:

  • 1 милливольт = 1 мВ = 10-3 В;
  • 1 микровольт — 1 мкВ = 10-6 В;
  • 1 киловольт = 1 кВ = 103 В;
  • 1 мегавольт = 1 MB = 106 В.

Сопротивление как физическая величина является характеристикой электрических свойств проводника и зависит от геометрических и физических его параметров. Природа электрического сопротивления объясняется взаимодействием движущихся носителей заряда с другими структурными элементами проводника, в частности с ионами кристаллической решетки.
Формула закона Ома позволяет рассчитать сопротивление проводника:

Электрический ток - определение и понятия с примерами

Если при напряжении 1 B в проводнике проходит ток 1 А, то сопротивление этого проводника составляет 1 Ом.

Электрический ток - определение и понятия с примерами

Кратные и долевые единицы электрического сопротивления:
1 миллиом = 1 мОм = 10-3 Ом;
1 килоом = 1 кОм = 103 Ом;
1 мегаом — 1 МОм — 106 Ом.

Сопротивление проводника также можно рассчитать, зная его вещество и геометрические размеры:

Электрический ток - определение и понятия с примерами

где р удельное сопротивление вещества проводника; I — длина проводника; S — площадь поперечного сечения проводника.
 

Работа и мощность тока

Энергия электрического тока в цепи может превращаться в другие виды энергии: тепловую, химическую, механическую, световую и т. п. Любое это превращение происходит при выполнении работы.

Работа, выполняемая полем при перенесении частиц с общим зарядом ∆Q по участку цепи, определяется по формуле:
Электрический ток - определение и понятия с примерами

Поскольку ΔQ = IΔt, то для определения работы можно применить формулу:

Электрический ток - определение и понятия с примерами

Работа электрического тока, как и любая другая работа, измеряется в джоулях (Дж). Таким образом

1 Дж = 1 В • 1 А • 1 с = 1 В • А • с.

Для характеристики способности выполнять работу применяют понятие «мощность». Мощность равна работе, выполняемой за единицу времени:
Электрический ток - определение и понятия с примерами

В СИ единицей мощности является ватт (Вт).
Электрический ток - определение и понятия с примерами

Для электрического тока 1 Bт=1 B ∙ 1 A = 1 B ∙ A.

Для измерения мощности электрического тока применяют также кратные и долевые единицы:

  • 1 микроватт = 1 мкВт = 10-6 Вт;
  • 1 милливатт = 1 мВт = 10-3 Вт;
  • 1 гектоватт = 1 гВт = 102 Вт;
  • 1 киловатт = 1 кВт = 103 Вт;
  • 1 мегаватт = 1 МВт = 106 Вт.

Одним из наиболее применяемых на практике действий тока является его тепловое действие. На нем основаны различные тепловые электрические приборы — электрические утюги, водонагреватели, электрочайники, обогреватели и т. п. Количество теплоты, выделяемое проводником при прохождении электрического тока за определенное время, определяется законом Джоуля-Ленца:
Электрический ток - определение и понятия с примерами

где I — сила тока; R — сопротивление проводника; Δt — интервал времени.

Электродвижущая сила источника тока

Ранее мы установили, что для возникновения электрического тока в цепи необходимо создать па концах проводника разность потенциалов и поддерживать ее длительное время.

Это условие может быть выполнено, если в электрической цепи будет источник тока, который за счет своих внутренних взаимодействий будет выполнять работу по разделению электрических зарядов. Эти взаимодействия получили название сторонних сил, поскольку имеют неэлектростатическое происхождение.

Сторонние силы выполняют работу по разделению электрических зарядов в электрической цепи. Они имеют неэлектростатическое происхождение.

Так, в гальванических элементах эта энергия возникает вследствие химических реакций между разнородными веществами. В солнечных батареях заряды разделяются вследствие взаимодействия фотонов с атомами вещества. В электрофорной машине разделение зарядов происходит при выполнении механической работы при вращении дисков.

Поэтому для характеристики способности источника тока создавать разность потенциалов используют понятие электродвижущей силы — ЭДС.

Электродвижущей силой источника Электрический ток - определение и понятия с примерами называют физическую величину, которая характеризует способность сторонних сил создавать и поддерживать разность потенциалов в цепи. Она равна отношению работы сторонних сил Aстор к значению разделенных зарядов ∆Q.

Электрический ток - определение и понятия с примерами

Электродвижущая сила является характеристикой источника тока и не зависит от того, какую внешнюю нагрузку присоединяют к его полюсам.

Как и напряжение, она измеряется в вольтах (В).
Рассмотрим более детально роль источника токи в электрической цепи (рис. 1.50).
Электрический ток - определение и понятия с примерами
Рис. 1.50. Сомкнутая электрическая цепь

Во внешней цепи положительные носители заряда будут двигаться от полюса А к полюсу В. Сторонние силы для поддержания постоянной разности потенциалов внутри источника будут разделять электрические заряды за счет работы сил неэлектростатический природы. Таким образом, в источнике тока будет проходить ток, сила которого будет такой же, как и но внешней цепи. Поэтому источник тока будет иметь определенное сопротивление, определяющее силу тока в цепи. Это сопротивление называют внутренним сопротивлением, источника тока.

Учитывая наличие источника тока, в цепи можно выделить внутреннюю и внешнюю часть. Общее сопротивление цепи будет равно сумме сопротивлений внешней и внутренней частей.

Правила безопасного пользования электрическими приборами

Жизнь современного человека сильно связана с использованием электрических приборов и установок. Тело человека является хорошим проводником электрического тока, который может вызвать в человеческом теле опасные и необратимые изменения. Человек, подвергшийся действию электрического тока, получает не только значительные ожоги, но и нарушения в работе нервной системы. Даже при сравнительно небольшом напряжении человек может получить серьезные поражения организма, если он находится, например, во влажном помещении, на влажной земле, касается проводов обеими руками и т. п.

Чтобы предотвратить поражение электрическим током, необходимо придерживаться определенных правил.

  1. Не пользоваться неисправными электроприборами и оборудованием.
  2. Не пользоваться электрическими приборами (утюги, фены, плойки, обогреватели) в ванных комнатах и влажных помещениях.
  3. Не играть под линиями электропередач.
  4. Экспериментируя с электрическими установками, необходимо использовать источники тока с ЭДС не выше 36 В.
  5. Выполняя лабораторные работы, включать электрические установки только после проверки и разрешения на то учителя.
  6. Не касаться частей электроустановок с поврежденной изоляцией.
  7. Если товарищ поражен электрическим током, то в первую очередь необходимо, придерживаясь правил безопасности, отсоединить его от источника тока, вызвать врача и по возможности оказать пострадавшему первую медицинскую помощь.

Электрический ток в различных средах

Согласно определению электрический ток — это направленное движение заряженных частиц. Таким образом, создать электрический ток можно только в той среде, где есть свободные заряженные частицы — носители электрического заряда.

Создать поток таких частиц, которые будут двигаться в одном направлении, можно по-разному. Это может быть действие электрического поля, изменение магнитного поля, проявление инерции. В средах, где заряженные частицы связаны с молекулами или атомами и нс могут свободно двигаться, электрический ток практически невозможен. Среды, в которых есть свободные заряженные частицы, в свое время назвали проводниками — веществами, проводящими электрический ток.

Электрический ток в металлах

Решающим в исследованиях природы электрического тока в металлах был опыт, осуществленный в 1916 г. американскими физиками Р. Толмеиом и Т. Стюартом. В опыте использовалась катушка с большим количеством витков медного провода, которая могла вращаться вокруг своей продольной оси. Выводы обмотки катушки через подвижные
контакты соединялись с чувствительным гальванометром (рис. 1.57).

Электрический ток - определение и понятия с примерами
Рис. 157. Схема опыта Толмена и Стюарта

При быстром вращении катушки стрелка гальванометра была неподвижной. Но при резкой остановке катушки стрелка отклонялась от нулевого деления, фиксируя кратковременный ток в цепи. Тщательные рассчеты показали, что частицами, образующими ток, были электроны. Получив большую скорость при вращении катушки, электроны некоторое время после остановки катушки продолжали двигаться но инерции, образуя ток. Поскольку полученный в опыте ток ничем не отличался от электрического тока в обычной цепи, то был сделан вывод об электронной при роде .электрического тока в металлах.

В кристаллической структуре металла электроны находятся в свободном состоянии, совершая только хаотическое тепловое движение. Если появляется электрическое поле, то электроны начинают смещаться в направлении силовых линий поля, создавая электрический ток (рис. 1.58).

Электрический ток - определение и понятия с примерами
Pиc. 158. Движение электронов в металле

При движении между узлами кристаллической решетки металла электроны взаимодействуют с ионами и отдают им часть энергии, полученной под действием электрического поля. Полученная ионами энергия увеличивает амплитуду их колебаний, что проявляется как тепловое действие электрического тока. Отдавая часть энергии ионам, иле «троны уменьшают скорость своего упорядоченного движения, что служит ни видом утверждать, что существует сопротивление проводника. Если проводник нагревать от внешнего источника тепла, то увеличивается амплитуда тепловых колебании ионов и степень их взаимодействия с электронами. Поэтому сопротивление металлического проводника при повышении температуры увеличивается. Типичным свойством металлических проводников является то, что в большом интервале температур изменение их сопротивления происходит пропорционально изменению температуры.

Исследованиями голландского физика Камерлинга-Онесса установлено, что сопротивление металлического проводника почти исчезает при температуре, близкой к абсолютному нулю. Исследуя зависимость электрических свойств ртути от температуры, он заметил, что при температуре 4,12 К не сопротивление практически равно нулю. Состояние проводника, в котором он не имеет сопротивления, назвали сверхпроводимостью. Возбужденный в таком проводнике электрический ток может существовать довольно длительное время.

В состоянии сверхпроводимости проводники практически полностью теряют электрическое сопротивление.

Сверхпроводимость интересует ученых и инженеров, которые видят в ней средство для создания новых экономичных технологий. В первую очередь это касается передачи без потерь электроэнергии на большие расстояния, создания сверхмощных электромагнитов для научных исследований, разработки принципиально новых сверхмощных компьютерных систем и т. п.

Электрический ток в вакууме

В вакууме, в котором практически отсутствуют свободные носители электрического заряда, создать электрический ток можно при условии искусственного введения их в ограниченное пространство, в котором существует электрическое поле. Свободные заряженные частицы в вакууме можно получить при нагревании металлических электродов (термоэлектронная эмиссия) или при облучении их электромагнитным излучением (фотоэлектронная эмиссия).

Свободные электроны, появляющиеся при эмиссии, накапливаются возле электрода, из которого они эмитировали, и образуют электронное облачко. Если температура электрода поддерживается постоянной или поток электромагнитного излучения не изменяется, то электроны находятся в динамическом равновесии, при котором количество электронов, вылетающих из электрода, равно количеству электродов, возвращающихся в него.

Если создать электрическое поле, присоединив, например, к электроду, из которого вылетают электроны, отрицательный полюс источника тока, а к другому электроду — положительный полюс, то электроны начнут двигаться упорядоченно.

Ток в вакууме создают свободные электроны эмиссии.

Этим потоком можно управлять с помощью электрического или магнитного полей, что ученые и использовали для создания вакуумных электронных приборов.

Например, при помощи электрического поля поток электронов в вакууме можно сформировать в узкий пучок, управляя которым можно создавать различные изображения на экране электронно-лучевой трубки или дисплея, которые применяются в электронных осциллографах или в некоторых моделях телевизоров (рис. 1.59).

Электрический ток - определение и понятия с примерами
Рис. 159. Электроннолучевая трубка

Электрический ток в жидкостях

Если в стакан с дистиллированной водой опустить два электрода, входящих в электрическую цепь, то тока в цепи не будет. Аналогичный результат получим, если электроды опустим в керосин, спирт или другую чистую жидкость. Это свидетельствует о том, что в таких жидкостях отсутствуют свободные носители электрического заряда и они тока не проводят.

Кел и повторить опыт с водой, влив в нее небольшое количество серной кислоты, то полученный раствор будет иметь свойства проводника (рис. 1.60).

Электрический ток - определение и понятия с примерами
Рис. 1.60. Исследование электрического тока в растворе электролита в воде

В растворах и расплавах электролитов ток образуют положительные и отрицательные ионы.

Какова же природа свободных носителей заряда в растворе кислоты? На уроках химии вы узнали, что при растворении в воде кислот, щелочей или солей (электролитов) происходит электролитическая диссоциация. Сложные молекулы электролита распадаются на положительные и отрицательные ионы, которые при наличии электрического поля будут двигаться вдоль линий напряженности этого поля (рис. 1.61).

Электрический ток - определение и понятия с примерами
Pиc. 1.61. Схема движения ионов в растворе электролита под действием электрического поля

Положительные ионы (катионы) будут двигаться к отрицательному электроду, а отрицательные (анионы) — к положительному. Т. е. в растворе электролита будет возникать электрический ток. На электродах ионы нейтрализуются и оседают в виде нейтральных атомов или молекул.

Ток в растворах и расплавах электролитов сопровождается выделением вещества на электродах.

Таким образом, при прохождении тока в растворах электролитов происходит выделение вещества на электродах. Это явление используется для рафинирования металлов, добывания алюминия, покрытия поверхности деталей машин .защитными материалами, изготовления копий деталей сложной формы и т. п.

Электрический ток в газах

В обычном состоянии нее газы являются диэлектриками (непроводниками электричества). Известно, что при размыкании электродов выключателя ток в цепи прекращается, хотя между контактами будет существовать разность потенциалов. Это объясняется тем, что все газы в нормальном состоянии имеют нейтральные молекулы, которые не могут образовать направленного потока под действием электрического ноля. Но несмотря на это при определенных условиях и в газах может возникнуть электрический ток.

Например, если взять дне металлические пластины, между которыми находится воздух, и присоединить их через гальванометр к источнику тока (рис. 1.62), то при обычных условиях тока в цепи не будет, поскольку воздух не проводит электричество.
Электрический ток - определение и понятия с примерами
Рис. 1.62. Установка для исследования электропроводности воздуха

Если же в промежуток между пластинами внести пламя спиртовки, то стрелка гальванометра покажет наличие тока в цепи (рис. 1.62). Об этом явлении в физике принято говорить как о несамостоятельном разряде в газе. Несамостоятельный разряд будем наблюдать и тогда, когда спиртовку заменим специальной лампой — источником ультрафиолетового излучения. Образование электрического тока в газах объясняется тем, что под действием пламени или электромагнитного излучения про-исходит ионизация, вследствие чего появляются свободные электроны. Соединяясь с нейтральными молекулами, они могут образовывать отрицательные ионы. Таким образом, в пространстве между пластинами в нашем опыте кроме электронов находятся также нейтральные молекулы, положительные и отрицательные ионы.

Электрический ток в газах — это поток отрицательных и положительных ионов и свободных электронов.

Под действием электрического поля заряженные частицы начинают упорядоченно двигаться, образуя электрический ток. Он прекратится, если убрать ионизатор.

Если напряжение между электродами постепенно увеличивать, то в определенный момент энергия носителей зарядов становится достаточной, чтобы, сталкиваясь с нейтральными молекулами, образовывать новые ионы и свободные электроны. Процесс приобретает лавинообразный характер, когда количество свободных носителей заряда резко увеличивается. В этот момент в цепи резко увеличивается сила тока, а также температура газа, и он начинает излучать свет.

Электрический разряд в газе возможен и без наличия ионизатора. Ведь в воздухе всегда находятся заряженные частицы, образующиеся под действием космического и радиоактивного излучения, ультрафиолетового излучения Солнца и т. п. При значительной разности потенциалов (большая напряженность электрического поля) они приобретают большую скорость и энергию, достаточную для ионизации нейтральных молекул. В таких условиях возникает самостоятельный разряд.

В природе самостоятельными разрядами являются молния (рис. 1.63), коронный и тлеющий разряды.

Электрический ток - определение и понятия с примерами
Pиc. 1.63. Молния

Среди различных видов электрического разряда особое место занимает электрическая дуга, которую применяют для сварки металлических изделий (рис. 1.64).

Электрический ток - определение и понятия с примерами
Pиc. 1.64. Сварка металла при помощи электрической дуги

Если два проводника, соединенных с источником тока, свести вместе, то в месте их касания начнет выделяться значительное количество теплоты при резком увеличении температуры. Oro приведет к появлению в газе значительного количества свободных электронов, которые будут поддерживать ток в цепи. При силе тока в несколько сотен и даже тысяч ампер температура в дуге может достигать 3500…7000 К. В таком состоянии газ ярко светится, а электроды — плавятся.

Электрическая дуга широко применяется во многих областях техники и производства для сварки (соединения) металлических деталей, для плавки чистых металлов. Всему миру известны достижения ученых Института электросварки им. Е.О. Патона HAH Украины. Благодаря их исследованиям процесс электросварки применяется в различных производствах, включительно с космической отраслью.

Электрический ток - определение и понятия с примерами Евгений Оскарович Патон (1870-1953) — украинский инженер, основатель Института электросварки HAH Украины, провел исследования в области расчета и прочности сварных конструкций, механизации сварочных процессов, научных основ электрической сварки.
  • Электрические цепи постоянного тока
  • Методы анализа сложных электрических цепей
  • Метод узловых напряжений
  • Метод узловых потенциалов 
  • Синтез электрических цепей
  • Цепи с распределенными параметрами
  • Электрическая энергия, ее свойства и применение
  • Электрическая цепь

Напряжение показывает, какую работу совершает электрическое поле при перемещении единичного положительного заряда из одной точки в другую.

U=Aq, где (U) — напряжение, (А) — работа тока, (q) — электрический заряд.

Таким образом:

Напряжение на концах участка цепи численно равно работе, которая совершается при прохождении по этому участку электрического заряда в (1) Кл.

При прохождении по этому же участку электрического заряда, равного не (1) Кл, а, например, (10) Кл, совершённая работа будет в (10) раз больше.

Это означает, что, чтобы определить работу электрического тока на каком-либо участке цепи, надо напряжение на концах этого участка цепи умножить на электрический заряд, прошедший по нему: A=U⋅q.

Для выражения любой из величин можно использовать приведённые ниже рисунки.

Работа и мощность тока - Справочник студентаЭлектрический заряд, прошедший по участку цепи, можно определить, измерив силу тока и время его прохождения: q=I⋅t. Используя это соотношение и подставляя его в формулу A=U⋅q, получим формулу для нахождения работы электрического тока: A=U⋅I⋅t.

Работа электрического тока на участке цепи равна произведению напряжения на концах этого участка на силу тока и на время, в течение которого совершалась работа.

Чтобы выразить любую из величин из данной формулы, можно воспользоваться рисунком.

Работа и мощность тока - Справочник студента

Как известно, работу измеряют в джоулях, напряжение — в вольтах, силу тока — в амперах, а время — в секундах.

Тогда  1 джоуль = 1 вольт · 1 ампер · 1 секунду, или 1 Дж = 1 В · А ·С.

Из вышесказанного следует, что для измерения работы электрического тока нужны вольтметр, амперметр и часы.

Например, для определения работы, которую совершает электрический ток, проходя по спирали лампы накаливания, необходимо собрать цепь, изображённую на рисунке.

Вольтметром измеряется напряжение на лампе, амперметром — сила тока в ней. А при помощи часов (секундомера) засекается время горения лампы.

Работа и мощность тока - Справочник студентаРабота и мощность тока - Справочник студента

Например:

I = 1,2 АU = 5 Вt = 1,5 мин = 90 сА = U⋅I⋅t = 5⋅1,2⋅90 = 540 Дж 

Обрати внимание!

Работа чаще всего выражается в килоджоулях или мегаджоулях.

(1) кДж = 1000 Дж или (1) Дж = (0,001) кДж;(1) МДж = 1000000 Дж или (1) Дж = (0,000001) МДж.

На практике работу электрического тока измеряют специальными приборами — счётчиками. Счётчики электроэнергии можно видеть в каждом доме.

Работа и мощность тока - Справочник студента

Из курса физики известно, что мощность численно равна работе, совершённой в единицу времени: N = Аt. Следовательно, чтобы найти мощность электрического тока, надо его работу, A=U⋅I⋅t, разделить на время.

В отличие от механической мощности мощность тока обозначают буквой (Р):

P=At=U⋅I⋅tt=U⋅I. Отсюда следует:

Мощность электрического тока равна произведению напряжения на силу тока: P=U⋅I.

Из этой формулы можно определить и другие физические величины.Для удобства можно использовать приведённые ниже рисунки.

Работа и мощность тока - Справочник студента

  • За единицу мощности принят ватт: (1) Вт = (1) Дж/с.
  • Из формулы P=U⋅I следует, что
  • (1) ватт = (1) вольт х (1) ампер, или (1) Вт = (1) В ∙ А.

Обрати внимание!

Используют также единицы мощности, кратные ватту: гектоватт (гВт), киловатт (кВт), мегаватт (МВт).(1) гВт = (100) Вт или (1) Вт = (0,01) гВт;(1) кВт = (1000) Вт или (1) Вт = (0,001) кВт;

(1) МВт = (1 000 000) Вт или (1) Вт = (0,000001) МВт.

Измерить мощность электрического тока можно с помощью вольтметра и амперметра.

Работа и мощность тока - Справочник студента

Чтобы вычислить искомую мощность, необходимо напряжение умножить на силу тока. Значение силы тока и напряжение определяют по показаниям приборов.

I=1,2АU=5ВP =U⋅I=5⋅1,2=6Вт.

Существуют специальные приборы — ваттметры, которые непосредственно измеряют мощность электрического тока в цепи. Они бывают аналоговые и цифровые. В зависимости от сферы применения у них различаются пределы измерения.

Подключим к цепи по очереди две лампочки накаливания, сначала одну, затем другую и измерим силу тока в каждой из них. Она будет разной.

Сила тока в лампочке мощностью (25) ватт будет составлять (0,1) А. Лампочка мощностью (100) ватт потребляет ток в четыре раза больше — (0,4) А. Напряжение в этом эксперименте неизменно и равно (220) В.

Легко можно заметить, что лампочка в (100) ватт светится гораздо ярче, чем (25)-ваттовая лампочка. Это происходит оттого, что её мощность больше.

Лампочка, мощность которой в (4) раза больше, потребляет в (4) раза больше тока. Значит: 

Обрати внимание!

Мощность прямо пропорциональна силе тока.

Что произойдёт, если одну и ту же лампочку подсоединить к источникам различного напряжения? В данном случае используется напряжение (110) В и (220) В.

Можно заметить, что при большем напряжении лампочка светится ярче, значит, в этом случае её мощность будет больше. Следовательно:

Обрати внимание!

Мощность зависит от напряжения.

Рассчитаем мощность лампочки в каждом случае:

I=0,2АU=110ВP=U⋅I=110⋅0,2=22Вт I=0,4АU=220ВP=U⋅I=220⋅0,4=88Вт.

Можно сделать вывод о том, что при увеличении напряжения в (2) раза мощность увеличивается в (4) раза.

Не следует путать эту мощность с номинальной мощностью лампы (мощность, на которую рассчитана лампа).

Номинальная мощность лампы (а соответственно, ток через нить накала и её расчётное сопротивление) указывается только для номинального напряжения лампы (указано на баллоне, цоколе или упаковке).

В таблице дана мощность, потребляемая различными приборами и устройствами:

Название Рисунок Мощность
 Калькулятор (0,001) Вт
 Лампы дневного света (15 — 80) Вт
 Лампы накаливания (25 — 5000) Вт
 Компьютер (200 — 450) Вт
 Электрический чайник (650 — 3100) Вт
 Пылесос (1500 — 3000) Вт
 Стиральная машина (2000 — 4000) Вт
 Трамвай (150 000 — 240000) Вт

Источники:

Пёрышкин А.В. Физика, 8 класс// ДРОФА, 2013.

http://уроки.мирфизики.рф/%d1%80%d0%b0%d0%b1%d0%be%d1%82%d0%b0-%d0%b8-%d0%bc%d0%be%d1%89%d0%bd%d0%be%d1%81%d1%82%d1%8c-%d1%8d%d0%bb%d0%b5%d0%ba%d1%82%d1%80%d0%b8%d1%87%d0%b5%d1%81%d0%ba%d0%be%d0%b3%d0%be-%d1%82%d0%be%d0%ba/

http://phscs.ru/physicsus/electric-power

http://class-fizika.narod.ru/8_34.htm

Источник: https://www.yaklass.ru/p/fizika/8-klass/elektricheskie-iavleniia-12351/rabota-i-moshchnost-toka-12367/re-aa44330a-39ec-4dd0-a3d2-d70ed142a191

Решение задач по теме: «Работа и мощность электрического тока». Видеоурок. Физика 8 Класс

  • Работа – произведение силы тока, напряжения и времени, в течение которого протекает электрический ток.
  • Мощность – отношение работы ко времени, в течение которого протекает электрический ток.

Работа и мощность тока - Справочник студента

Из закона Ома получили эквивалентные формулы.

Условие задачи:

«В течение 10 мин по некоторому участку протекает электрический ток, значение которого – 250 мА. Напряжение на этом участке – 4 В. Необходимо определить мощность электрического тока, который выделяется на этом участке, и работу электрического тока, произведенную за это время».

Краткое условие задачи и решение

Комментарий к решению:

10 минут – это время протекания электрического тока. Напряжение на концах участка цепи – 4 В. Сила тока определяется как 250 мА (миллиамперметры). 1 мА = 0,001 А.

  1. Переведем все значение в интернациональную систему (СИ):
  2. t = 10 мин = 10∙60 с = 600 с;
  3. І = 250 мА = 250∙0,001 А = 0,25 А.
  4. U = 4 В (так как вольт (в системе СИ) – международная единица)
  5.  Первое уравнение – это вычисление работы.

Работа и мощность тока - Справочник студента

  • Получаем ответ: А=600 Дж.
  • Существует 2 варианта определения мощности:
  • 1.     Зная, что работа равна 600 Дж, а время протекания тока – 600 с, определяем мощность по формуле, или

Работа и мощность тока - Справочник студента

Ответ: А = 600 Дж; Р = 1 Вт

Условие задачи:

 « Две лампы мощностью 25 Вт и 100 Вт включаем в электрическую цепь под напряжением 220 В. Насколько отличается сила тока в этих лампах?»

Краткое условие и решение задачи:

Комментарий к решению:

І означает, что мы должны найти разность сил тока в одной лампе и в другой. Из формулы для вычисления мощности  выражаем силу тока в первой лампе и во второй. Получаем, что в лампе мощностью 100 Вт протекает электрический ток в 0,45 А, в лампе с мощностью 25 Вт сила тока будет 0,11 А. Следовательно, І=0,45-0,11=0,34 А.

Лампа, которая обладает большей мощностью, будет гораздо ярче светить. Это значит, что чем больше электрический ток протекает в цепи, тем ярче будет гореть лампа. Можно заметить, что мощность первой лампы в 4 раза больше второй, тем самым в 4 раза больше и сила тока. Мощность, работа, сила тока, напряжение – величины, которые между собой связаны и характеризуют действие электрического тока.

Список литературы

  1. Генденштейн Л.Э, Кайдалов А.Б., Кожевников В.Б. / Под ред. Орлова В.А., Ройзена И.И. Физика 8. – М.: Мнемозина.
  2. Перышкин А.В. Физика 8. – М.: Дрофа, 2010.
  3. Фадеева А.А., Засов А.В., Киселев Д.Ф. Физика 8. – М.: Просвещение.

Дополнительные рекомендованные ссылки на ресурсы сети Интернет

  1. Electrono.ru (Источник).
  2. Utrew.hut.ru (Источник).
  3. Stoom.ru (Источник).

Домашнее задание

  1. П. 50–52, вопросы 1–6 стр. 121, 1–3 стр. 122, задание 26 (1). Перышкин А.В. Физика 8. – М.: Дрофа, 2010.
  2. Определите мощность электрического камина, спираль которого имеет сопротивление 500 Ом и потребляет ток 2 А.
  3. С помощью каких формул можно определить работу и мощность электрического тока?

Источник: https://interneturok.ru/lesson/physics/8-klass/belektricheskie-yavleniyab/reshenie-zadach-po-teme-rabota-i-moschnost-elektricheskogo-toka

Электрический ток. Работа и мощность в цепи постоянного тока. Закон Ома для полной цепи

Работа и мощность тока - Справочник студентаКристаллическая решётка

Электрический ток.        Все металлы являются проводниками электрического тока. Они состоят из пространственной кристаллической решетки, узлы которой совпадают с центрами положительных ионов. Вокруг ионов хаотически движутся свободные электроны.

В металлах электронная проводимость

Электрическим током в металлах называется упорядоченное движение свободных электронов. За направление тока принимают направление движения положительно заряженных частиц.

Электрические заряды могут двигаться упорядоченно под действием электрического поля, поэтому условием для существования эл. тока является наличие электрического поля и свободных носителей эл.заряда.

     Сила тока численно равна заряду, протекающему через данное поперечное сечение проводника в единицу времени.   Ток называется постоянным, если сила тока и его направление не изменяется с течением времени.

Работа и мощность тока - Справочник студентаI = 1 Кл/с = 1 А

1 ампер (А) равен силе постоянного тока, при котором через любое поперечное сечение проводника за 1 с протекает 1 Кл электричества.                 I = q0 nvS                                             Силу тока в цепи измеряют амперметром.  Условное обозначение в цепи

Работа и мощность тока.      Электрический ток снабжает нас энергией. Она возникает за счёт работы электрического поля по передвижению свободных зарядов в Работа и мощность тока - Справочник студентапроводнике. Рассмотрим участок цепи, по которому течёт ток I. Напряжение на участке обозначим U, сопротивление участка равно R. При протекании тока по однородному участку цепи электрическое поле совершает работу. За время Δt по цепи протекает заряд Δq = I Δt. Электрическое поле на выделенном участке совершает работу.  ΔA  = U I Δt  — эту работу называют работой электрического тока. За счёт работы на рассматриваемом участке может совершаться механическая работа; могут также протекать химические реакции. Если этого нет, то работа эл.поля приводит только к нагреванию проводника. Работа тока равна количеству теплоты, выделяемому проводником с током:  — закон Джоуля — Ленца  

Мощность электрического тока равна отношению работы тока ΔA к интервалу времени Δt, за которое эта работа была совершена на данном участке:    P = IU  или                Работа и мощность тока - Справочник студента.  Работа электрического тока в СИ выражается в джоулях (Дж), мощность – в ваттах (Вт).

Закон Ома для замкнутой цепи.          Источник тока имеет ЭДС () и сопротивление (r), которое называют внутренним. Работа и мощность тока - Справочник студентаЭлектродвижущей силой (ЭДС) называется отношение работы сторонних сил по перемещению заряда q вдоль цепи, к значению этого заряда  (1В=1Дж/1Кл). Рассмотрим теперь замкнутую (полную) цепь постоянного тока, состоящую из источника с электродвижущей силой  и внутренним сопротивлением r и внешнего однородного участка с сопротивлением R.  (R+r) — полное сопротивление цепи.  Закон Ома для полной цепи записывается в виде   или Работа и мощность тока - Справочник студента

Сила тока в электрической цепи прямо пропорциональна ЭДС источника тока и обратно пропорциональна полному сопротивлению цепи.

Источник: http://kaplio.ru/elektricheskij-tok-rabota-i-moshhnost-v-tsepi-postoyannogo-toka-zakon-oma-dlya-polnoj-tsepi/

Работа и мощность электрического тока

Урок разработан с целью организации исследовательской деятельности учащихся, направленной на формирование понятия «работа тока», «мощность», причин от которых зависит работа тока и мощность. Ознакомление учащихся с расчетными формулами, единицами измерения.

Цель урока:

  • Деятельностная: формирование способности учащихся к новому способу действия, расширение понятийной базы за счет введения нового понятия.
  • Образовательная: организация исследовательской деятельности учащихся, направленной на формирование понятия «работа тока», «мощность», причин от которых зависит работа тока и мощность. Ознакомление учащихся с расчетными формулами, единицами измерения.

Планируемые результаты:

  • умеет объяснять понятия «работа тока», «мощность»;
  • умеет находить «работа тока», «мощность»;
  • умеет объяснять проделанные эксперименты.

Оборудование:

  • Приборы для определения мощности в электрической цепи и для определения параметров, от которых зависит работа тока в электрической цепи: аккумулятор, ключ, амперметр, реостат, вольтметр, соединительные провода; лампочка, калькулятор.
  • Компьютер с мультимедийным проектором, экран;
  • Мультимедийное приложение к уроку.

Учебник: Перышкин А.В. «Физика – 8»

Ход урока

1. Организационный момент

Учитель: Здравствуйте. Посмотрите друг на друга, улыбнитесь. Сегодня на уроке я предлагаю вам побывать в роли сотрудников научной лаборатории. Работать вы будете в своих лабораториях по 2 человека и группами.

Результаты вашей работы необходимо будет отмечать в индивидуальных оценочных листах, которые лежат у вас на столе (приложение 1). В конце урока мы подведем итог вашей работы по накопленным баллам.

2. Постановка цели и задач урока

Как настоящие мастера своего дела засучили рукава, хлопнули в ладоши, потерли их друг о друга.

Учитель: Что мы с вами сделали? (Совершили работу). Значит в названии темы нашего урока мы используем слово «работа».

У вас на столе лежит лампочка. Внимательно рассмотрите её.

  • Когда вы покупаете лампочку, то обязательно смотрите…Что?
  • Так как звучит тема нашего урока?
  • Ученик: работа тока и мощность.

Учитель: Запишите тему. Работа и мощность электрического тока. (слайд 1)

  1. Учитель: Определите цели нашей работы, используя слова узнать …., учиться…, узнать:
  2. – как вычислить работу электрического тока и мощность тока;
  3. учиться:
  4. – объяснять проделанные эксперименты;
  5. – объяснять понятия «работа тока», «мощность тока»;
  6. Учитель: Скажите, зачем нам нужно знать о работе электрического тока, мощности? (слайд 2)(бытовая техника) (Оцените свою работу на этом этапе)

3. Актуализация знаний (слад 3)

Но прежде, чем приступить к работе, вам необходимо убедиться в том, что сотрудник лаборатории обладает достаточной базой знаний, чтобы участвовать в проведении экспериментальной и исследовательской работы.

Учащимся предлагается самостоятельная работа «Установите соответствие»

Работа и мощность тока - Справочник студента

Слайд 4 Проверка

А Б В Г Д Е Ж З
4 7 1 3 2 8 5 6

Взаимопроверка в парах, выставление баллов в оценочный лист. За каждый правильный ответ 1 б.

4. Открытие нового знания

Вспомним: Как обозначается работа? (А)

Давайте подумаем: от чего же зависит работа электрического тока? (на доске написать: Работа электрического тока зависит от …)

Слайд 5. Сравните два рисунка. На рисунке 2 лампа светит ярче, чем на рисунке 2.

Значит, работа тока зависит от напряжения.

Вывод: (записываем вывод на доску и в тетрадь)

Слайд 6. При увеличении силы тока лампа светит ярче.

Значит, работа тока зависит от силы тока.

Вывод:

Слайд 7. Если лампа будет работать 1 час, большую работу совершит ток? (Да)

Значит, работа тока зависит от времени работы цепи..

Вывод:

Выведем формулу для расчета работы электрического тока.

Работа и мощность тока - Справочник студента

  • Слайд 8
  • Вывод: работа электрического тока на участке цепи равна произведению напряжения на концах этого участка на силу тока и на время, в течение которого совершалась работа.
  • Нарисуйте «волшебный треугольник»

Работа и мощность тока - Справочник студента

  1. Проверьте, правильно ли это сделали? (1 б)
  2. Слайд 9. Для того чтобы измерить работу тока в цепи, нужны следующие приборы:
  3. а) Вольтметр
  4. б) Амперметр
  5. в) Часы (прибор для измерения времени) (в любой квартире – счётчик)

Вспомним, в чем измеряется работа? [А ] = [ Дж ] = [ А .В .c]

  • Какой величиной характеризуется быстрота выполнения работы?
  • (мощностью: N=A/t )
  • Слайд 10
  • Мощность электрического тока обозначается P.
  • Выведем формулу мощности электрического тока

Работа и мощность тока - Справочник студента

Нарисуйте «волшебный треугольник»

Работа и мощность тока - Справочник студента

1 б

Вывод: мощность электрического тока равна произведению напряжения на силу тока.

Слайд 11, 12. Для измерения мощности нужны: амперметр и вольтметр – это сочетается в ваттметре.

Работа с учебником. Таблица «Мощности различных электрических устройств, кВт». Стр.120

Рассмотрите таблицу и сравните мощности устройств, применяемых в быту, технике, на производстве.

Практически на всех электроприборах, используемых в быту и технике, в техническом паспорте указывается мощность тока, на которую они рассчитаны. Зная мощность, легко можно определить работу тока за заданный промежуток времени: Как по другому можно записать формулу работы?

  1. A =P∙t.(1 б)
  2. Слайд 13
  3. Тогда
  4. 1Дж = 1Вт ∙ с.

Однако эту единицу работы неудобно использовать на практике, так как в потребителях электроэнергии ток производит работу в течение длительного времени, например в бытовых приборах – в течение нескольких часов, в электропоездах – даже в течение нескольких суток. Поэтому на практике, вычисляя работу тока, удобнее время выражать в часах, а работу не в джоулях, а в других единицах: ватт ∙ час (Вт ∙ ч) и кратных им единицах.

1 Вт · ч = 3 600Дж

1 к Вт · ч = 1 000 Вт · ч = 3 600 000 Дж

А знаете ли вы, что…значение экономии электроэнергии велико для народного хозяйства страны? Например, 1 кВт ∙ ч энергии позволяет выплавить около 20 кг чугуна.

Оцените свою работу

5. Самостоятельная работа, закрепление материала

Необходимо восстанавливать свои силы. Давайте проведём физминутку, а поможет нам в этом ЭЛЕКТРОНИК.

  • Работаем в группах. Практические задания:
  • Слайд 14 1) Определение работы тока и мощности тока в лампе.
  • Слайд 15 2) Какой телевизор затратит больше электроэнергии
  • Решение: А=Рt
  • A1=24 Вт•3ч=72 Вт•ч=0,072•2.49=0,18 руб

А2=50•3=150Вт•ч=0,15•2.49=0,37руб.

На каждом из электроприборов вы можете найти информацию о потребляемой энергии и выбрать себе наиболее экономный в этом плане прибор.

Слайд 16. 3) Знакомство со счетчиком.

Вывод. Знание физических величин нужно не только на уроках физики, но и при покупке и использовании электрических приборов.

6. Итог урока

  1. Подсчитайте количество баллов и оцените свою работу на уроке.
  2. Если вы набрали за урок:
  3. Более 20 баллов, вы сегодня были в роли ученика-исследователя
  4. 14-19 баллов, вы сегодня были в роли ученика-теоретика
  5. 11-13 баллов, вы сегодня были просто учеником

7. Домашнее задание

Слайд 17

§ 50,51.

Узнайте мощности имеющихся у вас в квартире электроприборов (телевизора и холодильника). Посчитайте, сколько часов они работают в течение 1 дня. Вычислите стоимость электроэнергии, израсходованной ими за это время.

8. Рефлексия

  • Слайд 18
  • Заполнить лист рефлексии.
  • Похлопаем в ладоши.

Источник: https://rosuchebnik.ru/material/rabota-i-moshchnost-elektricheskogo-toka-20548/

1.11. Работа и мощность тока

При протекании тока по однородному участку цепи электрическое поле совершает работу. За время Δt по цепи протекает заряд Δq = I Δt. Электрическое поле на выделенном учестке совершает работу

ΔA = (φ1 – φ2) Δq = Δφ12 I Δt = U I Δt,

где U = Δφ12 – напряжение. Эту работу называют работой электрического тока.

  • Если обе части формулы выражающей закон Ома для однородного участка цепи с сопротивлением R, умножить на IΔt, то получится соотношение
  • Это соотношение выражает закон сохранения энергии для однородного участка цепи.
  • Работа ΔA электрического тока I, протекающего по неподвижному проводнику с сопротивлением R, преобразуется в тепло ΔQ, выделяющееся на проводнике.

Закон преобразования работы тока в тепло был экспериментально установлен независимо друг от друга Дж. Джоулем и Э. Ленцем и носит название закона Джоуля–Ленца.

  1. Мощность электрического тока равна отношению работы тока ΔA к интервалу времени Δt, за которое эта работа была совершена:

    Работа и мощность тока - Справочник студента

  2. Работа электрического тока в СИ выражается в джоулях (Дж), мощность – в ваттах (Вт).
  3. Рассмотрим теперь полную цепь постоянного тока, состоящую из источника с электродвижущей силой и внутренним сопротивлением r и внешнего однородного участка с сопротивлением R. Закон Ома для полной цепи записывается в виде
  4. Умножив обе части этой формулы на Δq = IΔt, мы получим соотношение, выражающее закон сохранения энергии для полной цепи постоянного тока:

    R I2Δt + r I2Δt =  IΔt = ΔAст.

  5. Первый член в левой части ΔQ = R I2Δt – тепло, выделяющееся на внешнем участке цепи за время Δt, второй член ΔQист = r I2Δt – тепло, выделяющееся внутри источника за то же время.
  6. Выражение  IΔt равно работе сторонних сил ΔAст, действующих внутри источника.

При протекании электрического тока по замкнутой цепи работа сторонних сил ΔAст преобразуется в тепло, выделяющееся во внешней цепи (ΔQ) и внутри источника (ΔQист).

Следует обратить внимание, что в это соотношение не входит работа электрического поля. При протекании тока по замкнутой цепи электрическое поле работы не совершает; поэтому тепло производится одними только сторонними силами, действующими внутри источника. Роль электрического поля сводится к перераспределению тепла между различными участками цепи.

Внешняя цепь может представлять собой не только проводник с сопротивлением R, но и какое-либо устройство, потребляющее мощность, например, электродвигатель постоянного тока. В этом случае под R нужно понимать эквивалентное сопротивление нагрузки.

Энергия, выделяемая во внешней цепи, может частично или полностью преобразовываться не только в тепло, но и в другие виды энергии, например, в механическую работу, совершаемую электродвигателем.

Поэтому вопрос об использовании энергии источника тока имеет большое практическое значение.

Полная мощность источника, то есть работа, совершаемая сторонними силами за единицу времени, равна

Работа и мощность тока - Справочник студента

Во внешней цепи выделяется мощность

Работа и мощность тока - Справочник студента

Отношение равное

Работа и мощность тока - Справочник студента

называется коэффициентом полезного действия источника.

На рис. 1.11.

1 графически представлены зависимости мощности источника Pист, полезной мощности P, выделяемой во внешней цепи, и коэффициента полезного действия η от тока в цепи I для источника с ЭДС, равной , и внутренним сопротивлением r. Ток в цепи может изменяться в пределах от I = 0 (при ) до (при R = 0).

Рисунок 1.11.1.Зависимость мощности источника Pист, мощности во внешней цепи P и КПД источника η от силы тока

Из приведенных графиков видно, что максимальная мощность во внешней цепи Pmax, равная достигается при R = r. При этом ток в цепи а КПД источника равен 50 %.

Максимальное значение КПД источника достигается при I → 0, т. е. при R → ∞.

В случае короткого замыкания полезная мощность P = 0 и вся мощность выделяется внутри источника, что может привести к его перегреву и разрушению. КПД источника при этом обращается в нуль.

Лучшие школы, лагеря, ВУЗы за рубежом

Источник: https://physics.ru/courses/op25part2/content/chapter1/section/paragraph11/theory.html

Работа и мощность постоянного тока — Класс!ная физика

  • «Физика — 10 класс»
  • Вспомните, как определяется работа кулоновских сил при перемещении заряда q из точки с потенциалом φ1 в точку с потенциалом φ2.
  • При упорядоченном движении заряженных частиц в проводнике электрическое поле совершает работу.
  • Её принято называть работой тока.

Рассмотрим произвольный участок цепи. Это может быть однородный проводник, например нить лампы накаливания, обмотка электродвигателя и др. Пусть за время Δt через поперечное сечение проводника проходит заряд Δq.

Электрическое поле совершит при этом работу А = ΔqU (U — напряжение между концами участка проводника).

А = IUΔt.         (15.12)

  1. Работа тока на участке цепи равна произведению силы тока, напряжения и времени, в течение которого шёл ток.
  2. Согласно закону сохранения энергии эта работа должна быть равна изменению энергии рассматриваемого участка цепи. Поэтому
  3. энергия, выделяемая на данном участке цепи за время Δt, равна работе тока.

Если на участке цепи не совершается механическая работа и ток не производит химических действий, то происходит только нагревание проводника, т. е. увеличивается внутренняя энергия проводника. Нагретый проводник отдаёт тепло окружающим телам.

Нагревание проводника происходит следующим образом. Электрическое поле ускоряет электроны. В результате столкновения с ионами кристаллической решётки они передают ионам свою энергию. Энергия беспорядочного движения ионов около положений равновесия возрастает.

Это и означает увеличение внутренней энергии. Так как температура — мера кинетической энергии тела, то температура проводника повышается, и он начинает передавать тепло окружающим телам.

Спустя некоторое время после замыкания цепи процесс устанавливается, и температура проводника перестаёт изменяться со временем. За счёт работы электрического поля в проводнике непрерывно выделяется энергия.

Но его внутренняя энергия остаётся неизменной, так как проводник передаёт окружающим телам количество теплоты, равное работе тока. Таким образом, формула (15.12) для работы тока определяет количество теплоты, передаваемой проводником другим телам.

Если в формуле (15.12) выразить либо напряжение через силу тока, либо силу тока через напряжение с помощью закона Ома для участка цепи, то получим три эквивалентные формулы

Работа и мощность тока - Справочник студента

Формулой A = I2RΔt удобно пользоваться при последовательном соединении проводников, так как сила тока в этом случае одинакова во всех проводниках. При параллельном соединении удобна формула так как напряжение на всех проводниках одинаково.

Закон Джоуля—Ленца.

Закон, определяющий количество теплоты, которую выделяет проводник с током в окружающую среду, был впервые установлен экспериментально английским учёным Д. Джоулем (1818—1889) и русским учёным Э. X. Ленцем (1804— 1865).

Количество теплоты, выделяемой в проводнике с током, равно произведению квадрата силы тока, сопротивления проводника и времени прохождения тока по проводнику

Q = I2RΔt.         (15.14)

Мы получили этот закон с помощью рассуждений, основанных на законе сохранения энергии. Формула (15.14) позволяет вычислить количество теплоты, выделяемой на любом участке цепи, содержащем какие угодно проводники.

Мощность тока.

Любой электрический прибор (лампа, электродвигатель и т. д.) рассчитан на потребление определённой энергии в единицу времени. Поэтому наряду с работой тока очень важное значение имеет понятие мощность тока.

Мощность тока равна отношению работы тока ко времени прохождения тока. Согласно этому определению мощность тока

Работа и мощность тока - Справочник студента

Электрическая мощность, так же как и механическая, выражается в ваттах (Вт).

Это выражение для мощности тока можно переписать в нескольких эквивалентных формах, используя закон Ома для участка цепи:

Работа и мощность тока - Справочник студента

  • На большинстве электроприборов указана потребляемая ими мощность, предельное значение силы тока, а также предельное значение напряжения.
  • В быту для расчётов потребляемой электроэнергии часто используется единица кВт • ч, 1 кВт • ч = 3,6 • 106 Дж.
  • Источник: «Физика — 10 класс», 2014, учебник Мякишев, Буховцев, Сотский

Следующая страница «Электродвижущая сила» Назад в раздел «Физика — 10 класс, учебник Мякишев, Буховцев, Сотский»

Законы постоянного тока — Физика, учебник для 10 класса — Класс!ная физика

Электрический ток. Сила тока — Закон Ома для участка цепи. Сопротивление — Электрические цепи. Последовательное и параллельное соединения проводников — Примеры решения задач по теме «Закон Ома.

Последовательное и параллельное соединения проводников» — Работа и мощность постоянного тока — Электродвижущая сила — Закон Ома для полной цепи — Примеры решения задач по теме «Работа и мощность постоянного тока.

Закон Ома для полной цепи»

Источник: http://class-fizika.ru/10_a163.html

Работа и мощность электрического тока

16 августа 2013. Категория: Электротехника.

Работа электрического тока

Работа и мощность тока - Справочник студента
Рисунок 1. Работа и мощность электрического тока

К цепи, представленной на рисунке 1, приложено постоянное напряжение U.

U = φА – φБ

За время t по цепи протекло количество электричества Q. Силы электрического поля, действующего вдоль проводника, перенесли за это время заряд Q из точки А в точку Б. Работа электрических сил поля или, что то же, работа электрического тока может быть подсчитана по формуле:

  • A = Q × (φА – φБ) = Q × U,
  • Так как Q = I × t, то окончательно:
  • A = U × I × t,
  • где A – работа в джоулях; I – ток в амперах; t – время в секундах; U – напряжение в вольтах.
  • По закону Ома U = I × r. Поэтому формулу работы можно написать и так:
  • A = I 2 × r × t.

Мощность электрического тока

  1. Работа, произведенная в единицу времени, называется мощностью и обозначается буквой P.
  2. Из этой формулы имеем:
  3. A = P × t.
  4. Единица измерения мощности:

1 (Дж/сек) иначе называется ваттом (Вт). Подставляя в формулу мощности выражение для работы электрического тока, имеем:

P = U × I (Вт).

Формула мощности электрического тока может быть выражена также через потребляемый ток и сопротивление потребителя:

  • Кроме ватта, на практике применяются более крупные единицы измерения электрической мощности. Электрическая мощность измеряется в:
  • 100 Вт = 1 гектоватт (гВт); 1000 Вт = 1 киловатт (кВт);
  • 1000000 Вт = 1 мегаватт (МВт).

Электрическая мощность измеряется специальным прибором – ваттметром. Ваттметр имеет две обмотки (катушки): последовательную и параллельную.

Последовательная катушка является токовой и включается последовательно с нагрузкой на участке цепи, где производятся измерения, а параллельная катушка – это катушка напряжения, она соответственно включается параллельно этой нагрузке.

 Принцип действия ваттметра основан на взаимодействии двух магнитных потоков создаваемых током, протекающим по обмотке подвижной катушки (токовой катушки), и током, проходящим по неподвижной катушке (катушке напряжения).

При прохождении измеряемого тока по обмотке подвижной и неподвижной катушек образуются два магнитных поля, при взаимодействии которых подвижная катушка стремится расположится так, чтобы направление ее магнитного поля совпадало с направлением магнитного поля неподвижной катушки.

Вращающему моменту противодействует момент, созданный спиральными пружинками, через которые в подвижную катушку проводится измеряемый ток. Противодействующий момент пружинок прямо пропорционален углу поворота катушки. Стрелка, укрепленная на подвижной катушке, указывает значение измеряемой величины. Схема включения ваттметра показана на рисунке 2.

Рисунок 2. Схема включения ваттметра

Если вы решили измерить потребляемую мощность, какой либо имеющейся у вас нагрузки, и при этом у вас отсутствует ваттметр, вы можете «изготовить» ваттметр своими руками.

Из формулы P = I × U видно, что мощность, потребляемую в сети, можно определить, умножив ток на напряжение. Поэтому для определения мощности, потребляемой из сети, следует использовать два прибора, вольтметр и амперметр.

Измерив амперметром потребляемый ток и вольтметром напряжение питающей сети, необходимо показание амперметра умножить на показание вольтметра.

  1. Так, например, мощность, потребляемая сопротивлением r, при показании амперметра 3 А и вольтметра 220 В будет:
  2. P = I × U = 3 × 220 = 660 Вт.
  3. Для практических измерений электрической работы (энергии) джоуль является слишком мелкой единицей.
  4. Если время t подставлять не в секундах, а в часах, то получим более крупные единицы электрической энергии:
  5. 1 Дж = 1 Вт × сек; 1 Вт × ч = 3600 ватт × секунд = 3600 Дж; 100 Вт × ч = 1 гектоватт × час (гВт × ч);
  6. 1000 Вт × ч = 1 киловатт × час (кВт × ч).
  7. Электрическая энергия измеряется счетчиками электрической энергии.
  8. Видео 1. Работа и мощность электрического тока
  • Видео 1. Работа и мощность электрического тока

Видео 2. Еще немного о мощности

Пример 1. Определить мощность, потребляемую электрическим двигателем, если ток в цепи равен 8 А и двигатель включен в сеть напряжением 220 В.

P = I × U = 8 × 220 = 1760 Вт = 17,6 гВт = 1,76 кВт.

Пример 2. Какова мощность, потребляемая электрической плиткой, если плитка берет из сети ток в 5 А, а сопротивление спирали плитки равно 24 Ом?

P = I 2 × r = 25 × 24 = 600 Вт = 6 гВт = 0,6 кВт.

При переводе механической мощности в электрическую и обратно необходимо помнить, что 1 лошадиная сила (л. с.) = 736 Вт;

1 киловат (кВт) = 1,36 л. с.

Пример 3. Определить энергию, расходуемую электрической плиткой мощностью 600 Вт в течение 5 часов.

A = P × t = 600 × 5 = 3000 Вт × ч = 30 гВт × ч = 3 кВт × ч

Пример 4. Определить стоимость горения двенадцати электрических ламп в течение месяца (30 дней), если четыре из них по 60 Вт горят по 6 часов в сутки, а остальные восемь ламп по 25 Вт горят по 4 часа в сутки. Цена за энергию (тариф) 2,5 рубля за 1 кВт × ч.

  1. Мощность четырех ламп по 60 Вт.
  2. P = 60 × 4 = 240 Вт.
  3. Число часов горения этих ламп в месяц:
  4. t = 6 × 30 = 180 часов.
  5. Энергия, расходуемая этими лампами:
  6. A = P × t = 240 × 180 = 43200 Вт × ч = 43,2 кВт × ч.
  7. Мощность остальных восьми ламп по 25 Вт.
  8. P = 25 × 8 = 200 Вт.
  9. Число часов горения этих ламп в месяц:
  10. t = 4 × 30 = 120 часов.
  11. Энергия, расходуемая этими лампами:
  12. A = P × t = 200 × 120 = 24000 Вт × ч = 24 кВт × ч.
  13. Общее количество расходуемой энергии:
  14. 43,2 + 24 = 67,2 кВт × ч
  15. Стоимость всей потребленной энергии:
  16. 67,2 × 2,5 = 168 рублей.

Источник: Кузнецов М.И., «Основы электротехники» — 9-е издание, исправленное — Москва: Высшая школа, 1964 — 560с.

Источник: https://www.electromechanics.ru/electrical-engineering/541-after-the-electric-current-and-power.html

Работа и мощность тока

Практическая работа № 8.

«Работа и мощность тока».

I) Прочитайте § 106 учебника (Мякишев Г. Я. Физика. 10 класс: учеб. для общеобразоват. учреждений — М.: Просвещение, 2012).

II) Составьте конспект параграфа 57 по плану:

    Дайте характеристику физической величины работа тока по плану:

    • Определение;
    • Формула;
    • Единица измерения.

    Дайте характеристику физической величины мощность тока по плану:

    1. Определение;
    2. Формула;
    3. Единица измерения.

    Дайте характеристику закона Джоуля — Ленца по плану:

    • Формулировка;
    • Математическая запись;
    • Границы применимости;
    • Пример проявления (применения).

III) Прочитайте задачу № 802 (Рымкевич, А.П. Физика. Задачник. 10 – 11 классы.: пособие для общеобразоват. учреждений – 17-е изд. — М.: Дрофа, 2013).

  1. IV) Разберите и перепишите в тетрадь решение первой части задачи № 806:
  2. 3, 5 В – это напряжение, под которым работает лампочка от карманного фонарика; 0,28 А – это сила тока в лампочке.
  3. Дано: Решение:
  4. U = 3,5 В U U 3,5
  5. I = 0,28 А I = — закон Ома для участка цепи; R = = = 12,5;
  6. Найти: R I 0,28
  7. R, Р — ? В

А

Р = I . U = 3,5 . 0,28 = 0,98;

[Р] = В . А = Дж . А = Дж . А = Дж = Вт

  • Кл А . с с
  • Ответ: Сопротивление лампы 12,5 Ом, мощность лампы 0,98 Вт.
  • V) Решите вторую часть задачи № 802.
  • VI) Объясните, почему при одинаковой силе тока сетевая лампа выделяет мощность больше, чем лампочка от карманного фонарика.
  • Практическая работа № 8DOC / 30 Кб

Источник: https://xn--j1ahfl.xn--p1ai/library/rabota_i_moshnost_toka_143620.html

Работа и мощность электрического тока

·  При последовательном включении в электрическую цепь не­скольких источников энергии с различным направлением э. д. с. общая э. д. с. равна алгебраической сумме э. д. с. всех источников. Суммирование э.

д. с. одного направления берут со знаком плюс, а э. д. с. противоположного направления — со знаком минус. При составлении уравнений выбирают направление обхода цепи и про­извольно задаются направлениями токов.

·  Обычно замкнутая цепь является частью сложной цепи, как показано, например, на рис. 21. Замкнутая цепь обозначена буква­ми а, б, в и г. Ввиду наличия ответвлений в точках а, б, в, г токи I1, I2, I3  и I4, отличаясь по силе, могут иметь и различные направле­ния. Для такой цепи в соответствии со вторым законом Кирхгофа можно написать:

  • ·  где r01, r02, r03  — внутренние сопротивления источников энергии,
  • ·     r1, r2, r3 — сопротивления приемников энергии.
  • ·  В частном случае при отсутствии ответвлений и последователь­ном  соединении  проводников  общее  сопротивление равно сумме всех сопротивлений.
  • ·  Если внешняя цепь источника энергии с внутренним сопротив­лением r0 состоит, например, из трех последовательно соединенных резисторов с сопротивлениями, соответственно равными r1, r2, r3, то на основании второго закона Кирхгофа можно написать следующее равенство:

·  При  нескольких  источниках  тока  в левой части этого равенства была бы алгебраическая сумма

·  э. д. с. этих источников.

·  При параллельном включении двух или не­скольких источников энергии токи, протекающие в них, в общем случае неодинаковы.

·  Если два параллельно соединенных источника энергии (рис. 22), имеющих э. д. с. Е1 и Е2 и внутренние сопротивления r1 и r2,. замкнуть на какое-либо внешнее сопротивление r, то силу тока во внешней цепи I и в источниках I1 и I2 можно определить из следующих выражений:

·  Отсюда сила тока во внешней цепи

· 

·  Сила тока, протекающего через первый и второй источники энер­гии,:

·  Пример 1.  В схеме, изображенной на рис. 21, э. д. с. источников энергии и сопротивления  приемников энергии имеют следующие значения: Е1=6 в, Е2=12 в, Е3=9 в, r1=8 ом, r2=5 ом, r3=4 ом и r4=10 ом.Решение. Алгебраическая сумма э. д.с. в цепи

·  В этом выражении э. д. с. Е1 взята со знаком минус потому, что первый источник энергии включен встречно второму и третьему.

·  Общее сопротивление цепи

·  Сила тока в цепи

·  Напряжение между точками а и г

·  Пример 2. Два параллельно соединенных генератора (см. рис. 22), имеющие э. д. с. Е1=Е2=120 в и внутренние сопротивления r1=3 ом и r2=6 ом, замкнуты на сопротивление r=18 ом.

·  Определить силу тока во внешней цепи и токи в первом и во втором гене­раторах.

·  Решение. Внутреннее сопротивление двух параллельно соединенных гене­раторов.

  1. · 
  2. ·  Сила тока во внешней  цепи
  3. · 

·  Токи в первом и во втором генераторах обратно пропорциональны внутрен­ним сопротивлениям этих генераторов, т. е.

  • · 
  • ·  Таким образом, I1 + I2 =3I2 = 6 a,  откуда  I2  = 2 a, I1  = 2I2  = 4 a.
  • ·  § 22. РАБОТА И МОЩНОСТЬ ЭЛЕКТРИЧЕСКОГО ТОКА

·  Способность тела производить работу называется энергию тела.  Например, поднятый на высоту какой-либо груз обладает некоторым запасом энергии и при падении производит работу.

Энергия тела тем больше, чем большую работу может произвести это тело при своем движении. Энергия не исчезает, а переходит из одной формы в другую.

Например, электрическая энергия может быть превращена в механическую, тепловую, химическую, механи­ческая — в электрическую и т. д.

·  Для переноса зарядов  в замкнутой цепи источник электриче­ской энергии затрачивает известную энергию, равную произведению э. д. с. источника на количество электричества, перенесенного через эту цепь, т. е. EQ.

·  Однако не вся эта энергия является полезной, т. е. не вся работа,  произведенная  источником  энергии,  сообщается  приемнику энергии, так как часть ее расходуется на преодоление внутреннего сопротивления  источника  и  проводов.  Таким образом, источник энергии производит полезную работу, равную

  1. · 
  2. ·  где U — напряжение на зажимах приемника.
  3. ·  Так  как  количество  электричества равно произведению силы! тока в цепи на время его прохождения:
  4. · 
  5. ·  формулу работы можно представить в следующем виде
  6. · 

·  т. е. электрическая энергия или работа есть произведение напряже­ния, силы тока в цепи и времени его прохождения.

·  Если же выразить напряжение на зажимах участка цепи как произведение силы тока на сопротивление этого участка, т. е.

  • · 
  • ·  то формулу работы можно записать и таким образом:
  • · 

·  Однако ни одна из указанных формул не определяет размеров генератора электрической энергии, от которого получена эта рабо­та, так как и большой и малый генераторы могут дать одинаковую работу, но в различные промежутки времени.

Поэтому размеры генератора определяются не выполненной работой, а его мощно­стью.

Это относится к любому электротехническому аппарату и ма­шине, хотя бы они были не поставляющими, а  потребляющими электрическую энергию (например, электродвигатели, электриче­ские лампы, нагревательные приборы и т. д.).

Источник: http://fiziku5.ru/uchebnye-materialy-po-fizike/rabota-i-moshhnost-elektricheskogo-toka

Понравилась статья? Поделить с друзьями:
  • Велосипедист проехал 128 км от березина до вязова с одинаковой скоростью первую часть пути
  • Венчурный капитал инвестируется в покупку инновационной компании с целью получения высокой
  • Внутрикорпоративная система электронного бизнеса позволяющая организовать работу персонала
  • Водитель случайно проехал перекресток разрешено ли ему использовать задний ход чтобы затем
  • Восстановите пропущенное слово реквизит это обязательный элемент оформления документа тест